Surgery

¡Supera tus tareas y exámenes ahora con Quizwiz!

Hematuria causes mnemonic

"I": infection, infarction, iatrogenic (drugs) "T": trauma, tumor, TB "S" Stone, sickle cell, cystitis other mnemonic from first aid: I PEE RBCS Infection (UTI) Polycsytic kindney disease Exercise External trauma Renal glomerular disease BPH Cancer Stones

Types of colostomies

*Minor problems encounteered with colostomies are irregularity of function, irritation of the skin due leakage of enteric contents, or bleeding from the exposed mucosa following trauma. Most common:Sigmoid colostomy. -can have single or double barrel stomas. Single barrel (end colostomy) is more common. **most serious complication of end colostomies is parastomal herniation. Descending colostomy: -can also have single or double barrel stomas. Single barrel (end colostomy) is more common. Ascending colostomy: Transverse colostomy

What does positive pressure mechanical ventilation do? Indications for it?

*The inability to maintain a Pa02 of 60 or an oxgen sat. of more than 91% with a supplemental non-rebreathing -2 mask is indicative of a significant alveolar-arterial (A-a) gradient, and intubation and mechanical ventilation may be needed. Acutely increases intrathoracic pressure, increasing right atrial pressure and decreaseing systemic venous return. The sudden loss of venous return may cause acute circulatory failure and death. In patients with hypovolvemic shock, this effect may cause ciruclatory collapse if the patient's intravascular volume is not replaced before mechanical ventialtion is attempted. *Also, you can get Ventilator-induced Lung Injuy (VILI). inury can happen secondary to overinflation, distension, and repetitive opening and closing of the alveoli. HIgh oxygen content delivery to the lungs also prodcues oxygen-free-radical associated lung injuries. Ways to minimze VILI include using a low tidal volume, reducing Fi02, and application of positive ende expiratory pressure (PEEP).

Who should get a carotid endarterectomy?

- Patients with symptomatic carotid artery stensos of 70%-99%. -Men w/asymptomatic carodi artery stenosis of 60-99% may also benefit from CEA compared to medical management alone. All patients with cerebrovascular disease should be started on antiplatelty agents and statins with optimizatio of risk factors (smokin cessation, diabetes mellitus, and hypertension control).

Indications for intubation

-Airway protection (patients with GCS<8) -Failure to oxygenate -Failure to ventilate leading to progressive hypercapnia with acidosis and signs of mental status change Mechanical ventilation: goals are to improve gas exchage and decrease work of breathing *you adjust the ventilation based on ABG *minute ventilation =RR x Tidal Volume ---you can adjust rr or tidal volume to change minute ventilation. Know this simple rule: increasing miinute ventilation will decrease PC02 and increase ph. Decreasing will do the opposite. *indication for oxygenation: P02 *indication for ventilation: Pc02

Types of esophageal rupture/perforation

-Boerhaave's syndrome: A full thickness tear. Due to forceful vomiting, cough, trauma. *can occur during episodes of vomiting, particularly when the patient is resisting the vomiting reflex. Cuz high intraabdominal pressures are transmitted into the mediastinal esophagus. -Mallory-Weiss syndrome: A partial-thickness tear. Due to forceful vomiting/wretching. Common presentation is self-limited hematemesis. -Foreign body: like endoscopy can perforate the esophagus. iatrogenic cause is most common cause. *esophageal perforation is a surgical emergency. Signs: pain in chest, dysphagia, dyspnea. SUBCUtANEOUS EMPHYSEMA, mediastinal emphysema hear as a "crunching sounds" with heartbeat. (Hammon's crunch) Pneumomediastinum: air within mediastinal space. Dx: -esophagogram w/ water-soluble contrast (KNOW). Water-soluble contarst is preferred during the initial examination because it causes less mediastinal irritation than barium if a large leak is discovered. -X-ray-left sided PLEURAL EFFUSION (pleural fluid analysis is low ph and very HIGH AMYLASE (cuz of salivary amylase)) , mediastinal or subcutaneous emphysema Tx: Full thickness tear: surgery. Partial thickness: may resolve spontaneously Full thickness tears: surgical closure of full-thickness tears within 24 hours: 80-90% survival rate -drain the contaminated mediastinum -monitor for recovery from sepsis *In patients with and unerlying motility disorder, stricture, or malignancy, surgical intervention must address both the perforation and the esophageal abnormality. For patients with a distal esophageal carcinoma, treatment usually requires esophagectomy. Esophageal exclusion or proximal diverstion (with a cervical esophagostomy or "spit fistula") are typically reserved for patients in whom a late diagnosis of esophageal perforation was made.

Bilateral undescended testes management

-Chorionic gonadotopin is occasinaly effective in these patients. If no testicular descent after 1 month of endocrine therapy, surgical intervention consisting of inguinal orchidopexy should be perforemd before the age of 2. After 2 years of age, a testicle not in the cooler environment of scrotal sac will begin to undergo histologic changed and higher incidence of malignant degeneration.

Dr. Corbridge acidosis/alkalosis lecture during medicine clerkship Acute respiratory acidosis/alkalosis things to remeber

-For every 10 Co2 changes, ph should change in opposite direction by .08 for only respiratory conditions. If you don't see that then (im guessing) you have anohter metabolic process -for acute respiratory stuff, bicarb changes around 1 or 2 for every 10 Co2, .08 ph changes. Bicarb doesn't change drastically so not that important. You're only going to see big changes in bicarb when your kidney and/or GI tract is involved like in metabolic processes. Compensated respiratory acidosis/alkalosis things to remember -Bicarb takes some days to compensate for respiratory process Metabolic things to remember -with metabolic you don't have the acute or compensated stuff to remember as with respiratory. The question is was their effective respiratory compensation? -AG MA: use Winters or that rule that says is Pco2 number matches the number at right side of decimal point, then their was effective compensation. Like 7.20ph and 20 Pco2. remember too its around that area, you don't need exact matching. -General idea of NG MA is bicarb wasting -General idea of AG MA adding ions -Delta Delta: idea is added ions get buffered by bicarb, so bicarb levels should go down by same amount that anion gap is. So the math is AG-10(always 10) =x. then add x to your bicarb you have and if its around 24 then no other process going on. If its considerably over then that means you have extra bicarb and you have a concomittant met. alkalosis going on. If its under 24 then you have a concomitant bicarb wasting state (NG MA) going on. -with ABG they get the bicarb level by calculating it by Henderson Hasselbach. The bicarb in venous samples is the bicarb direclty measured. -every decrease in albumin by 1 your AG should decrease by around 2.5. Hypoalbuminemia is common cause of low AG. With metabolic alkalosis you have Chloride responsive and Chloride unresponsive. -Cloride responsive (saline responsive): Vomiting a big cause. When you vomit you lose Cl-, Na+ and volume so your kidney gonna be grabbing to to Cl-, Na+ so you're not going to see much Cl- in urine. But if you get Cl- deplete your kidneys gonna want an anion to hold on to with the Na+ grabbing so if Cl- not there, its gonna start grabbing Bicarb (it's anion). That's why saline responsive, if you give NaCl then your kidney can pee out bicarb. -For chloride unresponsive the big ones are hypokalemia, and steroid excess. Also..UAG Basically used to decide between RTA or diarrhea as cause of NG Metabolic Acidosis. The idea is cations must equal anions in urine. So UNa + UK+ Unmeasured cations (NH4+ being the important one) = UCl + Unmeasured anions If you rearrange the equation you get... Urine Anion Gap=UNa + UK -UCl = U. anions - U. cations. If UAG>0 that means decreased NH4, so its RTA If UAG <0 that means incraesed NH4 generated in kidneys. Normal or GI loss (diarrhea).

Trauma: thoracic trauma DDx

-PERICARDIAL TAMPONADE -----Remember tamponade characterized by "Beck's triad". distant heart sounds, distended jugular veins, and hypotension. Chest x-ray shows enalred cardiac sillhouette from fluid accumulation. Tachycardia gonna happen to mainatin cardiac output. Also pulsus paradoxus (decrease in systolic BP by more than 10 at end of inspiration). Also, there will be equalization of pressures across all 4 chambers. *Tamponade can be a life-threatening complication that can occur after CABG. Clotting of the mediastinal chest tube followed by hemodynamic decompensation with decreseased MAPs and CO w/increasing filling pressures is suggestive of tamponade. Tx in this case: exploration and drainage of mediastinal hematoma. -TRAUMATIC RUPTURE OF AORTA : (TRA: traumatic rupture of aorta). many of these people die quick but if its a contained rupture the classic X-ray is a left sided hemothorax with widened mediastinum. CT angiography has become the most commonly appplied diagnostic study for TRA identificatio. Tx: endovascular repair is preferred repair for many -PNEUMOTHORAX: air in pleural space. -primary spontaneous pneumothorax: no preceding event or history of lung diseease -secondary spontaneous pneumothorax: complication of underlying lung disease (COPD) -tension pneumotrhoax: life threatening, trapped air with mediastingal shift and compromised cardiopulmoanry function Tx: small (<2cm between lung and bhest wall on x-ray): observation and oxygen large (stable): needle aspiration or chest tube *if spontaneous pneumothorax is recurrent then one can do thoracotomy with bleb excision and pleural abrasion. large (unstable) or TP: urgent needle deocmpression, then chest tube placement (tube thoracostomy) -Tension pneumothorax: life threatening, caused by air entering pleural space (most often via a hole in lung tissue) and unable to escape. you see all sings of pneumothorax + tracheal deviation --------if you have to dx tension pneumothorax by x-ray you've missed it. it's a emergency and you need to dx by physical exam. x-ray is only confirmation. *you get rapid onset servere dypsnea, tachyardia, tachypnea, hypotension, AND distention of neck vens due to SVC compression. *placement of central venous catheters accounts for about 1/4 of iatrogenic pneumothorax. -Hemothorax: blood in chest -----tx: chest tube placement and drainage and control of bleeding -Rib fractures -Pulmonary contusion: hemorrhage into alveolar and interstitial spaces leads to pulmonary contusion. -Myocardial contusion: no universally accepted criteria for diagnosis of myocardial contusion. If clinically suspected, monitor on telemtery for 24 hours.

Colon cancer screening, recommendations for colonic polyp follow up?

-Rectal exam yearly after age 50 years (uworld i think). But MKSAP says annual rectal examination with office FOBT is not considered adeuqate screening cuz of poor sensitivity. -Occult blood in stool yealry after age 50 years. This is the home high-sensitivity FOBT, sampling 2 to 3 consecutive specimens. -Colonoscopy every 8-10 years -Patients with an affected first degree relative should begin screening (colonoscopy) at age 40 or 10 years before the age of relative's diagnosis. *Tumors of the right side are less likely to cause obstructive symptoms until late in the course of the disease. Anemia is the more common presentation in these patients. *Once the colon is cleared of polyps, do a repeat colonoscopy at 3 years and if results are negative, to repeat colonoscopies at 5 eyars.

Diagnoses of stones

-Urinary Ph can help differentiate -X-ray will reveal only radiopaque stones -Intravenous pyelogram is gold standard. It can clearly outline the entire urinary sstyem making it easy to see hydronephrosis and presence of any type of stones.. a. Noncontrast helical abdominal CT has replaced IV pyelography as the gold standard for diagnosing kidney stones. However, noncontrast helical abdominal CT is expensie and has a higher radiation exposure than other imaging studies. IV pyelography has high sensitiviy and specificy for diagnoses of stones, however this study requires bowel perparation and the use of IV iodinated contrast agents, which are contraindicated in patients with acute kidney injury and chronic kidney disease.

D

.

A core needle biopsy is performed. What are 7 biopsy findings for which you should consider an excisional biopsy to r/o occult malignancy.

1. Atypical ductal hyperplasia. 2. Atypial lobular hyperplasia 3. LCIS (if pleomorphic) 4. Radial scar: a sclerosing lesion. shown to be a risk factor but in itself does not progress to malignancy. 5. Intraductal papilloma 6. Flat cell epithelial hyperplasia 7. Discordance between imaging and histology

Secondary hyperparathyroidism

1. Bad kidney-> can't excrete phosphate--> hyperphosphatemia---> this binds calcium and lower free calcium in serum---> PTH incrased---> more PTH releasing calcium from bone 2. Bad kidneu--> can't make calcitriol--> Brain sees this and increases PTH to help kidney out to make calcitrio---> extra PTH releases calcium frome bone 3 types of bone lesions associated with secondaru hyperparathyroidism 1. Oseitis fibrosa cystica: bone replaced by fibrous tissue, woven bone, 2. Osteomalacia: Assoaciated with Vit D deficiency, characterized by defective oseiod mineralization 3. Adynamic bone disease: cause unknown. Tx: goal is to normalize calcium-phosphate balance. Do things like dietary phosophate restriction. Take Vit D with calcitriol. *there is also tertiary hyperparathryoidism which results from chronic renal disease. This is autonomous hypersecrtion of pTH even after correction of hyopcalecmia.

Dialysis types

1. Hemodialysis Two most commonly used vascular access sites are arterovenous fistulae (AVF) usually in the foream and artificial grafts inserted between an artery and a vein. Dialysis needles are placed directly into the graft. A tunneled dialysis catheter (TDC) is usually placed in the right internal jugular vein and serves as a bridge that allows hemodialysis to take place until AVF or graft is palced. 2. There is also peritoneal access. The idea is that the peritenium is used as the membrane for transfer of electrolytes instead of a membrane in a machine.

Layers of abdominal walla

1. Skin, then fat 2. Scarpa's fascia 3. External oblique 4. Internal oblique 5. Transversus abdominis 6. Transversalis fascia 7. Preperitoneal fat 8. Peritoneum

Types of Burns

1st Degree burns Location: Epidermis Characteristic: Erythema and pain Course: Heals in 3-4 days without scarring. Tx: Lotions (like ao) and NSAIDS (anti inflammtory effect) 2nd Degree Burns Location: Through epidermis to the dermis Characteristic: Swelling, blisters, very painful Course: -- Superficial dermal: heal within 3 weeks w/o scarring --Deep dermal: heal in 3-8 weeks, w/scarring Tx:Excise and graft deep dermal burns 3rd degree burns Location: All the way through dermis Characteristic: White or dark, leathery, painless Course: Burns can heal only by epithelial migration from periphery and contraction. Unless they are tiny (cigarretty burn size), they wiill need grafting. Tx: Excise and graft

What do you gotta wory about with smoke inhalation?

1st you woorry about airway edema from irritation from smoke 2nd you worry about cyanide and CO that you absorb. These people should be suspected to have an acute carbon monoxide poisoning and are treated with 100% oxygen via a facemask. Early symptoms are nerulogical and include agitation, confustion, and somnolence. Dz confirmed clinically and by documenting en elevated carboxyhemoglobin level. Pulse ox is unreliable cuz can't differnetiale carboxyhemoglobin from oxyhemogloin. cyanide blocks oxidative phosphorylation and promotes anaerobic metabolish, causing lactic acidosis. with cyanide toxicity, you treat with HYDROXyOCOBALAMIN or SODIUM THIOSULFATe, which directly bind s cyanide milecules. An alternate treatment is induction of methemoglobinemia with nitries to increase Fe3+ in hemoglobin. Cyanide binds avidly to Fe3+ and so methemoglobinemia provides an laternate binding site. *Methemoblobin if Fe3+ iron in hemoglobin. Fe3+ is unable to bind oxygen, and it also increases the affinitiy of Fe2+ to oxygen, causing a LEFT SHIFT in oxygen dissociation curve. Result is functional anemia due to reduced oscygen delivery to tissues. Methemoglobinemia occurs after expsorue to oxidizing agents (DAPSONE, nitrates, TOPICAL/LOCAL ANESTHESTICS)

Renal transplantation

3 most common causes are diabetes, hypertension, glomerular diseases. Currently, patients on the renal transplant waitlist in U.S. outnumber kidney tranplated patients by a ratio 4:1, conseqeuntly, the median time on waitlist prior to receiving transplant has been about 39 months. In U.S., people are considered for renal transplant whne GFR falls below 20 ml/min. In patients with prior history of cancr, must be at least 2 years without evidence of disease. Age is noo longer a contraindication to organ transplant. HIV is no longer a contraidication, providied that is recieiving HAART. Cyclosporin, Tacrolimus, Sirolimus, MMF, and azathioprine used for postrtransplant immune suppression. *Cyclosporine is frequently held for a bit post-op until creatinine returns to normal cuz its a bit nephrotoxic. *Azathioprine has bone marrow toxicity as a major side effect and so both WBC and platelets needs to be monitored post-op If acute graf rejectino occurs ,which after a few weeks to months person gets fever, malaise, heypertension, oliguria, increase in serum creatinine, and tenderness and swelling over transplated kidney. High dose steroids used, if those don't work then antilymphocytic antibodies are often given. Baceterial infections are most cmoon infections after first month following tranplant. Opportunistic stuff like CMV, pneumocystisi jiroveci, toxo, aspergilloisis, nocardiosis, crypto, and blastomycosis. Prophylactic antibioitcs SMP/TMX for 6 months after transplant is good. Also, suppresion of immune system posttranplant really increases risk of malignancy. Greatest risk being viral-associated neoplasms like HPV, EBV (Kaposi's), Hep B and Hep C. Chronic allograft nephropathy: refers to chronic fibrotic changes and accelerated loss of renal functions that occurs in tranplated kidneys. Generally eyars after tranplant and you see progressive increase in serum creatinine, proteinuria, and microscopic hematuria. Need to do biopsy to confirm this diangoiss. *Tissue typing is important in kidney matching. BUt with CARDIAC transplants you don't worry about typing cuz cardiac allografts have a cold iscehmia preservation time of 4 to 5 hours, and therefore tissue typing is not practical. Cardiac donrs are matched to recipients only by size and ABO blood type.

Hypomagnesemia

<1.8 -Drugs (loop diuretics, amphotericin) -Low magnesium causes hypokalemia and hypocalcemia that WILL NOT RESPOND to replacement if magnesium not fixed first -ECG changes similar to hypokalemia

Hypophosphatemia Serum phosphate

<2.5 Usually alchoholism or malnutrion the cause. *in chronic alchoholics that show up to hospital....despite total body phosphorus depletion, these patients may have normal serum phosphorus levels on admission to hospital. Severe hypophosphatemia often develops over the first 12 to 24 hours after admission, usually because of IV glucose administration, which stimulates insulin release and causes phosophate to shift into cells. The suddes development of hypophosphatemia may cause confusion, rhabdo, hemolytic anemia, and severe muscle wakness that can lead to respiratory failrue. Neuro symptoms and muscle weakness. Treat with phsophate supplementation

Hypermagnesemia

>2.3 Iatrogenic, magnesium contatinin druges (some laxative, antacids) if given to a patient with renal failure *earliest clinical sign is loss of DTRs. Tx: IV fluids, calcium if there are ECG changes, dialysis if refractory

Hyperphosphatemia Serum

>5 Renal failure , tumor lysis syndrome, iatrogenic (too much given) Tx: Calcium carbonate (binds phosphate in gut and decreases absorption),

What should be ordered every time you put a central venous catheter in? What are some potential complications?

A chest x-ray should be obtained in all patients who get a central venous catheter to confirm proper placement and and absence of complications before administering drugs via the catheter. Central venous catheter measures pressure in superior vena cava when placed via internal jugular vein or subclavian vein. POTENTIAL COMPLICATIONS: 1. Tension pneumothorax -tx: decopmression with needel or placement of chest tube 2. Hemothorax -Tx: chest tube 3. air embolism. Tx: place patient in left lateral decubits Trendelenburg (head down) position and aspirate the central venous cathether. Thoracotomy with aortic cross clamping if everything else fails i think.

ABCD criteria of suspicious skin lesions

A: Asymmetry B: Borders-bad if rugged or blurred C: Color- bad if 2 or more colors D: Diameter-bad if >6 mm E: Elevation (raised surface) and Evolving

Esophaeal motor disorders

ACHALASIA -weight loss, cough, diffuse chest pain -LES CAN'T RELAX on swalloing -looks like birds beak on barium swallow. Esophageal motility study usually required to confrim diagnoses because of how other disorders can mimic achalasia. You find SMALL-AMPLITUDE contractions in esophagus, incomplete relaxation of LES, and higher-than-normal pressure in body of esophagus. -relieved by nitroglycerin -Tx: Safest and most effective treatemnt for this condition is surgical treatment with an esophagomyotomy. Also botulinum toxin injection. DES (diffuse esophageal spasm) -dysphagia, diffuse chest pain, often precipiated by ingestion of hot or cold liquids -swallowing induced large wave. will have high-amplitude contractions in body of esophagus. This is diagnsotic. -looks like corkscrew on barium swallow -releived by nitroglycerin Tx: long myotomy guided by manometric evidence. Really effective if performed correctly. -also known as nutracker esophagus *Dysphagia to solids AND liquids often indicated a motility problem (like achalasia, DES). Dysphagia to ONLY solids indicated mechanical obstruction ZENKER's DIVERTICULUM -elderly -dyshpagia and regrurgitation -halitosis -at risk for aspiration pneumonia -occurs due to posterior herniation between fibers of the cricopharyngeal muascle Dx: A contrast (barium) esophgram Tx: surgical: most common procedures is a cervical pharyngocricoesophageal myotomy (incising the cricopharyngeus). *When Zenker's causes Zymptoms it requires Zurgery. Asymptomatic Zenker's is treated only if it >2cm in size.

Admission Orders stuff

ADA-Vitamin L A: Admit to: (name of admitting doctor and unit) D: Diagnosis A: Allergies : Allergies V: Vital Signs I : Ins (diet (clears, general, npo, etc) , ivf) T: Telemetry or other monitoring A: Activity M: Meds *Meds For everyp g g atient consider the following categories and if they are indicated for your patient (not all are neededin all patients): Pain meds Antibiotics DVT prophylaxis (may not be a med) Ulcer prophylaxis Previoous meds I: Is and Os N: Nursing : Nursing L: Labs *Vital Signs- Vital signs q one hour (in ICU) "VS" typically includes temperature, blood pressure, heart rate and respiratory rate. If you want any additional vital signs they should be want any additional vital signs, they should be ordered specifically Example: Monitor CVP q one hour Monitor pulse ox q 4 hours All IVF orders need the type of fluid, any additives, p and the rate. Examples: D5.45NS with 20 KCl at 100cc per hour LR at 85 ccper hour .9NS at TKO (to keep open rate) Heplock IV (this means the IV is capped off and no fluids) Maintenance IVF: 4-to-1 thing A: Activity Examples: As tolerated bed rest no weight bearing to right foot ambulate with assist only Consider how frequently you want UO checked: • UO q one hour (typical for ICU pt with foley) • UO q shift NURSING Can include many things. Examples: Call HO for BP>170/90 <90/60 HR>120<60 RR>30<12 T>38 Call HO for BP>170/90, <90/60, HR>120<60, RR>30<12, T>38, UO<30cc/hour or 200/shift, Glucose >200<70 Dressing changes: wet to dry to right leg wound tid Keep leg elevated Call HO for any change in mental status Check and record pedal dopplers q 4 hours JP to bulb suction Foley to gravity Incentive spirometer q one hour while awake Teds and boots at all times unless ambulating

Indications for emergent hemodialysis

AEIOU Acidodi Elctrolyte disturbances Ingestions Overload (volume) Uremia

Anal problems

ANAL FISSURE -Severe anal pain with defecation, bleeding, itching, and minimal drainage -if you suspect it, examination under anesthesia, anoscopy, and proctoscopy are approprirate in evaluating a patient suspected of having it. Tx: Sitz baths, stool softeners, suppositorties, nitoglycerin ontiment. Chronic fissures can be treated with botulinum toxin injection or internal sphincterotomy. -when you see anal fissure. Rmember Crohn's, ulcerated hemorrhod and low anorectal malignancy. HEMORRHOIDS hemorrhoids tx: 1. General measures to ease symptoms -sitz bath -applicatino of ice packs to anal area and bed rest -stool softeners to reduce strain -high-fiber, high-fluid diet -topical steroids 2. rubber gand ligation for internal hemorrhoids: rubber bands applied to hemorrhoidal bundles leads to necrosis and sloughing of lesion 3. surgical (hemorrhoidectomy) PERIANAL ABSCESS Painful, fluctuant perianal mass or ulcer associated with fever and/or purulent drainage. Tx: Incision and drainage under local anesthetic. FISTULA-IN-ANO Drainage of pus or mucus or minimal stool soilage on undergarments Tx: Fistulotomomy w/drainage.

Fat embolism syndrome

ARDS caused by release of fat droplets from the marrow as may occur following a long bone fracture. Occludes pulmonary circulation. Classic triad for fat embolism: confustion, dypsnea, petechiae on chest symptoms may occur immeiately or 2-3 days after trauma HALLMARK finding: arterial hypoxemia. Arterial P02 <60 is suggestinve. -X-ray: snowstorm like infiltration Tx: oxygen. -pevent fat embolism syndrome by careful stabilization of fractures and effective treatment of shock.

AAA Dx and Tx

Abdominal or pack pain, pulsatile mass in abdomen, hypotension Gold standard for Dx: Angiogram. US, MRi, CT w/ contrast are really good as well. Study of choice for diagnoses of AAA? Abdominal ultrasound. Strognest predictor of AAA expanision and rupture?large diameter, rapid rate of expanision, and cigarrette smoking Tx: usual criterion for recommending repiar is when AAA reaches 5.5 cm in men and 5.0 cm in women. -The actual recomendation for EVAR (endovascular aneurysm repair) versus open AAA repiar should be made by a trained vascular specialist. -EVAR requires more intestive postprocedural follow-up than open repairs. AAA screening? clinical presentation? Male active or former smokers aged 65-75 should get a one-time abdominal ultrasound to evaluate for abdominal aortic aneurysm. -yearly ultrasound is part of conservstive follow up for patient with AAA

Perianal abscess vs perianal fistula vs pilonidal cyst

Abscess: anal pain and and very tender, erthematous bulge at the anal verge. Can be perianal and perirectal. Close to the anus. Perianal fistula: due to chronic anal crypt infection or Crohn's disease. External opening draining purulent material. usually within 3 cm of anal margin. My become plugged and dvelop a perianal or perirectal abscess. Pilonidal disease: acute pain and swelling of the midline located between gluteal clefts. The acute presentation involves infection of a dermal sinus tract orginiating over the coccyx. *Most prevalent in young males, particularly those with larger amounts of body hear. INfection of hair follicles in this region may spread subcutaneously forming an abscess that then ruptures forming a pilonidal sinus tract.

Paralytic ileus?

Absent bowel sounds with gaseous distention of both small and large bowels. This classically follows abdominal surgery but can also occur in cases of retroperitoneal hemorrhage associated with vertebral fracture.

Acute mesenteric ischemia vs bowel infarction Acute mesenteric ischemia vs chronic mesentirc ischemia

Acute mesenteric ischemia classically presents with severe acute-onset midabdominal pain out of proportion to physical exam findings. THe most common cause is an embolus from the heart. Progression to bowel infarction causes peritoneal signs and the passage of bloody stool. Acute mesenteric ischemia vs Chronic mesenteric ischemia ACUTE: Sudden onset of abdominal pain due to actue occlusion of mesenteric vessel by emboli or thrombosis; surgical emergency -SMA embolus most common. SMA provides blod to small bowel, ascending and transverse colon. -Arteriography can aid in Dx but may lead to treatment delays and should not be applied toward individuals who are suspected of having intestinal ischemic necrosis. Clinical judgment should be exercised in decidiing whether imaging should be performed prior to emergent laparotomy. CHRONIC: Episodic postprandial pain, due to slow narrowing of vessels, due to atherosclerosis; WEIGHT LOSS associated, not emergency -Best treatment for chronic is operative revascularization Gold standard Dx: angiogram *Nonocclusive mesenteric ischemia due to low-flow states can develop ina setting of critical illness or as aresult of vasoconstrictive medications administration.

Algorithm for evaluation of a thyroid nodule

Algorithm for evaluation of a thyroid nodule First: clinical evaluation, TSH level, and US ---if cancer risk factors or suspicious US findings---> then do Fine Needle Aspiration ---if no cancer risk facotrs or suspicious findings--> then look at the TSH you measured..then... ..if TSH is normal or high do Fine Needle Aspiration ..if TSH is low then do radionuclide scan (iodine 123 scintigraphy). -----if "hot" nodule that meas its hyperfunctioning (increased isotope uptake in the nodule with decreased surrounding uptake)--->low cancer risk, treat hyperthyroidism -----if "cold" nodule means its hypofunctioning (decreased isotope uptake compared to surrounding tissue )--->higer cancer risk so you should do a Fine Needle Aspiration

Platelet disorders

All of the increse bleeding time (how long you bleed) and decrease plaetelet count (excepto glanxman thromboasthenia and Uremia). Bernard soulier syndrome: defect in platelt plug fomration Glanzmann thrombasthenia: defect in platelt plug fromation by another mnechanism. Periph smear shows no plaetles clumpgin. ITP (immune thrombocytopenia): make anti Gp2b antibodies so spleen macrophages start to eat the platelt/antibody complexes. Trigggers by viral illness commonly. Lab: increased megakaryocytes on bone marrow biopsy. TTP: FAT RN Fever Anemia Thrombocytopenia Renal dysfunction Neurologic abnormality you don't have the vWF metalloprotease ADAMTS 13 so you can't degrade the vWF multimers--> platelets aggregation and thrombosis. Labs: schicstocytes and increased LDH TX: exhange transfusion (cuz you wanna get rid of the antibodes against ADAMT13 in the plasma and steroid) ALSO.. Uremia: qualitative plaelet disorder can be result of uremia. Metabolic by products inhibit plaetletes.

Clavicle fractures

All patients with a clavicular fracture should have a careful neruovascular examination to rule out injury ot the underlying brachail plexus and subclavian artery. If a bruit is hear u need to do an ANGIOGRAM to r/o injury to underlying vessel. the majority of clavicual fractures occur in the middle third of the bone. Injury to this bone classically occurs during athletic events and follows a fall on an outstreteched arm or a direct blow to the shoulder. Patients have pain and immiblity of affected arm and classicaly use contralateral hand to support the weith of affected arm. Most clavicular fractres can be treated nonoperatively with brace, rest and ice.

Leriche syndrome?

Aortoiliac occlusion which is characterized by triad of bilateral hip, thigh and buttock claudiction, impotence and symmetric atrohphy of bilateral lower extremities due to chronic ischemia.

Explain frostbite and treatment for it

As the body reduces cutaneous blood flow in freezing temperatures to maintain the core body temperature, capillary blood becomes more vicous, and ice crystal sform in the extracellular space. This causes direct tissue injury and osmotic pressure that cause intracellular dehydration. Tx: Immersion of the affected part in water at 104-111 degrees. Just watch out for excessive heat or trauma during this cuz the frobitten region is numb and especialy vulnerable.

Medial collateral ligmanent injry

Associated with abduction injury to the knee. Valgus stress test will help in Dx. You get excessive outward deviation.

Gallstone disease

BILIARY COLIC: waxing and waning, poorly localized postprandial upper abdominal pain radiating to the back. Caused by CCK stimulated gallbladder contractions following food ingestion. Condition is generally produced by gallstone obstruction at the gallbladder neck or gall bladder dysfunction (if its gall bladder dysfunction causing the trouble, then its called BILIARY DYSKINESIA). -no elevated leukocyte count, normal LFTs -afebrile -no wall thickening or CBD dilation on ultrasound -RUA pain <6 hrs in duration Tx: elective cholecystectomy *Biliary dyskinesia diagnosis: confrmied by CCK-HIDA scan. HIDA injected IV, after it fills the gallbladder, CCK is injected IV to stimulate gallbladder contraction. A gallbladder ejection fraction of less than 35% at 20 minutes is dagnostic of biliary dykinesia. ACUTE CHOLECYSTITIS Cholecystitis. Usually form obstructing stone. -May be febrile or not febrile, tachycardia, RUQ pain, gaurding, Murphy's sign -elevated leukocyte count -RUQ pain lastin >8 hrs Dx: Abdominal US first. IF unclear after that, seems like you do a HIDA. HIDA-study of choice. Technetium 99m labeled is injected IV and teaken up by hepatocytes. Normally, the gallbladder is outlined within 1 hour, absence of visible gallbladder on HIDA=cholecystitis. -you also see eleveated PMNs, alk phosph, total bili Tx: Cholecystectomy, Antibiotics *ERCP uses a fiberoptic camera and fluoroscopy to visualize the biliary and pancreatic ducts for diagnostic and theraputic purposes. It would be reasonable to perform ERCP if the patient had choledocholithiasis or a gallstone in the COMMON BILIARY DUCT (CBD) causing CBD dilatation. In such cases, sphincterotomy can help facilitate passage of stones. Also it is useful for CHOLANGITIS. *The HIDA scan is a nuclar medicine study used to diagnose gallbladder obstruciton Whit this technique, a nuclear tracer is injected into the blood and is excreted from the liver into the bile. Typically, the radiotracer can be seen pooling in a normal gallbladder. Failure to visualize tracer in the gallbladder is suggestive of obstruction. Ultrasonography demonstrates cholecystitis in this patient, so the scan would not add to the diagnosis. A HIDA scan can be used in patients WHERE ULTRASOUND CANNOT make a clear diagnosis of acute cholecystitis. ACALCULOUS CHOLECYSTITIS -What is it? Acute inflammation of the gallbladder in the absence of gallstones,caused by biliary stasis, most commonly seen in hospitalized and severely ill patients in the following conditions....extensive burns, sever trauma, prolonged TPN, prolonged fasting, mechanical ventilation. Imagin studies show gallbladder wall thickening and distention and pericholecystic fluid. The emergemcy treatment of choice is antibiotics and percutaneous cholecystoostomy, followed by cholecystectomy when the medical condition stabilitizes. CHRONIC CHOECYSTITIS -repeated bouts of biliary colic and/or acute cholecystitis leading to gallbladder wall inflammation and fibrosis CHOLEDOCHOLITHIASIS -may or may not be jaudiced, RUQ abdominal tenderness -ultrasound shows gallstones in gallbladder, CBD usually dilated -elevated LFT VALUES -CDB diameter >5mm *Gallstone pancreatitis should be considered if amylase and lipase are high -Tx: hospital observation for development of cholangitis, ERCP clearance of CBD stones, in addition to cholecystectomy. CHOLANGITIS-infection of biliary tract -Charcot's triad: Jaundice (so you see elevaetd LFTs in cholangitis or choledokolithiasis), Fever, RUQ pain. most commnly caused by obstruction of bile ducts by gallstones or strictures. May lead to sepsis and ogan failure. -air in the biliary tree is consistent -Tx: antibiotics and supportive care. If really severe , do ERCP or surgery to decompress bile duct SEPTIC CHOLANGITIS -Reynold's pentad: Charcot's triad + Hypotension + Altered mental status. A worse cholangitis. I think you do ERCP or surgery here.

Leading complication of total surface body burns in the setting of adequate fluid resuscitation

Bacterial infection (usually bronchopneumonia or burn wound infection) leading to sepsis and septic shock is the leading complication.

Tools for assessing esophageal functions

Barium Swallow Flexible endoscopy Manometryand ph monitering: combined procudre in which small eletctronic pressure transducer is swallowed by patient to be positioned in vicinity of LES>

Resuscitation in trauma

Begins during primary suvery IV FLUID -isotonic crystalloid :either lactated ringers or normal saline -"3-to-1" rule as rough estimate for total amount of cystalloid volume needed to replace blood loss IMAGING -"trauma series": x-rays of C-spine , chest, and pelvis usually occur early in resuscitative efforts but should never interrupt the process -DPL (diagnostic peritoneal lavage) and FAST (focused abdominal sonogram for trauma) are tools used for rapid detection of intrabdominal bleeding

Secondary survery in trauma

Begins once primary survery is complete and resuscitative efforts are well under way AMPLE history (allergies, medications/MOA of injury, pmh/pregnant?, last meal, events surrounding mechnism of injury Tetanus prophylaxis as needed

Femoral hernias where?

Below the inguinal ligament

Esophageal varices

Beta blockers are used as prophylaxis against future bleeding Managemtn of ruptured varices causing acute bleed 1. stabilize hemodynamics: use NS or LR and packed RBCs, NG tube 2. Use octreotide/vasopressin to reduce splanchic blood flow and portal pressure 3. Urgent endoscopic sclerotherapy: injection of bleeding vessels with sclerosing agent via catheter, also endoscopic band liation is equivalent to sclerotherapy initially with fever complications 4. Balloon tamponade to apply direct pressure 5. For refractory acute bleeding, TIPSS procedure 6. Intraoperatie placement of a portocaval shunt

Describe the types of hematuria and what they mean?

Blood at beginning of urination (inital hematuria) typically indicates a lesion in the urethra such as urethritis. Hematuria t the end of voiding, called terminal hematuria, often suggests a prostatic or bladder cause. Hematuria during the entire urinary cycle (total hematuria) may indicate disease within the ureters or kidneys. A renal cause of hematuria will not show clots.

BRCA1 and BRCA 2 info. how to manage?

Both are associated with increased risk of both breast and ovarian cancer. Prophylactic bilateral mastectomy and reconstruction is reommended foR BRCA muation carriers. If that is not performed, intensive surveillance should be done w/ biannual clinical examinations and annual mammograms starting at age 25. *BRCA1 (usually receptor negative) is the one that is associating with colon cancer * BRCA2 (usually hormone receptor positive)

How to not get tricked by hemithorax vs tension pneumothorax after bchest trauma

Both can cuse tracheal deviating. TP causes deviation and decreased breath wsounds with hyperresonance. Hemithorax causes trachea deviation, decreased breath sounds, and dullness to percussion.

Prostatitis vs UTI clinical presentation?

Both can have frequency, urgency, dysuria, and decreased urinary steam, and fever. But UTI is NOT associated with perineal or pain on rectal examination, prostatitis is. Tx of prostatitis is 3 weeks of oral antibiotics.

Possible complication for AAA repair?

Bowel ischemia and infarction. Results from indeuqate colonic collateral arterial perfusion to the left and sigmoid colon after loss of the inferior mesenteric artery during aortic graft placement. Patients presents with abdominal pain and bloody diarrhea. Fever and leukocytosis may be present also. This adverse effect can be minimzed by checking sigmoid colon perfusion folowing placement of the arotic graft..

NG tube indicated for what?

Bowel obsruction, enteral nutrition and gastric lavage, i think GI bleed too?

Shifts oxygen-hemoglobin disassociation curve to the right?

C-BEAT C02 BPG Exercise Acid/Altitude Temperature

Pseudomembranous colitis

C. difficle colitis is another name for it. Occurs following antibiotic treatment and causes a voluminous watery diarrhea with pain and fever. bloody diarrhea is rare. Dx confirmed with colonoscopy or mor commonly by detecting toxin in the stool with ELISA.

Indications of CABG vs PCI (percutanous coronary intervention)

CABG: indicated in patients with angina (chronic, unstable, or postinfarction) and in asymptomatic patients with ischemia on cardiac stress test. -Good for multivessel disease, left main coronary artery disease and 1-vessel and 2 vessel disease with LAD obstruction. Also treatment of choice in diabetics. -PCI is also appropriate for 2 vessel disease w/ LAD obtruction.

Cardiac index?

CO (SV x HR) / total body surface area

Screenings studies and confirmatory study of functional adrenal lesions; Cushings syndrome, pheo, primary aldosteronism

CUSHINGS SYNDROME Screening: Dexamethasone test Confirmatory study: Low ACTH and DHEAS PHEOCHROMOCYTOMA Screening: Plasma metanephrines Confirmatory study: 24 hour urine metanephrine (VMA, metanephrine, and normetanehprines) PRIMARY ALDOSTERONISM Screening: Plasma aldosterone-renin ration >30 Confirmatory: Saline salt loading test or captopril challenge Surgery is recommeneded for.. all functioning tumors nonfunctionaing tumors 4cm or greater tumors less than 4cm that are enlarging

Types of stones and treatment

Calcium oxolate: strongly radiopaque (visible on x-ray), treat with thiazides to decrease calciuria. 75% Struvite 15% -moderately radiopaque. Common in Proteus UTIS due to high urinary ph. Proeus makes urease which cleaves urianary urea giving two molecules of ammonia, the conjugate base of ammonium ion. Tx: by lowering urine Ph. uric acid -radiolucent, increased in myeloproliferative disorder and gout due to increase purine turnover -treat by raising urine ph Cystine -hexagonal crystals, moderatley radiopaque. Seen in congenital cystiuria, NOT homocystinuria. -Treat by raising urine Ph.

Ankle-brachial index

Calculated by dividing the higher ankle (dorsalis pedis or posterior tibial) systolic pressure in each lower extremity by the higher brachial artery (left or right) systolic pressure ABI <.90= abnormal (diagnositc of ABI) 0.91-1.30= normal >1.30=suggestive of calcified and uncompressibnle vessels; additional vascular studies should be considered. *Arterial ultrasound is less sensitive and less specific than ABI for initial diagnoses of PAD. Generally done in symptomait patients with abnoraml ABI who are being considered for interventialn procdues.

Lower GI bleeds causes

Can U Cure Aunt Di's Hemorrhoids? Cancer or polyps UGI bleed Colitis (infections, IBD, ischemic, etc) Angiodysplasia Diverticulosis Hemorrhoids Signs: hematochezia, melena (tarry stool, usually associated with Upper GI bleed but can occur w/ascending colon bleed)), tachy, hypotension Dx:NG tube aspiration. If no blood, positive bile on NG tube aspirae... -then do colonoscopy to find source. If you can't find the source and the patient stops bleeding then do small bowel studies (capsule endoscopy). But if you can't' find source and th patient does not stop bleeding do a Technetium-99 labeled ertyrhoctye scintigraphy. It localized source of blood loss so that region can be further evaulated by coloscopy or angiography. If blood found, and plus/minus bile on NG tube aspirate.. -then do upper endoscopy. Tx: IV fluids, blodo, embolization or surgery

Gastic outlet obstruction

Can be caused by many disease and is characterized by early satiety, nausea, nonbilious vomiting and weight loss. Common causes are malignancy, PUD, Crohns, strictures secondary to ingestion of acid.

Gallbladder polyps management?

Can be observed with serial ultrasounds if they are less than 1 cm in size. If they are symptomatic they should be removed regardless of size. 90% are benign lesion. Indications for operative managment remain controversial. Patients with suspected gallbladder carcinoma should undergo cholecystectomy with intraoperative frozen section, and if there is invastion of the serosa and no evidence of metastatic or extensive local disease, they should undero radical cholecystectomy (portal lymphadenectomy and either wedge or formal resection of the liver surrounding the gallbladder fossa in addition tothe cholecystectomy).

Ligation of the IMA too far from the aorta can cause problems in blood supply where?

Can interfere with the collateral blood supply to the rectosigmoid colon and lead to ischemia. Abdominal distention, fever, elevation of WBC, and/or bloody diarrhea should raise suspicion for colon ischemia. IMA usually ligated in AAA repair so watch out for this as side effect after.

Acute mediastinitis?

Can occur following cardiac surgery and present with fever, chest pain , leukocytosis, and mediastinal widening on chest x-ray. It is a serious condition that requires drainage, surgical debridement, and prolonged antibiotic therapy.

Acute colonic pseudoobstruction

Can result from tauma and presents with nausea, abdominal pain, abdominal distention, tympanitic bowel sounds, and hyperactive bowel sounds. Abdominal film tends to show a massively dilated colon without significant small bowel dilation.

Periampullary cancer

Cancers of pancreas, distal bile duct, duodenum, ampulla of Vater *Pancreatic cancer is most common of periampullary cancers Painless jaundice, palpable, nontender gallbladder (Courvosier sign). Also weight loss, diabetes, abdominal pain and gastric outle obstruction can be common signs of pancreatic cancer. Dx: Being w/ US to r/o stones as source ob obstruction. THen do CT scan. Tx: The majorit of patients with pancreatic carinoma have UNRESECTABLE disease at time of diagnosis. For relief of biliary obstruction, you can do an endoscopic placemnt of a biliary stent. -for unresectable do chemo and radiation Pancreaticoduodenectomy: An operation involving resection of the duodenum, the head of the pancres, the CBD, and sometimes the distal stomach. PD indicated for Tx of patients with tumors and benign disease localized in area surrounding ampulla of Vater. Classic form of the operation is called a Whipple resection. This is for cases where biopsy has not shown cancer and clinical suspicion of cancer is high.

Caustic injury to esophagus

Caused by acid (househould cleaning agents) or alkali (lye, sodium, hydroxide tablets). -Alkali burns are worse than acidic, as acid substances usually burn the mouth immediately and are less frequenlty swallowed, alkakine substances are more frequently ingested. - Acidic substances also cause coagulative tissue necrosis, which limits their penetration, whereas alkaline substances cause injury deep into the tissue as they dissolve the tissue. There is an acute phase and a chronic phase. Acute: control immediate tissue injury and perforation potential Chronic: managing strictures and swallowing disturbances that have developed *fibrosis, retraction and narrowing os esophagus can occur. Stricture develop in 80% of patients within 2 months. Signs: oral and substernal pain, hypersalivation, fever (strongly associated with esophageal lesion), vomiting, acidosis, laryngospasm, dysphagia (occurs acutely. then reappears in chronic phase) Dx: if perforation suspected do esophagram with water-soluble contrast. Tx: early esophagoscopy (<24 hr) limit the burn by giving neutralzing agents: -lye/alkali can be neutralized with half strenth vinegar, lemon juice, or orange juice -acid can be neutralized with mildk, egg white, or anti-acid -dilatations are controversial cuz of perforation risk -extensive necrosis leading to perforation is best managed by resection. is esophagus is viable use intraluminal stent. -is really bad stenosis/stricture, i think you use stomach, jejunum, or colon to replace the esophagus. *Pre test says neutralizing agents are often unwise to use becuase of the potential damage caused by the reaction. Also dilution with water is not recommended given that most damage has already occurred and increaseing gastric volume may induce nausea and vomiting.

Plantar fasciitis

Causes burning pain in the plantar (not dorsal) are of the foot that worsens with the first steps in th emorning. The paain decresases as activity increases during the day but usually worsens at the end of the day with prolonged weight bearing. Common in runners with repeated microtrauam who develop local point tenderness on the plantar aspect of the foot.

Causes of hypernatremia

Causes of hypernatremia Hypovolemic (most common). Renal loss, diarreha, sweating, burns, iatrogenic. -Tx: Normal saline Isovolemic: Diabeteis insipidus -Tx: Hypotonic fluid (5% dextrose preferred over .45% saline). Vasopressin for DI. Hypervolemic: Mineralcorticoid excess ( Conn's tumor, Cushings) -Tx: half normal saline to correct hypertonicity. Diuretics to increase Na excretion.

Central vs anterior vs posterior cord syndrome

Central cord syndrome: typically results with HYPEREXTENSION injuried in elderly patients with pre-existing DEGENERATIVE CHANGES in the cervical spine. It is characterized by weakness that is more pronouned in the upper extremities than the lower and can be accompanied by localized "cape" distrubtion of pain and temperature loss. Anterior cord syndrome: bilateral spastic motor paresis distal to the lesion. Usually due to occlusion of anterior spinal artery. *anterior cord syndrome is commonly associateed with burst fracture of the vertebra and is characterized by total loss of motor function below the level of lesion with loss of pain and temperature on both sides below the lesion and with intact proprioception. Posterior cord sndrome: bilateral loss of spinothalamic, dorsal colum. Often with weakness, paresthesias, and urinary incontinence or retention. May be due to a varitey of causes, but MS and vascular disruption are most common

Patient has a Dieulafoy lesion?

Characteristially located within 6 cm distal to the gastroesophagleal junction. Dieulafoy lesion typically consists of an abnormally large submucosal artery that protrudes through a small, solitary mucosal defect. For unclear reasons these lesions may bleed pretty massively and in that case would require emergency surgical intervenion. Uper endoscopy is usually successful in localizing lesion and permanent hemostasis can be obtained endoscopically in most cases with injection sclerotherapy, electrocoagultion or heater probe. If surgery is required, a gastrotomy and simple ligation or wedge resection may be adequate.

How does left sided portal hypertension present? What if there is acute hemorrhage?

Characterized by gastric varices in the setting of splenic or portal vein thrombosis in the abscence of cirrhosis. If there is acute hemorrhage the treatment is splenectomy.

Metabolic alkalosis divided how?

Chloride responsive: urine chloride <20 due to hypovolemia and hypochloremia. Saline responsive because normal saline administration restores arterial volume, vorrects hypochloremia, and increases urinary bicarb excretion. -vomiting or prolonged NG tube drainage -diuretics -laxative abuse -diuretic discontinuation -pyloric stenosis Chloride unresponsive: urine chloride >20. Saline resistant. Excess mineralcorticoid causing hydrogen an dpotassium loss and increased sodium retention leading to increased EC volume. Kidneys respond by excreting both sodium and chloride to result in high urine chloride. -Mg or K deficiency seevere -increased mineralcorticoids -licorice -inherited disorders

What is atheroembolism?

Cholesterol embolism. A common complication of cardiac catheterization and other vacular procedures. -can result in lower extremity ischemia, MI, ischemic bowel or renal failure. Labs: eosinophilia is strongly suggestive of cholesterol atherombolization, and other lab findgins includes microscopic hematuria or proteinuria and elevated ESR. You see cutanoues findings ("blue toe syndrome", livedo reticularis (reticular, lacy skin discoloration/erythema that blanches on pressure), cerebral or itestinal ischemia, acute kidney injury, and Hollenhorst plaques (bright, yellow, refractile plaques in retinal artery). Tx: is supportvie and includes statin thearpy for risk factor reduction and prevention of cholesterol embolism.

Addison disease

Chronic primary adrenal insufficiency (addisons) -due to disease, automimmune, TB, metastasis. Defieicny of aldosterone and cortisosl, causing hypotension, hyperkalemia, acidosis, and skin and mucosal hyperpigmentation. Involves all 3 cortical divisions. Secondary insufficiency is different (commonly get from abruptly stopping steroids after long use or not tapering well enough) -decreased pituiatry ACTH so you get no skin hyperpigmentation. Diefiency of mineralcorticoid usually does not occur so you do not develop hyperkalmiea *Also remember about Acute adrenal insufficiency (waterhouse frideriechens syndrome) that can be due to Neisseria meningitis, septicemia, DIC and endotoxic shock * Also the best screening tests for patients with suspected adrenal insufficiency are early-morning cortisol, ACTH, and cosyntropin (ACTH analogue) stimulation. An increase in serum cortisol >20 30-60 minutes after giving 250 cosyntropin virtually RULES OUT primary adrenal insuffinecy (Addisons)

How does primary sclerosing cholangitis present?

Chronic progressive disorder of unkown etiology with inflammation, fibrosis and stricutring of medium and large sized intrahepatic and extrahepatic bile ducts. Continued bile duct destruction leads to end-stage liver disease and portal hypertension. Frequently associated with UC. Labs usually show unexplained elevated LFTs in a cholestatic pattern. (elevated ALT and AST, severely elevated Alk phosph and bilirubin. Dx: Not US. Cholangiogram shows multifocal narrowing and duct dilataion "beading". Also "onion" skin fibrosis. Tx: Only cure is liver tranplant

PCL injury

Classically seen in the "dashboard injury" which refers to forceful posterior directed force on the tibia with the knee flexed at 90 degree. The posterior draer, reverse pivot shift and posteiror sag test will help in clinical diagnosis.

Wound classes

Clean wounds: no part of respiratory, GI or GU tract is entered. Clean/contaminted wound: Those tracts are entered but without evidence of active infectio or gross spillage. Ex: elective cholecystecomty or elective colon resection. Contaminated wound: open accidental wounds. Dirty wounds: Traumatic wounds in which a sigfnicicant delay in treatment has occured and in which necrotic tissue is present.

Epidermoid cancer of anus treatment?

Combined radiation with chemo, also known as the Nigro protocol, is standard treatment. Local radical surgery results have been disappointeing, surgery now reserved for treatment failure cases. THe surgery that would be done is abdominal-perineal resection that involves removing the rectum and anus with formation of permanenct end colostomy. *APR also procedure of choice for distal rectal cancers that involve sphinceters OR are too close to obtain adeuqate margins (2cm) OR for patients for whome sphincter-sparing surgery is contraindicated because of fecal incontinence. ---Preoperative or neoadjuvant chemoradiaion can sometimes cause distal rectal tumors to shrink in size so that sphincter-sparing operation can be performed. *LAR (low anterior resection:removeal of rectum to below the peritoneal reflection through an abdominal approach) is for proximal and midrectal cancers.

Iatrogenic injury to the common bile duct

Commonly occur in the proximal portion of the extrahepatic biliary system. If there is a stricture then bets way to deal with it is end-to-side choledochojejunostomy (Roue-en-Y) performed over a stent.

Shoulder dislocation

Complications common to all dislocations: -axillary artery injury. palpate radial pulse to check axillary artery. -venous injury -injury to brachial plexus nerves (most common being axillary nerve). check motor component of axillary nerve by assessing strength of deltoid muscle. check sensory component of axillary nerve by assessing sensation over lateral part of upper arm. do a neuro exam to evaluate all brachial nerves. ANTERIOR DISLOCATION -high risk of recurrence. -types: subcoracoid (most common), subclavicular, subglenoid -what happens: abductino and external rotation of arm causes strain on anterior capsule and glenohuemral ligaments -signs:shoulder is internally rotated. X-ray: look for Hill-Sachs deformity (compression fracture of posterolateral humeral head) in the posterolateral portion of humeral head ( occurs in 50%) Tx: reduction, immobilize shoulder, surgery if needed POSTERIOR DISLOCATION -diagnosses missed in 60% of cases. often preciipated by convulstion, seizures, falls -subacromial (most common) -what happens: internal rotaion and adduction causes it. (when one falls on an arm that is forwardly flexed and internaly rotated) -signs: patients holds arm adducted and inernally rotated, prominent coracoid process X-ray: look for loss of normal elliptical pattern produced by overlap of humeral head and posterior glenoid rim Tx: apply longitudinal traction (Stimson techniqe). Surgery if needed. INFERIOR DISLOCATION (luxatio erecta) -rare -hyperabduction always results in detachment of rotator cuff -patient in sever pain, arms held in 180 degree elevation, arm appears shorter compared to oopposite side

Insertion of foley review sheet

Concepts to Remember: 1. Urinary catheterization is a sterile procedure. 2. Urinary tract infection can occur following catheterization. 3. Place the drainage system below the level of the bladder or clamp tubing to prevent retrograde flow of urine. Common Types of Catheters: 1. Straight- for intermittent and/ or self-catheterization 2. Indwelling- foley catheter for monitoring and collection 3. Coude'- (male patients) to facilitate catheter placement due to history of BPH and/or bladder neck obstruction Procedure: 1. Identify patient, ascertain latex allergy 2. Explain procedure, provide privacy, position patient, provide for patient safety 3. Stand on left side of bed if right handed and vice-versa. 4. Wash hands; if necessary, don exam gloves & cleanse perineal area if grossly contaminated; discard appropriately 5. Open sterile catheterization kit on bedside table 6. Don sterile gloves 7. Organize supplies on sterile field- a) pour sterile antiseptic solution onto cotton balls b) test catheter baloon by inflating with fluid from prefilled 2 of 2 syringe; deflate balloon and leave syringe attached to port c) lubricate 2-3 inches of catheter tip d) attach end of drainage bag to catheter 8. Apply sterile drape under buttocks (cuff drape over hands) 9. Apply fenestrated drape over perineum 10. Work from bed-side table ( dictated by patient positioning) 11. Cleansing urethral meatus- a) with non-dominant hand expose urethral meatus ( considered contaminated); DO NOT TOUCH STERILE SUPPLIES WITH NON-DOMINANT HAND AFTER MEATUS IS EXPOSED b) with dominant hand cleanse with forceps & antiseptic cotton balls 12. Pick up catheter & drainage bag as one unit with dominant hand and place drainage bag on sterile drape between patient's thighs; retain control of tip of catheter 13. Instruct patient to take a deep breath; slowly insert catheter through meatus 14. Advance catheter 3-5 inches until urinary flow is established; advance another 2-3 inches but do not force cather when resistance is encountered 15. Release labia or foreskin/penis and hold catheter securely with non-dominant hand 16. Inflate balloon of indwelling catheter per directions; pull gently to feel resistance; disconnect syringe 17. Place drainage bag in dependent position but NOT ON SIDE RAIL 18. Tape catheter: a) for female- tape to inside of thigh; allow for slack to eliminate tension on catheter b) for male: tape to top of thigh or lower abdomen (penis directed toward abdomen) 19. Check for and correct any obstructions or kinks in catheter 20. Dispose of gloves and soiled equipment in appropriate container 21. Assist patient into comfortable position 22. If necessary, don exam gloves and wash/dry perineum; dispose of equipment in appropriate container 23. Wash hands 24. Palpate bladder to note distension or patient discomfort 25. Document character & amount of urine in drainage system, type & size of catheter inserted, amount of fluid to inflate balloon, & patient's tolerance of procedure

NG tube review sheet

Concepts to Remember: 1. The airway MUST be protected to avoid aspiration. Alert patients: • Must have gag reflex Comatose patients: • Cuffed endotracheal tube in place 2. Contraindicated in patients with facial trauma & skull fx (basal). 3. Considered a clean procedure. Procedure: Placement of Tube- 1. Identify patient, explain procedure, provide privacy 2. Wash hands & don clean gloves 3. Position patient in high Fowler's position; raise head of bed & support patient with pillows 4. Assemble & open equipment on bedside table; position self at right side of bed if right handed & vice-versa 5. Select nostril with greater airflow 6. Measure distance of tube insertion: (see illustration, p. 2) distance from tip of nose to earlobe to xiphoid process to sternum 7. Mark site with piece of tape or note distance from next mark 8. Prepare nose tape; curl tip (4-6 inches) of tube to curve 9. Lubricate tip & 4-6 inches of tube with water-soluble jelly 10. Instruct patient to extend neck and insert tip into naris (curved end pointing downward) 11. Continue to pass tube until resistance is felt; apply GENTLE downward pressure. DO NOT FORCE TUBE. 12. If resistance is met, withdraw tube, allow client to rest, relubricate and re-insert into opposite naris 13. Continue insertion by gently rotating tube toward opposite naris until just past nasopharynx 14. Instruct patient to flex head forward, dry swallow, suck on straw, or sip water (if fluids not contraindicated). Advance tube 1-2 inches per swallow. 15. If patient coughs or gags, stop advancement and allow patient to rest; encourage to breathe and sip; pull back tube 16. If coughing & gagging continues, visualize back of pharynx with pen light and tongue blade to identify 'kinking of tube' 17. If 'kinking' absent, continue to advance tube to determined length Checking Tube Placement- 1. Assess patient's ability to speak 2. Attach Asepto or Toomey syringe to end of tube and aspirate to verify gastric content 3. Measure pH of aspirate ( ph values between 1 and 4) 4. If no aspirate received, advance tube another 1-2 inches and repeat aspiration to verify 5. X-ray to confirm placement, if questionable Anchoring Tube- 1. After proper placement and verification, clamp tube or connect to suction 2. Tape tube to nose, avoiding pressure on nares 3. Fasten end of tube to gown by looping rubber band around tube in slip knot and pinning to gown 4. Elevate head of bed 30 degrees unless contraindicated or physician's order indicated otherwise 5. Reassure and evaluate patient 6. Wash hands 7. Document time, type of tube, patient's tolerance of procedure, confirmation of placement, character of gastric contents, drainage device Removal of Tube- 1. Explain procedure, position properly, provide privacy 2. Wash hands & don clean exam gloves 3. Disconnect drainage, loosen tape, unpin tube from gown 4. Instruct patient to take & hold a deep breath 5. Clamp or kink tube and pull out slowly, steadily & smoothly 6. Measure and note character & amount of drainage 7. Dispose of tube, drainage equipment, & gloves 8. Wash hands 9. Reassure & evaluate patient 10. Document removal of tube, character & amount of drainage, patient's tolerance, condition of nares & nose, presence of bowel sounds and abdominal assessment

ARDS etiology? Dx? Tx?

Condition that results from increased permeability of alveolar capillaires causing fluid to fill avleoli. Etiology: Shock, sepsis (most common cause), aspiration, severe pancreatitis, blood transfusion Diagnosis -Bilateral FLUFFY infiltrates on X-ray -----cuz of alveloar collapse owing to leakage of protein0rich fluid in intersitium and alveoli -Pa02/Fi02 <200 -Pulmonary wedge pressures of less than 18 mm HG (low filling pressure exclude diagnosis of pulmonary edema). You see 3 majors physiology alterations 1. Hypoxemia unresponsive to supplemental O2 2. Decreased pulmonary compliance, as the lungs become proressively stiffer and harder to ventilate 3. Decreased FRC Tx:02 treatment does not help the hypoxia in ARDS. Tx is hemodynamic support and treatemtn of underlying disease. in ARDS, mechanical ventilation with low tidal volumes and PEEP (positive and expiratory pressure) can improve oxygenation. PEEP improvess oxygenation by preventing alveolar collapse ath the end of expiration, which increases the functional residual capacity and decreases the work of breathing. Potential complications of PEEP include barotrauma and tension pneumothorax, alvelaor damage, and hypotension.

Cardiac transplantation contraindications

Contraindications: irreversible high pulmonary vascular resistance, irreversible renal insufficiency, diabetes with end-organ damage, current or recent malignancy (<2 years), cirrhosis, COPD, active infection, active PUD, high-risk for inability to copmly with medical regimen.

Treatment for Hyperkalemia mnemonic

Controlling K Immediately Diverts Bad arrhythmias Calcium Kayexalate, Diuretics Insuline and glucose Dialysis Bicarbonate Albuterol REmember you see PEAKED T WAVES Immediate treatment: only until definitive therapy is initated -IV clcium gluconate: GOAL is immediate cardiac membrane stabilization! acts in minutes -Insulin, albuerol, and bicarb all shift K into cell. acts in minutes Definitive therapy -Kayexalate, diuretics, and dialysis remove K+ from body

Carpal Tunnel

Cuased by median nerve compression within the tight confines of the carpal tunnel, causing numbness and pain in median nerve distribution. If long-standing and severe, atophy of thenar muscles may be seen. Usually worrse at night. SOmtimes patient pain/numbness can go as far back as the shoulder. Associated conditions: hypothyroidism, diabetes, repetitive use of hands in certain activities, pregnancy, recentr trauma Dx -Tinel's sign: tap over median nerve at wrist crease, causes paresthesias in median nerve distribution -Phalen's test: palmar flexion of the wrist for 1 minute; causes paresthesias in median nerve distribution -EMG and nerve conduction velocity study for definitive diagnosis Tx: 1. Wrist splint should be worn at night during sleep. Thepruprse is to prevent wrist flexion during sleep (which compresses the nerve) 2. Antiinflammatory medications (NSAIDs) 3. Local corticosteroid injection 4. Surgical release is very effective.

Breast cysts

Cysts are common lesions in the breasts of women in their thirties and forties and carry very low risk for malignancy. Simple cyst: almost never associated with malignancy Complex cyst; may be associated with malignancy and aspiration is usually recommended. If the cysts disappears with aspiration and the contents are not grossly bloody, the fluid does not need to be sent for cytologic analysis. If the lesion does not completely disappera or recurs multiple times after aspiration, then the fluid should be sent for cytology. Excision of a cyst is indicated if the cytologic findigs are suspicious for malignancy.

DVT stuff

DVT PRophylaxis -LMWH -LOW dose unfracitonated heparin -Complression boots *DVT stuff to remember, when and when maybe not do anticoagulation DVT below the knee (distal). Calf vein clot. -You can choose whether to anticoagulate or just wait. Many of these just dissolve be themselves within 14 days. So just send serial duplexes to make sure it's not moving. DVT above the knee (proximal). Uworld says iliofemoral vein clot. -If DVT moves above the knee then you have to anticoagulate. Tx is unfractionated heparin, LMWH, or fondaparinux for at least 5 days. Long-term anticoagulation for patients with DVT is achieved with Warfarin. Initial evaluation of possible DVT? Ultrasound for initial in ppl with moderate to high risk. If patient has low probablit, a D-dimer can further assess risk, with an ultrasound peforemd only in patients with a positive D-dimer. Posivie D-dimer is over >500 ng/ml.

adrenal insufficency?

Daily prednisone >20mg for >3weeks can suppress the HPA axis. SO if the person has surgery they can go in adrenal crisis. These patients require stress-dose perioperative steroids.

Patients with large ileostomy outpus are at risk for what?

Dehydration w/ hyponatremia, hypokalemia and NG metabolic acidosis. Should be managed with fluid replacement and stool-bulking agents.

Algorithm to evaluate hemoptysis Person has massive hemoptysis, what do you do?

Depends if its mild/moderate or massive (>600ml/24hr). If mild/moderate---> cxr, cbc, coags, renal function, UA--->CT scan plus/minus brochoscopy----> treat cause; peristent bleeding treated via bronchospic intervntions, embolization, or resection. If massive---> secure airwaya and breathing. If bleeding stops---> pick up where "cxr" starts in "mild/moderate" paragrapsh. I bleeding doesn't stop, go straight to treat cause; persistent bleeding treated via bronchospic interventions, emboilization, or resection. The greatest danger in masive hemoptyis is not exsanguination but asphyxiation due to airway flood with blood. INitial managemetn involves establishing ADEQUATE PATENT AIRWAY, maintaining ventilation and gas exchange, and ensuring hemodynamic stabilityu. Patients shoudl be placed with the bleeding lung in the dependent position (lateral position). Bronchoscopy is the proceudre of choice to identify the site and attempts early theraputic inervation. If this faily then go to pulmonary arteriography which can precisley identify the bleeding vessels and do therapetuci embolization. IF this fails then do thoracotomy.

Prosthetic joint gets infected? what is causing it most likely

Depends on if within 3 months of arthroplasty or more than than 3 months <3 months -Staph aureus, Gram neg rods, anaerobees. >3 months -Coagulase negative staph (such as Staph. epidermidis), Propionibacterium species, enterococci.

What is the Mohs surgery and whats the advantage?

Describes a technique for resecting either basal or squamos cell carcinomas on the face in order to achieve the optimal cosmetic result. Resection of the tumor is performed in small incrememtns with immediate frozen section analysis in order to ensure negative margins. The disadvanatage of the Mohs technique is the longer time is required. There is no difference in cure rate between Mohs surgery and wide local excision of a basal cell carcinoma. Wide local excision can be performed on all types of skin cancers.

When do you use imaging and not use it for appendicits?

Diagnses is based on classic clinical and lab reatures of appendicits, its highly specific. Does not warrant further imagin before surger. However, if patients who do not have the typical features of appendicits or those with other possible causes of RLQ pain (diverticuliti, IBS, ileitis) should hvae appropriate imaging with computed tomography or ultrasonography. *Ultrasonocgraphy is generally best modality if you're assessing pelvic pathology, whereas a CT scan is best way to assess nongynecologic abdominal procesess.

Radiation proctitis?

Diarrhea, rectal bleeding, tenesmus and incontinenece. Later, strictures and fistulae may form.

Crohn's disease

Disease patterns: Crohn disease can be intra-abdominal, perianal, or both. Intra-abdominal Crohn disease usualy results in one of three predominant patters: stricture, perforation, or inflammation. Perianal disease results in anal strictures, fistulas-in-ano, and abscesses. Crohns can affect small bowel and colon. A Ct scan is useful initial imaging study for assessing severity and extent of disease and for detecting intra-abdominal abscesses. Medication: Maintenance therapy or therapy for active flre-ups. Medical management can be done obstruction, fistulas, and inflammation. So remember than enterocutanoous fistulas don't awlays require surgical treatment. If pt. otherwise doing fine, trial of TNF alpha inhibitor can be appropriate, can close fistula. 5 Aminosalicylate derivatives: mild to moderate disease mainetnance therapy Metronidazole: mild to moderate disease meaintenance therapy Steroids: Moderate to severe disease: acute flares Azathrioprine and 6-MP: moderate to severe disease maintenance therapy MTX: Moderate to severe disease maintenance therapy Cyclosporin A: severe to fulminant disease Anti-TNFs: moderate to severe disease or severe perianal fistulizing disease. Surgical therapy: Options include bowel resection, strictureplasty and abscess drainage. *Obstruction from subacute inflammation can be resolved with anti-inflammtory and immunomodulators . In contrast, fibrotic strictures cannot be resolved with medial management and generally require surgical therapy. *One of the potential longer term comlications of repeated bowel resection is development of short bowel syndrome, that is, malabsorption requiring permanent TPNtherapy.

What is slipped capital femoral epiphysis?

Displacement of the femoral head on the femoral neck due to disrupation of the proximal femoral growth plate. Commonly seen in obese adolescent boys . THe physis( physical junction between the femoral head and necK) weakends during early adolescence because it is rapidly expanding and primarily composed of cartilage, which does not possess the strength of bone. When exposed to excessive shear stress, which is magnified by obesity, the physis fractures and the femoral head slips posteriorly and medially relative to the femoral neck. Patients typically present with hip or knee pain of insidious onset that causes limping. Knee pain (referred pain), not hip pain, is a common presenting complaint with this condition. PE shows loss of abduction and itnernal rotation of hip as well as external rotation of the thigh while hip is being flexed. These patients should be promptly treated with surgical pinning of the slipped epiphysis where it lies (in situ) in order to lessesn the riss of avascular necrosis of the femoral hed and chondrolysis.

Diverticulosis vs diverticulitis?

Diverticulosis should be distinguished from diverticulitis, which is characterized by abdominal pain and infectious symptoms, usually without associated bleeding.

Surgical site infections

Divided into superficial and deep depending on whether theh fascia is involved Superficial surgical site infections: Infectious processes above the fascia. Treated primarily by wound exploration and drainage of the infected material, although systemic antibioitc therapy may be neeeded when there is extensive surround celluliits or if the patient is immunocompromised. Deep surgical stie infections: Infectious processes involveing the fascia. The presence of deep surgical site infection mandates an evaluation for possible deep surgical space infection. A CT scan should be considered if deep surgical space infeciton is suspected. Deep surgical space infections: Also referred as intra-abdominal infections in the setting of posteoperaive abdoinal surgery. Should get CT scan to look for intra-abdominal abcess. They include... 1.secondary peritonitis (spillage of nedogenous microbes ito peritoneal cavity followign viscera perforation) -Tx: fix source of spillage and early initiation of preemptive antibioit c theray 2.tertiary peritonitis (occurs in patients who faili to recover from intra-abdominal infections depsite surgical and/or antimicrobial therapy because of diminsehed hos peritnoeal respon) - Tx:antibiotics 3.intra-abdominal abscesses. Tx: percuteanous drainage with CT guidance.

Blunt aortic injury?

Do chest x-ray if you suspect. You will see mediastinal widening, which is the most sensitive finding. You may have deviation of trachea or nasogastric tube.

APproach to alkali chemical burns?

Do immediate removal of alkali agent with large volumes of water. Attempts to neutralize the alkali with ewak acids are not recommended because the heat released by neutralization reaction induces further injury. Only do operative debridement for really bad burns.

Patient has congenital cystic dilations of extrahepatic ducts?

Do surgery in all cases. Present recommendatin is for complete resection of the cyst with cholecystectomy and Roux-en-Y hepaticojejunosotomy.

Pressors for shock

Dobutamine: increased CO and decreased SVR. --typical use: cardiogenic shock Isoproterenol: similar to dobutamine. increased CO and decreased SVR. --typical use: cardiogenic shock with bradycardia Milrinone: technically not a pressor, but important inotorope used in ICU --phsophodiesterase inhibitor. result increased CO and decreased SVR. --typical use: heart failure/cardiogenic shock Dopamine: dopamine has different action depending the dose. ----low dose: dilates renal vasculature. typical use: none ---- intermediate dose: cardiac dose. increases CO and used for cardiogenic shock. ---high dose: increases SVR a lot. Typical use: septic shock (replaced by norepinephrine). Norepinhephrine --- alot of SVR increase, increased CO. Typical use: septic shock. Epinephrine ---increases SVR a lot , +/- increased CO, bronchodilation. ---Typical use: anaphylaxis, septic shock, cardiopulmonary arrest *-Low infustion rates: Beta 1 effects predominate causing increased heart rate, stroke volume, and contractility. -Higher infusion rates: alpha affects predominate, increasing BP and SVR. Phenylephrine -pure alpha agonist. ----increases SVR a lot. Typical use: septic shock, neurogenic shock, anesthesia-induced hypotension

Best initial test for screening patients with carotid disease?

Doppler ultrasonography (duplex).

You're worried abou thyroid storm happening in surgery, what can you do to prevent it?

Drops of Lugol iodide solution daily beginning 10 days preoperatively should be done if person is pregnant Propylthiouracil or methimazole can also be used preoperatively but are contraindicated in pregnant women.

A common postgastrectomy complication?

Dumping syndrome. Pathophys involves rapid emptying of hypertonic gastric content into the duodenum and small intesting. This process leads to the fluid shift from IV space to the small intestine,release of intestinal vasoactive polypeptides, and stimulation of autonomic reflexes . Symptoms usually diminish overtime and dietary changes are helpful to control the symptoms. In resistanct cases, octreotide should be tried. Reconstructive surgery is reserved for intractable cases. Patient gets postprandial abdominal cramp,s weakness, light-headendess and diaphroesis.

ERCP, HIDA, Ultrasound usefulness in gallbladder problesm

ERCP uses a fiberoptic camera and fluoroscopy to visualize the biliary and pancreatic ducts for diagnostic and theraputic purposes. It would be reasonable to perform ERCP if the patient had choledocholithiasis or a gallstone in the COMMON BILIARY DUCT (CBD) causing CBD dilatation. In such cases, sphincterotomy can help facilitate passage of stones. The HIDA scan is a nuclar medicine study used to diagnose gallbladder obstruciton Whit this technique, a nuclear tracer is injected into the blood and is excreted from the liver into the bile. Typically, the radiotracer can be seen pooling in a normal gallbladder. Failure to visualize tracer in the gallbladder is suggestive of obstruction. Ultrasonography demonstrates cholecystitis in this patient, so the scan would not add to the diagnosis. A HIDA scan can be used in patients where ultrasonogram cannot make a clear diagnosis of acute cholecystitis. Acute cholecystisis: Tx is laproscopic cholecystectomy within 72 hours during same hospitilization.

Early signs of sepsis?

Early sepsis is a physiolgoically hyperdynamic, hypermetabolic state representing a surge of catecholamines, cortisol and other stress-related hormones. Changing mental status, tachypnea that leads to respiratory alkalosis, and flushed skin are often the earliest manifestations of sepsis.

Venous ulcerations? Venous

Edema, stasis dermatitis and venous ulcerations result from lower extremity venous insufficency due to valve iincompetenece. Such disease classically occurs on the medial leg superior to the medial malleolus. Venous insufficiency is the most common cause of lower extremity edema. It classicaly worsens throughout the day an resolves overnight when the patient is recumbent. ---Most common cause of venous insufficiency is Venous valvular incomeptetnce reulst sin pooling of venous blood and increased pressures in postcapillary venules. This increased pressure damages capillaries, causing fluid loss, plasma proteins and ertyhrocytes into tissue. Erythrocyte extravasation causes hemosidern deposition and the classic coloration of stasis dermatitis.

Calot's triangle

Edge of liver Common hepatic duct Cystic duct

Best way to do T staging of esophageal cancer?

Endoscopic ultrasound.

Escharatomy vs fasciotomy?

Eschar (dead tissu) formation (like from a burn) can sometimes compromise both blood and lymph circulation, resulting ina decrease in pulses and significant edema distal to the burn. This can present similarily to compartment syndrome. Do a Doppler to document peripheral pulses and estimate the tissue compartment pressure. A pressure of 25040 mm Hg is the threshold to perform escharotomy. Escharotomy involves incision of only the schar layer. Patient should be evaulated for clinical signs of adeqate perfusion after escharotomy, and fasciotomy (incision through all fascial layers) should be done if there is no sing of relief.

Potential locations of major blood loss in trauma?

External-exam can identify pleural space-chest x-ray can locate intraperitoneal-FAST or DPL retroperitoneal-FAST pelvic-pelvic x-ray can identifiy bony fractures and/or discolcations; which are the primary cause of extraperitoneal pelvic blood loss soft tissue-major long bone fractures are associated with destruction and bleeding from surrounding soft tissue; this type of bleeding can be identified on exam and x-ray *In hemodydynamically unstable patient, CT scan is not indicated. Cuz its importatnt to identify life-threatening problems in a timely fashion without having to transport patient to radiology suite.

Most commonly used predictor of postoperatie pulmonary reserve.

FEV1

Fistula that fails to close mnemonic

FRIEND Foreign body Radiation Infection Epithelialization Neoplasm Distal obstruction

Hereditar colon cancer syndromes

Familiar polyposis coli -get bunches of polys by age 25 -APC gene -adenomatous polyps are the type of polyp -Tx;: prophlylactic total colectomy HNPCC (or Lynch syndrome) -Auto Dominant. -3 or more relative of patient devleop colon caner -there is Lynch 1 (hereditary site specific colon cancer) and Lynch 2 (cancer family syndrome). Remember that Lynch II is distinctly associated with a high risk of extracolonic tumors, the most common being endometrial carcinoma, which deveops in up to 43% of females in affected families. *The Amsterdam critera are used to clinically daignose HNPCC and consist of... 1. at least 3 relatives with histologically confimred colorectal cancer, one of whom is first-degree realtive of other 2. involvment of at least 2 sucessive generations 3. diagnoses of at least one of cancers before age 50 *screening for patients with HNPCC should start at 20 or 25 or 10 years earlier than the youngest family member with colorectal cancer. Gardner's syndrome -AD -Polyposis coli, supernuemerary teeth, osteomas, fibrous dysplasia Peutz-Jeghers syndrome -multpile polyposis of fmall intestine with multiple pigmented melanin macules in oral mucosa -hamartomas are the type of polyp *BRCA1 (usually receptor negative) is the one that is associating with colon cancer * BRCA2 (usually hormone receptor positive)

What nerve is responsible for knee extension and hip flexion? what is alos the sensory loss if its damaged?

Femoral nerve. Also provides sensation to the anterior thigh and medial leg via the saphenous branch.

Types of hypnatremia

First calculate serum osmolarity to determine classification of hyponatermia. Explained on page 246-247 on First Aid Medicine HYPOTONIC HYPONATREMIA: Hypervolemic: CHF, cirrhosis, nephortic: Euvolemic: -------UNa>20, Uosm<300. Psychogenic polydipsia, beer potomanis -------UNa>20, Uosm>300.(can think of it like urine osmolality > than serum osmolality). SIADH -------------Tx: Fluid restriction and salt intake. Consider demeclocycline if this fails, it acts on renal collectin tubule to decrease responsiveness to ADH..but it can be nephortoxic and is rarely necessary. If the SIADH is severe or resistant to intial treatment, use hypertonic saline. Conivaptan and tolvaptan also used somewhere there. Hypovolemic: Lose water and salt, but more salt than water. -------Renal causes: diuretics, UT obstruction, etc UNa+ >20. ------ Nonrenal causes: vomiting, diarrhea, dehydration etc. UNa+ <10. ISOTONIC hyponatremia "pseudohyponatremia" HYPERTONIC Hyponatremia: you have alot of glucose or mannitol added to serum and thus water from ICF comes into serum and make you look hyponatremic although the Na in serum really hasn't changed. Still have symptoms of hyponatremia though. There is a corrected Na equation, sees what Na+ concentraion would be if you "corrected" for hyperglycemia. Its Na=((glucose-100)/100) x1.6 *quick way, for every 100 over 150 glucose, that should decrease sodium by 1.6. So increase your Na by 1.6 for every 100 glucose over 150 and that should be your corrected sodium. This came from a medstudent. Treatment of this stuff. Normal saline case of hypovolemic hyponatremia. Na and water restriction in hypervolemic hyponatremia.

Explain hydrocele and management

Fluid colection within the processus or tunica vaginali-the peritoneal projection that accompanies the tesis during its descent into the scrotum. When it fails to obliteralte, pertoneal fluid may accumulate within the proessus vaginalis causing a communciating hydrocele. A noncommunicating hydrocele would be the fluid collecting when it has properly obliterated its communication with the pertoneum. Hydroceles will transulliminate. Most hydrocles will disappear by age 12 months and can be safely observed during that period. If they do not resolve spontaneously, they should be removed surgically due to risk of inguiinal hernia.

Hemodynamically unstable patient in whom blunt abdominal trauam is suspected, what do you so?

Fluid resuscitate then do FAST (focused assessment with sonography for trauma). If there is intraperitoneal blood, the patient should then undergo urgent laparotomy for surgical repair.

Fondaparinux? Rivaroxaban?

Fondaparinux? Rivaroxaban? FACTOR Xa INHIBITOR -Oral-Rivaroxaban (Xarelto). Used for prophylaxis, Tx of VTE. Also for nonvalvlar A-fib. -Oral- Apixaban. Also can be used for prophylaxi, Tx of VTE and for nonvalvular A-fib. -Injection-Fondaparinex DIRECT THROMBIN INHIBITOR -Oral: Dagagtroban (Pridaxa). Approved for DVT, PE, nonvalvular A-fib. NOT used for prophylaxis for VTE. -IV: Argotobran: continuous IV drip. Usually used in case if patient has HIT -IV: Bivalirudin: sometimes used in ACS cases but not often LMWH Enoxaparin (Lovenox) Unfortunatley, if patient has kidney problems you probably wanna stay away. these are cleared renally. LMWH vs Coumadin -cancer patietns w/VTE. LMWH is better than Coumadin. -no data comparing LMWH and oral 10as. -for valvular A-fib, the only anticoag approved is Coumadin.

ACL tear

Forceful hyperextension injury to knee or a noncontact torsional injury of the knee during decerlation. Effusion is seen rapidly following injury. Lachman's test, anterior drawer test and pivot shift test are used for clincal diagnosis.

Distended colon DDx?

Foreign body, tumor, colitis, cecal or sigmoid volvulus Sigmoid volvulus: Use prostosigmoidoscopy which is preferable to barium enema cuz you can treat sigmoid volvulus vis rectal tube decompression with the sigmoidoscope. Cecal volvulus: barium enema for diagnoses. Tx: procedure of choice is right hemicolectomy *Excruciating abdominal pain and distenion can be presenting signs of distended colon. If bad there is risk for colon rupture and in that case you wanna do emergency celiotomy before rupture.

lady presents with abnromal mammogram in clinic, what her risk for cancer?

GAIL model: age, age of menarch (menarch before age 12) , age at first birth (nulliparityr or first birth after age 30 ), menopause after age 55, , how many first degree relatives have it, BRCA status, etc.

Dressing stuff

GAUZE Gauze dressings are made of woven or non-woven materials and come in a wide variety of shapes and sizes. Use on: infected wounds, wounds which require packing, wounds that are draining, wounds requiring very frequent dressing changes. Pros: usually readily available; may be cheaper than other dressing types; can be used on virtually any type of wound. Cons: must be changed frequently, which may add to overall cost; may adhere to the wound bed; must often be combined with another dressing type; often not effective for moist wound healing. TRANSPARENT FILMS Transparent film dressings allow oxygen to penetrate through the dressing to the wound, while simultaneously allowing moisture vapor to be released. These dressings are generally composed of a polyurethane material. Use on: partial-thickness wounds, donor sites, minor burns, stage I and II pressure ulcers. Pros: conforms to the wound well, can stay in place for up to one week; aids in autolytic debridement; prevents friction against the wound bed; does not need to be removed to visualize the wound; keeps the wound bed dry and prevents bacterial contamination of the wound. Cons: may stick to some wounds, not suitable for heavily draining wounds, may promote periwound maceration due to its occlusive nature. FOAMS Foam dressings are less apt to stick to delicate wound beds, are non-occlusive and are composed of a film coated gel or a polyurethane material which is hydrophilic in nature. Use on: pressure ulcers, minor burns, skin grafts, diabetic ulcers, donor sites, venous ulcers. Pros: comfortable, won't adhere to the wound bed, and highly absorbent; allow for less frequent dressing changes, depending on the amount of wound exudate; come in many shapes and sizes. Cons: may require a secondary dressing to hold the foam in place; if not changed often enough may promote periwound maceration; cannot be used on wounds with eschar or wounds that are not draining; some foams may not be suitable for certain wounds, such as those that are infected or are tunneling. HYDROCOLLOIDS Hydrocolloid dressings are very absorbent and contain colloidal particles such as methylcellulose, gelatin or pectin that swell into a gel-like mass when they come in contact with exudate. They have a strong adhesive backing. Use on: burns, pressure ulcers, venous ulcers. Pros: encourage autolytic debridement; provide insulation to the wound bed; waterproof and impermeable to bacteria, urine or stool; provide moderate absorption of exudate Cons: leave a residue present in the wound bed which may be mistaken for infection; may roll over certain body areas that are prone to friction; cannot be used in the presence of infection. ALIGNATES Alginate dressings contain salts derived from certain species of brown seaweed. They may be woven or nonwoven and form a hydrophilic gel when they come in contact with exudate from the wound. Use on: venous ulcers, wounds with tunneling, wounds with heavy exudate. Pros: highly absorbent; may be used on wounds that have infection present; are non-adherent; encourage autolytic debridement. Cons: always require a secondary dressing, may cause desiccation of the wound bed, as well as drying exposed tendon, capsule or bone (should not be used in these cases). COMPOSITES Composite, or combination dressings may be used as the primary dressing or as a secondary dressing. These dressings may be made from any combination of dressing types, but are merely a combination of a moisture retentive dressing and a gauze dressing. Use on: a wide variety of wounds, depending on the dressing. Pros: widely available; simple for clinicians to use. Cons: may be more expensive and difficult to store; less choice/flexibility in indications for use. Other dressings available on the market include dressings containing silver or other antimicrobials, charcoal dressings and biosynthetic dressings

GVHD

GVH is donor lymphoctes mounting immune response against host tissue. Common in bone marrow transplantation. Tx is more aggressive immunosuppresion. Current clnical practice includes depletion of lymphocytes from the marrow graft in order to prevent GVHD. Also happens with liver transplant. GVHD has not been described following heart, lung, pancreas, or kidney tranpslant.

Peptic Ulcer disease

Gastric ulcer pain made worse by food Duodenal ulcer pain is better by food -commonly associated with acid hypersecrtion. Acid-reductino therapy results in a high rate of ulcer reductino but eradication of H.pylori helps mainatin a long-term ulcer cure. Most common causes are H. pylori infection or ingestion of NSAIDs. Management algorthim: after H&P and Lab studies...do endoscopy and test for H.pylori (serology, urea breath test, urease assy, cultures). If H pylori found then treat that with tripple therapy w/ Omprazole, Amoxicilin, Clarithromycin, Metronidazole. If H.pylori is negative, then probably NSAID use is culprit. Stop the NSAID use can use prostaglandin analogue. *remember though people with NSAID induced ulcers can also have H.pylori involvement Surgical therapy: surery indicated for both kinds of ulcers (duodenal and gastric) if complications arise like GI hemorrhage, perforation, intracrtable pain, obstruction. Ulcer disease is intractable if t persists for more than 3 months despite active drug therapy or th eulcer recurs within 1 year after healing or if ulcer disease is characterized by cycles of prolonged activity with brief remission. *Gastric ulcers should undergo biopsy eraly in evaluation process cuz of risk of carcinoma. *Gastrectomy, vagotomoy (more popular now i think) are some surgeries done. -Only ulcers associated with acid hypersecretion require a vagotomy (type 2-body of stomach, with concomitant duodenal ulcer) or (type 3-prepyloric). So Type 1 (in the body and lesser curvature) and type 4 (near gastroesophageal junction) ulcers do not require vagotomy.

When is bronchoscopy used?

Generally indicated if there is conern for obstructing mass that may be causing rescurrent pneumonias in the same anatomical location. Even then, CT scan of chest is the preferred initial study, so bronchoscopy used when CT has given unclear results. 2nd thing its used for is recurrent epidosed of hemoptysis. *alos used used of aspiration pneumonitis if there is particular matter in the tracheobronchial ree that needs oto be removed ALSO, brochoalveolar lavage can be done with bronchoscopy. You basically do BAL for two reasons, evaulation of suspected malignancy and opportunistic infection.

Common peroneal nerve?

Gives rise to the superficial and deep peroneal nerves. These two nerves supply the muscles of the anterior and lateral leg. These nerves provide sensation to the anterolateral leg and dorsum of foot.

Hepatic tumors

HEPATIC ADENOMA Hepatic adenoma-benign tumor msot often seen in you or middle aged women taking OCPs. Severe intra-tumor hemorrhage and malignant transformation are the most dreaded complications. Common Physical exam findings are palpable abdominal mass, hepatomegaly, or jaundice. Alk phosph and GGT are most commonluy elevaetd with those patients with intratumoral bleeding or multiple adenomas. HEMANGIOMA Most common benign liver tumor. Frequently asymptomatic. Spontaenous rupture is rare. Biopsy is contraindicated cuz can result in life-threatening hemorrhage. FOCAL NODULAR HYPERPLASIA Second most common benign liver tumor, occuring mostly in reproductive age women. Most FNHs are asymptomatmic, if symptomatic then excise. * "central" scare pattern on CT scan, CT can be used to differentiate from hepatic adenoma. Biopsy may be needed if tumor can't be differnetiated from hepatic adenoma. PRIMARY LIVER TUMOR -Turmor originating in levers. Most common are hepatocellular carcinoma and cholagniocarcinoma. SECONDARY LIVER TUMOR: tumor in liver that came from other spot

Explain pros and cons of thoracic epidural narcotics

Have become an increasingly popular means of postoperative pain relief in thoracic and upper abdominal surgery. Local action on opiate receptors esnures pain relief. The less lipid-solube opiates are effective for long periods. Their slow absoprtion into the circulation also ensures a low incidence of centrally mediated side effects like respiratory depression or itching. When these do occu, IV naloxone is a good antidode. One poorly understood side effect if profound reduction in gastric activity.

Interventions in lowering ICP

Head elevation: increased venous outflow from the brain Sedation: Decreased metabolic demand and control of hypertension IV mannitol: Extraction of free water from brain tissue--> osmotic diuresis Hyperventilation: C02 washout--> cerebral vasoconstriction Removal of CSF: Reduction of CSF volume/pressure

How is heparin given

Heparin -Therapeutic: IV bolus of 70-80, then continuous IV infusion 15-20/hr. Now monitored using antifactor 10a levels. So anytime you see continuos drip you know its therapeutic. -Prophylactic: Low dose heparin that is given by injection, every 12 hours. LMWH -Therapuetic: given as a weight adjusted dose. 1mg/kg BID. -Prophylactic dose: once a day.

PE

Heparin does not dissolve already existing clots, rather it prevents futuer ones from forming. Signs and symptoms of PE -Are nonspeicific and highly variable. It is a diagnosis that should be suspected in any patient who presents with some combination of sudden-onset shortness of breath, pleuritic chest pain, low-grade fever, and hemoptysis. Tachypnea, tachycardia, and hypoxemia are common exam findings. Calf swelling and Virchow's triad (stasis, endothelial injury, and hypercoagulable state) are not always present in PE. -When a PE likley what do you do? First start anticoagulation (LMWH or unfractionalted heparin) BEFORE you do a CT angio because although it won't do anything for existing clot it will help stop any more from happening. THen do the CT angio for diagnoses. D-dimer is useful to EXCLUDE PE from patients where its UNLIKELY they have PE. Not realy useful ruling in though. DIAGNOSIS OF PE Helical CT: most common, fast. Downside: IV contrast can damage kidneys PUlmonaryangiogram: The gold standard for detection of PE. Very sensitive. Downside: Invasive. V/Q: Looks for perfusion at site of PE. TREATMENT OF PE: Heparin or LMW heparin-usually only therapy in STABLE patients Thrombolysis or pulmonary embolectomy if hemodynamically UNSTABLE TREATMENT OF CHRONIC PE: Prolonged: Orally anticoagulated with warfarin for 6 months Patients with multiple thromboembolic episodes and patients with an ongoing risk factor may need anticoagulation for indefinite period

Solitary Pulmonary Nodule in lung algorithm

High malignancy risk---> straight to surgery excisiton Intermediate risk a. if smaller than .8 cm---> serial CT scans b. if larger than .8cm-->FDG-PET* ( 18-flurodeeoxyglucose positron emission tomography scan)----> if suspicious for malignancy then surgicaly excise, if not suspicious for malignancy then do serial ct scans *the very first thing you do with nodule is look at old x-rays and compare *it looks like FDG-PET is the preferred study here if available, but flexible bronchoscopy or percutaneous needle biospy of pulmonary nodule seems like can be done in place of or with this test. I'm not really sure. *if cancer, then 5 most common sites of metastsis shjould be examines: contralateral and noninvolved ipsilateral lung, liver, adrenals, bone, and brain Low malignancy risk: serial CT scans

When would a tube cholecystectomy be indicated?

High risk, critically ill patients with multisystem disease have a much bigger operative risk for operative intervention. Tube cholecystectomies can be performed under local anesthesia if a cholecystectomy needs to be done.

Cyclosporine mechanism?

Highly effective immunosuppressive agent produced by fungi. more specific than anti-inflammatory agents like steroids or antiprolifertive agents like azathioprine. The effectiveness of cyclosprone in preventing allograft rejection is releated to its ability to inhibits interleukin 2 production.

Grade vs Staging in cancer?

Histologicg grading reflects the degree of anaplasia of tumor cells. Tumors in which seems to have prognostic valus is soft tissue sarcoma, transitional cell cancers of bladder, astrocytoma and chondrosarcoma. Staging is based on extent of spread rather than histologic apearance and is more relevant in predicting the course of lung and colorectal cancers.

Syndromes associated with lung cancer

Horner's syndrome Pancoast syndrome-causes shoulder pain radiation to arm. Can involve nerve roots C8 and T1 (causing pain of the dermatomes) as well as sympathetic trunk. Symp. trunk can give you Horners. Superior vena cava sndrome-obstructs vena cava and venous returne. ONe with facial swelling and other stuff SIADH Eaton-lambert Trousseau's syndrome-venous thrombosis associated with metastat cancer PTH-like hormone-results in high calcium, low phospohate, seen in squamos cell lung cancer

Primary hyperparathryoidism

Hyeprcalcemia "stones, bones, groans, an dpsychiatri overtoens" A chloride-to-phosphorous ratio of greater than 33:1 is consistent with a diagnosis of primary hyperparathyroidism. Most commone metabolic complications: stones *Hyperparathryoidism is associated with high secretion of calcium in urine, low serum phosphate, high serum chloride, and low seru mbicarb.

Tumor Lysis syndroem

Hyper..phophatemia, kalemia, uricemia Hypo...calcemia (cuz released phosphate binds to calcium) Allopurinol may greatly reduce the possibility of acute urate nephropathy. Chemo is big cause and the mediator is cytotoxic T cells.

Phases of septic shock?

Hypodynamic phase that is characterized by hypovolemia and myocardial depression Hyperdynamic phase that follows fluid resuscitation and is characterized by a normal to increased cardiac output

Treatment of colon cancer

I think... Do hemicolectomy in order to remove draining lymph node basin as well. If lymph node involvment has been confirmed then you wanna do adjuvant chemotherapy. Liver is most common site of metastases from colon cancer. You can do hepatic resection if liver is the only site of metastsis and lesion is able to be resected with clear margins.

Clinical Skills stuff for Surgery/osce

IDEAS. Indications, Don't if.., Education, Adverse events, Steps NASOGASTRIC TUBE -indications:Bowel obsruction, enteral nutrition and gastric lavage, gi bleeding -contraindications: esophageal varices, basilar skull fracture, perforation, high INR Steps: -ask patient which nostril he/she prefers, if has had nasal surgeries, polyps, etc -measure the length of tube you need by putting from nose to ear to xyphoid process, then put tape around tube to mark it -lube up tube, have pt. extend neck while you put in tube -when the tube has reached the nasopharynx part, have pt. flex neck (touch chin to chest) and take big swallows of water throgh straw to help with process -once in stomach, do taht thing where you listen to ttomach and quickly close the syringe using your body and listen for a sound -take syringe out of tube and put blue thing in -tape the NG tube to nose -go to X-ray to confirm its in right place -go back to room, attach suction, turn it on to lower intermittent suction JP DRAIN Complications: Fractured darin, stuck drain, adhesions, pain -uncap the grenade looking thing and then cap it again for something to do with suction -cut suture next to knot so that you don't have a lot of suture pieces everywhere -open up gauz packs, open lid for culture..this is getting stuff ready so that when you're sterile you don't have to -put on sterile gloves -take thos little gauze pads and apply pressure around drain while you slowly take it out -make sure the end doesn't come out flopping and hitting stiff because that end is sterile and you might want to cut it off in that sterile container you've already undone the lid for -put big gauze pad where hole is now, tape it FOLEY Indications for Foley at northwestern: surgery >4hrs, acute urinary retention, chronic catheters ABI -A low ABI means the blood pressure in the lower extremities is lower than the upper extremities, indicating that PAD is restricting blood flow to the legs -As ABI numbers decrease, severity of PAD increases - Be aware of known diabetics with calcified vessels and abnormally high ABI. Calculated by dividing the higher ankle (dorsalis pedis or posterior tibial) systolic pressure in each lower extremity by the higher brachial artery (left or right) systolic pressure ABI <.90= abnormal (diagnositc of ABI) 0.91-1.30= normal >1.30=suggestive of calcified and uncompressibnle vessels; additional vascular studies should be considered. For ARM 1. put the cuff about 1 inch above antecubital fossa, put gel there and put Doppler probe there and get an arterial pulse sound 2. inflate the cuff to about 20 mmg above when pulse stops 3. Deflate the cuff at a rate of 2 mm Hg per second until the first arterial pulse sound is heard. When this number is determined, deflate the cuff completely and record this systolic reading. 4. repeat on other arm For LEG 1. Apply the same blood pressure cuff snugly to the ankle on the same side of the body. 3. Palpate the area around the medial malleolus to find the posterior tibial (PT) arterial pulse. 4. If this pulse is palpable, apply Doppler gel to the area. If there is no palpable pulse, apply gel to the general area, turn on the Doppler, and move the probe around until the clearest arterial sound is heard. Keep the probe in that position. Continue inflating the blood pressure cuff as before, followed by deflation and reading. 5. Palpate the dorsal arch of the same foot for the dorsalis pedis (DP) arterial pulse. Apply the Doppler gel and use the Doppler probe as before. 6. Apply the blood pressure cuff to the opposite ankle and record the PT and DP pressures as before. DRESSING CHANGE Common Types of Dressings: 1. One layer- an adherent, occlusive dressing (Bioclusive, Op-Site): sterile, transparent polyurethane film 2. Dry sterile dressing- single or multilayered ( gauze 4 x 4s) applied dry over a clean incision with little or no drainage, secured with tape or circumferential wrap (kerlix, kling, elastic bandage 3. Three layer dressing- for moderate to heavy drainage a) Contact layer- nonocclusive (gauze), semiocclusive (foams, hydrogels, hydrocolloids), occlusive (fine mesh impregnated gauze: xerofoam, petrolatum) b) Intermediate layer- absorbs secretions which pass through contact layer ( gauze sponges, pads) c) Outer layer- holds contact and intermediate layers in place ( adhesive tape, elastic bandage, elastoplast, Montgomery straps, stockinette) 4. Pressure dressing- added to intermediate layer especially for plastics, ortho, and breast procedures (fluffed gauze, pads, cotton rolls, foam rubber 5. Wet-to-dry dressing- method of debridement to faciliate new tissue growth ( dressing materials soaked in saline and applied or packed into wound covered with dry pads & tape) Procedure: 1. Identify patient, ascertain allergies, & explain procedure 2. Provide privacy and comfortable environment 3. Position patient appropriately 4. Place disposable bag or waste container within reach 5. Apply mask, wash hands, and don clean disposable gloves 6. Remove tape, pulling parallel to skin and toward dressing 2 of 2 7. Remove dressings; do not dislodge drains or tubes 8. Note character and amount of drainage and appearance of wound 9. Dispose of soiled dressings in waste container and remove gloves 10. Open sterile dressing tray or sterile supplies onto bedside table 11. If wound is to be cleansed, open saline or prescribed solution and pour in to sterile basin 12. Apply sterile gloves; inspect wound avoiding contact with contaminated material 13. Cleanse wound with saline or antiseptic solution a) separate swab for each stroke b) clean from least contaminated to most contaminated area 14. Dry wound with gauze in same manner as above 15. Apply ointment if prescribed 16. Apply dry sterile dressings to incision or wound site (simple: telfa, gauze, ABD pad) 17. Apply tape, ties, or bandages 18. Remove gloves and discard in waste container 19. Assist patient into comfortable position 20. Dispose of all supplies in appropriate container 21. Wash hands 22. Document appearance of wound and drainage, if present, client's tolerance and type of dressing applied

Timeline of cause of postoperative fever

IMMEDIATE 0-2 hour: Prior trauma/infection, blood products, malignant hyperthermia 2-24 hour: nothing ACUTE 24 hour- 1 week: Nosocomial infections, Group A strept or C. perfringes, Noninfectious (MI, PE, DVT) SUBACUTE 1wk - >1 mo: other bacteria thats NOT gropu A strept or C perfringes and catheter infection, C dificile, Drug fever, PE/DVT DELAYED >1 month: viral infection, indolent organisms *Post op fever mnemonic thing.. Wind-atelectasis, pneumonia Water-UTI Wound-infection Womb-endometritis, uterin infection Walking-DVT Wonder: drugs,

Phases of wound healing and what things can affect it

INFLAMMATORY PHASE (1st week) begins immediately and ends within a few days. Inflammatory cells function in sterilizing the wound and secreting growth factors stimulating fibroblasts and keratinocytes in the wound repair process PROLIFERATION PHASE (3 days to 21 days) deposition of the fibrin-fibrinogen matrix and collagen, resulting in formation of the wound matrix and an increase in wound stregnth REMODELING PHASE (21 days to 1-2 years) capillary regression leads to a less vascularized wound, and with collagen cross-linking there is a gradual increase in wound tensile strength. Things that affect wound healing -Infections: delays wound healing -Nutrition: Vit C defieicney leads to inadequate collagen production. Vit A defieicny leads to bad epithelialization. Vit B6 causes impaired collagen cross linking. -Oxygen: collagen synthesis is augmented with oxgyen supplemenation -Corticosteroids: reduce wound inflamation, collagen synthesis, and contraction -Diabetes mellitus: Association with microvascular occlusive disease leading to poor wound perfusion

where do these traverse the diaphragm? IVC, esophagus, aorta?

IVC=t8 esophagus=T10 aorta=T12 "I ate 10 eggs at noon"

Explain ITP Idiopathic thrombocytopenia purpura

Idiopathic thrombocytopenia purpura is usually diagnosed after excluding other possible causes of thrombocytopenia. These patients should be tested for HEPATITIS C AND HIV as thrombocytopenia may be the initial presentation of HIV infection (up to 5%-10% of patients). Treatment of the underlying infection can affect the platelet count. Also, treatment usually involves systemic glucocorticoids for patents with severe thrombocytopenia (<30,000) or significant bleeding. For people who don't respond to steroids or require a very high dose or require chronic steroid therapy >1 year, splenectoy is recommended.

What is a risk of parotid surgery?

If it involves the deep lobe of the partoid gland it carries a sgnificant risk of facial nerve palsy resulting in facial droop.

Whistling noise during respiration following rhinoplasty?

If this happens one should suspect nasal septal perforation likely resulting from a septal hematoma.

What do you need to know about thermal inhalation injury?

In burn victims, clinical indicators of thermal inhalation injury to the upper airway and/or smoke inhalation injury to the lungs include burns on the face, singing of the eyebrows, oropharyngeal inflammation/blistering, oropharyngeal carbon deposits, carbonaceous sputum, stridor, carboxyhemoglobin level >10%, or history of confinement in a burning building. The presence of one or more of these indicators warrants early intubation to prevent upper airway obstruction by edema. SMOKE INHALATION These people should be suspected to have an acute carbon monoxide poisoning and are treated with 100% oxygen via a facemask. Early symptoms are nerulogical and include agitation, confustion, and somnolence. Dz confirmed clinically and by documenting en elevated carboxyhemoglobin level (>30%). Pulse ox is unreliable cuz can't differnetiale carboxyhemoglobin from oxyhemogloin. *important to bear in bind that the 30% can be reached within 3 minutes in a moderately smoky enclosed space faire.

Hernias

In groin, inguinal ligament divides inguinal hernias from femoral hernias. Inguinal hernias are further divided into indirect and direct hernias based on relationship to the inferior epigastric vessels. INDIRECT HERNIA: An inguinal hernia in which abdominal contents protrude through the internal inguinal ring through a patent processus vaginalis into the inguinal canal. In men, they follow the spermatic cord and may appear as scrotal swelling, whereas in women they may manifest as labial swelling. Indirect hernias are lateral to inferior epigastric vessels. Tx: Open repair with prosthetic mesh. "watchful waiting" is only reasonable if patient is minimally symptomatic and repair of hernia would not improve quality of life. DIRECT HERNIA: An inguinal hernia that protrudes through the Hesselbach triangle medial to the inferior epigastric vessels. FEMORAL HERNIA: A hernia that protrudes through the femoral canal below the inguinal ligament..unlike inguinal hernias. -incidence of strangulation in femoral hernias is high. Therefore, all femoral hernias, even asymptomatic ones, should be repiared. *Hesselbach triangle: defined by the edge of the rectus medially, inguinal ligament inferolaterally, inferior epigastric vessels superolaterally. Site of direct hernia. *Incarceration occurs if the abdominal contents become trapped. Tx: Reduction should be attempted first, if unsuccessful to uregent operation. *Strangulation occurs when the blood supply to the trapped contents beomes comprmosed, leading to ischemia, necrosis, and ultimatley perforation. *Intestinal obstruction can occur in an incarcerated or strangulated hernia.

How are maintenance fluids calculated?

In otherwise healthy individual, calculated based on body weights as 4 mL/kg/h for first 10kg, 2 ml/kg/h for 2nd 10 kg, and 1 ml/kg/h for every additional kg body weight

Hemothorax on x-ray?

Indistguishable from plerual effusion on x-ray. So blunting of costophernic angle.

Ludwig angina?

Infection of the submandibular and sublinguarl glands. The source of infection is most commonly an infected tooth, usually the second or thir mandibular molar. Asphyxiation is the most common cause of death in this disease. Presents with fever, dysphagia, odynophagia, and drooling.

Esophagitis etiologies

Infections: -viral: --- HSV, VZV have vesicles. CMV has intranuclear inclusions. Candida will have white plaques. ---Bacterial: -radiation -corrisives -medication: "pill esophagitis" (direct effects of a medication on esophageal mucosa). Common ones are potassium chloride, tetracyclines, bisphophonates, and NSAIDS. Tx: PPI is treatment of choice for erosive or severe esophagitis. THere is also nonerosive esophagitis which im not sure what the best therapy is for.

IVC indications

Inferior vena cavae filters may be used in aptients with lower extremity DVT when - anticoagulation is contraindicated (hemorrhagic stroke, active bleeding). - Anticoagulation failure (recurrent or extending thromboembolish while fully anticoagulated) is another indciation for IVC filter placement

Tenosynovitis

Inflammation of the tendon and its synovial sheath. It is usually seen in hands and wrist joints following a bit or puncture wounds. Patients have pain and tenderness along the tendon sheath, particulalry with flexion and extension movements.

Primary Survery in Trauma

Initial asessment and resuscitation of vital functions. Follow the prioritization. ABCDEF AIRWAY -use jaw thrust or chin lift initially to open airway -clear foreingn bodies -insert oral or nasal airway when necessary -obtunded/unconscious patients should be intubated (either perioral or perinasal) -when unable to intubate (cuz of failing or maxillofacial injuries) : do surgical airway: cricothyroidotomoy or tracheotomy. ---Cricothyrotomy is easier and quicker to perform than tracheotomy, does not require manipulation of the cervical spine, and is associated with fewer complications or tracheotomy. BREATHING AND VENTILATION -tension pneumothorax, flail chest, massive hemothorax, open pneumothorax CIRCULATION -place two large-bore peripheral IVs (14 or 16 gauge) -assess circulatory status (capillary refill, pulse, skin color) -direct pressure to control life threatening hemorrhage DISABILITY (neurologic status) -rapid neurologic exam -look at pupillary size and reactivity and level of consciousness using Glasgow Coma Scale or AVPU (alert, verbal, pain, unresponsive) EXPOSURE -completely undress the patient and assess every inch of body -don't forget to cover them and keep them warm afterwards FOLEY -foley contraindicated when urethral transection suspected, such as in case of pelvic fracutre ----signs of urethral transection: blood at meatus, "high riding" prostate, be suspiccious with any pelvic fracture -a retrorade urethrogram should be the first step in mangagemt of suspected urethral injury. *what to do about rupture of urethra? It can be partial or complete. If complete disruption, a suprapubic catheter is placed temporarily and definitive repair is delayed 4 to 6 months, at chich hematoma will likely have resolved.

Acute Abdomen

Initial lab evaluation should include: -CBC -Electrolyes -Amylase, lipase -ECG to rule out MI -LFTs for RUQ pain -B-hcG for all women of childbearing age -CXR and abdominal x-ray to look for free air *most common cause by far 90%, of free air under diaphragm is perforated peptic ulcer. other causes insclude trauma, mesenteric ischemis (usually under left hemidiaphragam) and large bowel perforation -Abdominal CT should be used after the above assessment is complete and the diagnosis remains elusive Surgical Causes of Abdominal pain RUQ -perforated duodenal ulcer *surgical emergency -acute cholecystitis -hepatic abscess -retrocecal appendicitis -appendicitis in pregnant woman RLQ -appendicitis -cecal diverticulitis -Mecke's diverticuliutus intussusception LUQ -splenic rupture -splenic abscess LLQ -sigmoid diverticulitis -volvulus DIFFUSE -bowel obstruction -leaking aneurysm -mesenteric ischemia Periumbilical -early appendicits -pain from small bowel obstruction SUPRAPUBIC -ectopic -ovarian torsion -tubo-ovarian abscess -psoas abscess -incarcerated groin hernia

What does the obturator nerve do?

Innervated the medial compartment of the thigh and controls adduction of the thigh. Provides sensation over the medial thigh.

IV insertion IV review sheet

Intravenous Catheter Insertion Introduces self with first/last name and title Wash hands Verifies patient using two patient identifiers Select appropriate equipment • IV fluid • Proper fluid • Clarity • Exp. Date • Appropriate size catheter • Blood tubes, tourniquet, gauze, chloraprep, 10mL saline syringe, heplock, tegaderm/tape Prepares equipment • Cut tape if needed • Connect/Prime IV tubing • Patient positioning **Takes body substance isolation precautions prior to venipuncture** Applies tourniquet • Appropriate distance from site • Allows for gravitational dilatation prior to applying tourniquet Palpates suitable vein • Site selection dependant on chief complaint o type and amount of fluid/medication o studies/tests anticipated o condition of patient Cleans site appropriately Performs venipuncture • Inserts stylette • Notes or verbalizes flashback • Occludes vein proximal to catheter • Removes tourniquet • Removes stylette • Obtaining labs? Connects heplock/IV tubing Secures catheter with tape/tegaderm Disposes needle/sharps in proper container Runs IV for brief period to assure patent line Adjusts flow rate as appropriate Patient assessment

Transtentorial (uncal) herniation signs

Ipsiplateral hemiparesis, ipsilateral mydriasis (CN III) and strabismus, contralateral hemianopsia (Posterior cerebral artery affected) , and altered mentation (reticular formation affected).

Duodenal hematoma

Isolated udoenal hematoma most commonlyu occurs in children following blunt abdominal trauma. It ist treated conservatibely with nasogastric and parenteral nutrition.

Atelectasis in surgery?

Its a common postoperative complication that results from shallow breathing and weak cough due to pain. It is most common on postoperative DAYS 2 AND 3 following abdominal or thoracoabdominal surgery. Pain and changes in lung complicance can cause ipaired cough and shallow breathing. Shallow inhalations limit recruitment of alveoli at lung bases, and weak cough predisposes to small-airway mucus plugging. This results in hypoxa--> RR increases---> low Pc02. *ABG usually shows hypoxiemia, hypocapnea, and respiratory alkalsosis. Large areas of atelectaiss may bause significant V/P mismatch, leading to hypoxemia and increased work of breathing. What happens is as compenation for the hypoxemia, patients usually hyperventilate and develop resp alkalosis and decreased Pc02. Adequate pain control, deep breathing excercises, directed coughing, early mobilization and INCENTIVE SPIROMETERY decrease the incidience of atelectasis.

In what patients are you worried about using succinycholine?

Its a depolarizing neuromuscular blocker that can cause life-threatening hyperkalemia. It should not be used in patients with or at high risk for hyperkalemia, such as burn and crush injury patients and patients with prolonged demyleination.

DDx of unlateral hip pain

Its broad and includes infection, trauma, arthritis, bursitis, and radiculopathy. Slipped capital femoral epihsysi Troachanteric bursitis is inflammation of the bursa surrounding the insertion of the gluteus medius onto the femur's greater trochanter. Excessive frictional forces secondary to overuse, trauma, joint crystals, or infection are responsible. Patients with this condition complain of hip pain when pressure is applied (as when sleeping) and with exernal rotation or resisted abduction. Hip osteoarthritis causes pain localized deep within the joint that may be referred to the linguinal area or rarely to the knee. Classicaly, internal rotation of the hip worsens this pain .

What is chylothorax?

Its drainage of lymphatic fluid into the pleural space secondary to disruption or blockage of the thoracic duct or one of its lymphatic tributaries. Malignancy is the most common cause, trauma is the second most common cause. Other causes are pulmonary tuberculosis, chronic mediastinal infections, sarcoidosis, lymphangioleiomyomatosis, and radiation fibrosis. Tx: -Intraoperative recognition of a thoracic duct injury is maganged by ligation of duct. Best approach is from a right thoracotomy. Operative ligation also done if the chyle drainage continues to be a lot like after weeks and above 5ooml/day. --Direct repair impractrical cuz of extreme friability of thoracic duct. -Injuries not recognized until several days after intrathoracic surgery frequently heal following low-fat diet (reverses flow of chyle) and either repeated thoracentesis or tube thoracostomy drainage.

Classifications of acute diverticulitis?

LLQ patin, leukkocytosis, fever. Uncomplicated (75%) -Tx: -stable patient: outpatient setting with bowel rest, oral antibiotics, and observation. -Hospitialization, treatment iwith IV antibiotics and obseration is recommended for patients who are elderly, are immunosuppressed, have high fever or significant luekocytosis and with significant comorbidities. - Complicated (25%). CT scan helps identifiy these. -refers to associated abscess, perforation, obstruction, or fistula formation. *fluid collection <3cm can be itreated with IV antibiotics. If >3cm, should to CT guided percutaneous drainage. If that doesn't work, surgery for drainage and debridement is the next step. *Fistulas can occur between the sigmoid colon and bladder, vagina, skin, or anothers segment of bowel. Tx: resection of sigmoid colon, escision of fistulous tract, and repair or resection of other involved organ. ---Diverticulitis is most common cause of GI tract fistulas *For just asymptomatic diverticuli in GI system (diverticulosis), only thing you recommend is increase fiber in diet.

Bile and fluids found in the small intestine have an electrolyte content similar to ? Saliva, gastric juice and right colon fluids have ?

Lactate ringers fluid High K and low Na content.

Classic X-ray of aortic injury?

Large left sided hemothorax and widened mediastinum. Also, deviation of mediastinum to the right occurs. If the rupture is complte, patient will probably die very quickly. But some patients have an incomplete or contained rupture.

What do you see with chronic rejection of liver and what do you do about it?

Late complication that takes months to years after transplantation and is characterized by a PAUCITY of bile ducts on biopsy due to immune-mediated injry to the biliary epithelium- the "vanishing bile duct syndrome". Tx options arent good and the best thing to do is retransplantation.

LCIS management?

Lobular carinoma iin situ (LCIS) is considered to be a risk factor for invasive breast carcinoma, NOT A PRECURSOR. Patients are encouraged to perform montly self-breast examinations and commit to yearly screening mammograms. Chemotherapy, radiation, and surgery are treatments reserved for DCIS and invasive carcinomas of the breast.

Most common comlications of cardiac catheterization?

Local vascular complications at the catheter insertion site.. -bleeding -hematoma (can extend to retroperitoneal space) -arterial dissection -acute thrombosis -pseudoaneurysm -AV fistula formation Retroperitoneal hematoma can present with sudden HEMODYNAMIC INSTABILITY and ipsilateral FLANK OR BACK pain. THe diagnosis is made with NON-CONTRAST CT of abdomen and pelvis or abdominal ultrasound. Tx is usually supportive w/intesive monitoring, bed rest, and IV fluids or blood transfusion. *since local hematoms formation is not uncommon, patients shoulds be advised to avoid strenous activity or lifting heavy objects for one week post catherization. Radial artery approach leads to fewer local vascular complications, which has led to an increasing number of cardiac catheterizations being performed via this approach.

INguinal hernias where?

Located above the inguinal ligament

MEN syndromes

MEN 1: para-pan-pit --pancreatic endocrine tumors can be: Zollinger Ellison, insulinoma,s VIP omas MEN 2: MPH -meduallry tryoid carcinoma (secretes calcitonin) -pheo -hyperplasia of parathryoid MEN 2B: MMMP -medullary thryoid, pheo, mucosal neuromas (GI, oral), marfaonid habitus *genetic testing (RET proto-oncogen germline mutaiton) is more sensitive than biochemicl measurement (serum calcitonin) and is the recommended screening test for suspected MEN 2 syndrome

Pheochromoctyoma associated with what 4 conditions?

MEN2A, MEN2B, Von Hippel-Lindau disease, and NF-1.

Small bowel obstruction?

MNEMONIC: ABCDE Absence of gases and/or bowel Bobgyrdofi (loud bowel sounds), this happens in early SBO cuz bowel is pushing against the obstruction, if bowel is dead then not gonna hear sounds period, Constipation, Distention, Emesis Bowel distal to the obstruction will not be distended. Additionally, hyperactive "tinkling" bowel sounds are typically present on PE. Peristaltic waves on the abdominal wall can also be ovserved *besides persitent pain, you can see fever, tachycardia, leukocytosis, elevated serum amylase. and evidence on x-ray. *mechanical obstruction of the bowel produces accumulation of fluid in the bowel lumen and bowel wall, and also extravasation of fluid into peritoneal cavity. The net result of these fluid shifts is a depletion of intravascular volume and decreased perfusion of all organs. SO one of the most VITAL aspect of treatemtn is early recognition and restoration of intravascular volume to reestablish organ perfusion. Should differentiate SBO between -mechanical vs functional -partial vs. complete -simple vs. strangulation *Generally, patients with proximal SBO have more frequent vomiting and those with more distal obstruction have more distension and less vomiting. Simple means just occlusion of lumen whereas strangulated refers to loss of blood supply to bowel wall. These people have peritoneal signs and signs of shock: fever, tachycardia, and leukocytosis. ADHESIONS are by far the most common cause of SBO. Typicallyr esults from abdominal operations or inflammatory processes. Other causes to know is hernia, ileus, Crohn's, gallsonte ileus, or tumor, volvuulus, inussectipon. Remember that the presence of boewl movement does NOT r/o bowel obstruction because stuff distal to the obstruction can be evacuated. *as opposed to large bowel obstruciton, SBO is rarely caused by neoplasm. If neoplasm is the cause, its most likely extrinsic compression as opposed to intraluminal compression LABS: CBC, electrolytes, amylase, UA. Leukocytosis can happen and if its prrsistent even after hydration then that should raise suspicion of complications. Algorithm for diagnosis: main idea is that you order a CT scan if your suspicious for large bowel obstruction, or if you think small bowel obstruction and have ruled out incarcerated hernia and h/o abdominal recent surgery. CT alos helps you know if its ileus or not. -you can also see dilated loops on x-ray TX: -Uncomplicated partial small bowel obstruction from adhesions: initially treated with nonoperative therapy of NPO, NG tube placement, monitor fluids, and f/u. If no improvment, get a CT scan or UGI/SBFT to confrim diagnosies or see other possibilites. Surgery can be done, if non surgical stuff not working. -High grade bowel obstriction or strangulate bowel: prompt resuscitation and early operative therapy.

B

MU

Predictors of having a cardiac event during noncardiac surgery?

Major, intermediate, or mild predictors. MAJOR: unstable angina, recent MI, decompensated CHF, significant arrhythmias, severe valvular disease INTERMEDIATE: mild angina, prior MI, compensated or prior CHF, diabetes, renal insufficiency MINOR: advanced age, abnormal ECG, irregular rhythm, prior stroke, hypertension,

Squamos cell vs basal cell carcinoma on face?

Majority of lip malignancies occur on lower lip. Most of time its squamos cell. Basal cell occur mucl less frequently and affect upper lipe more than lower lip. If basal cell on face then Mohs procedure is best. *basal cell is most common type of skin cancer. rarely metastasize. surgical excision is tx (mohs if one face). if lesion is really small then can do stuff like curettage, electrodesication or laser.

Upper GI bleeds causes

Mallory's Vices Gave An Ulcer mallory weis tear varices gastritis/esophagitis av malformation ulcer: peptic/duodenal Signs: hematemesis (bright red or coffee grounds, hypotension, tachycard, black tarry stool (melena), Do a nasogastric tube to determine whether bleeding is active. Irrigate with water or saline until gastric aspirates are clear. Then do upper GI endoscopy establishes Dx in most cases. Surgery indicated if bleeding not stopping.

Local anesthesia lidocaine dosing?

Maximal safe does of lidocaine to a 70kg person is 4.5 mg/kg or about 30mL of a 1% solution. *The addition of epinephrine to lidocaine, procaine, or bupivacaie not only doubles the duration of infiltration anesthesia, but increases the maximal safe total dose by one-third (7 mg/kg for lidocaine with epinephrine) by decreasing the rate of absorption of drug into the bloodstream. However, epinephrine-containing solutions should not be injected into tissues supplied by end arteries (fingers, toes, ears, nose, penis).

Ventilator associated pneumonia

May occur in parients on assisted ventialation for <48 hours and is a results of impairments in the lungs' natural defenses. FEver and hypoxia are common, and Pseudomonas infection must be considered. Samples may be obtaine using bronchoscopy with bronchoalveolar lavage. What is diagnositc? Greater than or equal to 10,000 colony forming U/ml

Morton neuroma

Mechanically induced degenerative neuropathy commonly seen in runners that presents with pain between the third and fourth toes reproducible with palpation on physical examination. Treatment is conservative, with bilateral show inserts. Surgery is reserved for patients who fail sonservative treatment.

Treatment of C. dif colitis (pseudomembranous colitis)?

Metronidazole is FIRST LINE ORAL VANC is 2nd LINE

MELD score?

Model for End-stage LIver Disease enables liver allocation to be based on objective variables: total bilirubin, INR, and creatinine. Has high predictive capacity at identifiying end-stage liver disease pateitns with greatest risk of mortality within 3 months.

Diaphragmatic rupture?

More common on the left side because the right side tneds to be protected by the liver. Patients usully have respiratory distress and can have deviation of the mediastinal contents to the opposite side. Elevation of the hemidiaphragm on the chest x-ray might be the only abnormal finding. Chest x-ray showing a nasogastric tube in the pulmonary cavity is diagnostic. Common cause is blunt abdominal trauma

"apple-core" lesion treatment?

Most common cause is carcinoma. Presents as bowel obstruction. Patients should get prompt surgical treatment; colostomy.

Meckel's diverticulum?

Most common cause of small intestinal bleeding for people under age 30. Can contain ectopic gastric mucosa that causes small bowel ulcerations. 99Tc pertechnate scan is diagnostic of it.

Supracondylar fractures of humerus

Most common fractures in pediatric populastion. Majority of cases result from a FALL ON OUTSTRECHD HAND. the most common complication is ENTRAPMENT of the BRACHIAL ARTERY or MEDIAN NERVE. The radial and brachial pulses must be assessed before and after reduction as brachial artery can be impinted. Motor and sesnory funciton should also be assessed due to rusk of median nerve injury.

Wilims tumor presentation

Most common presentation is asymptomatic upper abdominal or flank mass in a child 1 to 4 years of age. Neurblastoma is the most common type of retroperitoneal mass in child older than 1 year

Graft Versus Host Diesease

Most comon organ involved is SKIN, skin rash is alsmost always seen. You also commonly see liver, lung, and intestine problems. I guess a notalbe absence is NO KIDNEY PROBLEMS.

Signs of adrenal failure, or Addison's disease?

Most have nausea, vomiting, abdominal pain, etc. Hyponatremia is most common, cuz of mineralcorticoid deficiency Hyperkalemia is also common due to dcreased activation of aldsoterone receptors. You get hyperkalemic, hyperchloremic, acidosis. Weight loss, hyperpigmentation, low blood pressure and vitilitgo as well

CLosed Head Injury

Most important management principle for a patient with severe head injury is the avoidance of secondary brain injury. Due to loss of cerebral autoregulation, injured brain is much more susceptible to hypoxia and hypotension. This is why early endotracheal intubation is done in many cases, so hypoxia doesn't happen. -if increasing ICP making you worried: intubate patient and appropriate resuscitate patient, then do hyperventilatino and mannitol. BUT, mannitol should not be given unless the patient is adequatley resuscitated because it can aggravate hypovolemia and cause uncompensated shock. GLASGOW COMA SCALE -Can predict the severity and prognosis of coma and other condictions, during the primary survery. It assesses the patient's ability to open his/her eyes, motor response, and verbal response. -Its not used to diagnose coma in a patient. -Severe head injury: GSC of 8 or less -Mild head injury: GSC of 9 to 12 Burr Hole: a hole drilled through the skull, usually on the side of the larger pupil to decompress an intracranial mass lesion. *Interventions in lowering ICP Head elevation: increased venous outflow from the brain Sedation: Decreased metabolic demand and control of hypertension IV mannitol: Extraction of free water from brain tissue--> osmotic diuresis Hyperventilation: C02 washout--> cerebral vasoconstriction Removal of CSF: Reduction of CSF volume/pressure *Transtentorial (uncal) herniation Ipsiplateral hemiparesis, ipsilateral mydriasis (CN III) and strabismus, contralateral hemianopsia (Posterior cerebral artery affected) , and altered mentation (reticular formation affected)..

Popping sensation in knee?

Most meniscal tears in the knee joint occur during a distincitly recalled acute knee injury, often associated with a popping sensation. Subsequent joint swelling develops gradually, and is often not noticed until the following day. This is in contrast to ligamentous tears, which may also be associated with a popping sensation, but which cause rapid joint swelling due to hemarthrosis. LIke an ACL tear. *McMurry test: painful click...with passive extension from full flexion while applying tibial torsion....of the knee with the examiner's thumb and index finger placed on the medial and lateral joint lines. *Apley test: pain with pressing the heel toward the floor while internally and externally rotating the foot with knee flexed to 90 degrees. *Locking of the knee joint on extension is generally seen in "bucket handle" tears, while ROM at knee is limited by pain in all meniscal tears.

External fixation

Most often used in setting of high-energy trauma with significant soft tissue injury, vascular injuries needing repair, and in polytrauma patients as a "damage control" procedure.

Burns (everthing except types of burns)

Most patient swith burns less than 15% TBSA can be resuscitated with oral fluids. For larger burns, isotonic IV fluids like LR should be used. Fluid needs are estimated by PARKLAND formula. ---24 hour volume of IV fluids needed is 4 ml x ?kg x ?% burn. Half ot this amount is given in the first 8 hours and the remainder in the next 16 hours. *measuring UOP is helpful way of assessing adequacy of resuscitation. Excess UOP should also be avoided unless one is treating myoglobinuria (which is commonly associated with deep burns and electrical injuries). Care not to fluid overload during resucscitative period. *Calculating burn area: "Rule of nines" is useful. limbs are 9%, front of torso is 18%, back of torso is 18% TX: attention must be given for prevention of hypothermia and infection. So STEROIDS should NOT be used for any burn greater than 10% TBSA. Prophylactic IV antibiotics are not usually used cuz they can select for resistant organisms. So creams w/local broad antimicrobial activity are used (some examples: silver sulfadiazine, silver nitrate, and mafenide acetate ) *main complicatin of silber nitrate are electorlye abnormalities and metheglobinemia * mian comlication of silver sulfadiazine is neutrobpenia Burn complications: basically every ogran affected some way.

What does CT scan show with diffuse axonal injury?

Most signficnat cause of morbidity in patients with traumatic brain injury. commonly due to traumatic decerlation injury and results in vegetative state. CT scan shows numerous punctate hemorrhages with blurring of grey-white interface.

Usefulness of CEA level?

Most useful as a marker for postoperative recurrence in colorectal cancer. A level obtained every 3 months during the first 2 years after surgery. CEA is nonspecific tumor marker that is elevated in only about one-half of patients with colorectal tumors and is often elevated in patients with lung, pancreatic, gastric, or gynecologic malignancies.

A good way to increase FRC after surgery?

Moving the patient form supine to sitting can increase the FRC by 20 to 35%. Increasing the FRC can help pvenet post-op atelectasis. Respiratory complications are a common cause of post-op morbidity and mortality. After upper adbominal surgery, Vital Capacity can fall 50% and FRC can fall 30%. First, post-op pain promotes shallow rapid breathing. Narcotic analgesics futher decrease respiratory drive, deep inspirations and coughint. Some anesthetic agents depress mucocilliary clearance and may promote bronchiolar obstruction as well. Additionally, obese patients will experiience a PIckwikian-like syndrome when kept chronically supine following surgery. These factors together promote alvoalar atelectasis, a major cause of FRC reduction.

Inflammatory breast cancer tx

Multimodality therapy with neoadjuvant chemo, surger, and radiation. The peua d'orange results from invations of dermal lymphatics with resulting edema. Clinical picture can resemble that of mastitis and care mus be taken to differentiate between the 2 pathologies.

You have lower GI bleed, what is algorithm you follow

NG tube aspiration. If no blood, positive bile on NG tube aspirae... -then do colonoscopy to find source. If you can't find the source and the patient stops bleeding then do small bowel studies (capsule endoscopy). But if you can't' find source and th patient does not stop bleeding do a Technetium-99 labeled ertyrhoctye scintigraphy. It localized source of blood loss so that region can be further evaulated by coloscopy or angiography. If blood found, and plus/minus bile on NG tube aspirate.. -then do upper endoscopy.

What properties of nitric oxide make is less desirable to use as an anesthetic gas in cases of intestinal obsturction?

NO may cause progressive distension of air-filled spaces during nanesethsia . If diffuses into these gas-filled compartments faster than nitrogen can diffuse out, so its use can lead to worsened distention.

Indicators for extubation attempt when person is intubated?

No single parameter is 100% predictive. Rapid shallow breathing index is ratio of respiratory rate to tidal volume. There is evidene to suggest that an index between 60 to 105 predicts successful extubation. Negative inspiratory force should be at least greater than -20 cm H20. Patient should be weaned to 5cm H20 PEEP before attempting extubation. The minute ventilation, which is the product of the tidal volume and respiratory rate, should be less than 10L/min. The spontaneous RR should also be below 20 breaths/minute.

Renal cancer no responsive to what?

Not responsive to radiation and chemo. Radical nephrectomy remaines the main treatment for localized renal cancer.

Guy has leukoplakia (white patch in oral cavity) and is worried about cancer?

ONly about 5% of patients with leukoplakia develop cancer. First have patient do a strict oral hygiene and avoidance of source of irritants like alcohol and tobacco. Biopsy is reserved only for those with thick lesions or those in whom the lesion does not resolve after avoidance therapy.

Prostate cancer

Observation, or active surveillance, is a perfectly acceptable managment option in patient with early cancer cuz may early prostate cancers are slow-growing and will remained conficed to prostate gland for a long time. Active surveilleance is visits with doctor every 3-6 months with DRE looking for any changein prostate gland. In addiction, PSA is looked at and imaging studies can be done to detect spread.

Person needs FFP transfusion for surgery after having abnormal coagulation studies, when is optimal timing of the transfusion?

On call to surgery

What kind of intubation do you geive people with cervice spine injuries after you stabilize the spine?

Orotracheal intubation with rapid-sequence intubation is preferred for establishing an aiwray in an apneic patient with a cervica spine injury. Nasotracheal intubation is a blind procedure that is contraindicated in apneic/hypopneic patietns. Also contraindicated if the patient has a basilar skull fracture cuz tube could go in intracranially if cribirfrom plate disrupted.

Lab values on osteoporosis? osteopetrosis? Paget disease? Osteomalacia/Rickets? Hypervitaminosis D?

Osteoporosis: Calcium and PTH are normal. everything normal. Low bone mass but adequate mineralization. Osteopetrosis: -bad osteoclasts don't do their job-->dense brittle bones. - Mostly normal lab values, in severe disease decreased calcium. Paget disease: -bad bone remodeling by osteoblasts and osteoclasts--> bones are weak and fracture. abnormal "mosai" patter of bones. -commonly get cranial enlargement (increased hat size) and hearing loss -Lab: only increased ALP. Osteomalacia/Rickets -Vit D deficiency causes increased pTH secretion which excretes a lot of phosphate. You get soft bones. -Lab: decreased Calcium, decreased Phosphate, Increased ALP, Increased PTH. -Bone pain and muscle weakness. -x0ray findings unclude PSEUDOFRACTURES (looser zones). Hypervitaminosis D -caused by oversupplmentation or granulmonatous disese (sarcoidosis) Lab-increased vit D leads to increased Ca and Phosph. PTH is decreased -Symptoms are similar to that of hypercalcemia: constipation, polyuria, polydipsia, abdominal pain

Trauma: Abdominal Trauma

PAI (penetrating abdominal injuries) resulting from a gunshot create damage via 3 mechanims 1. direct injury by bullet 2. fragmentation of bullet causing injury 3. indirect injury from resultant "shock wave" Management for penetrating abdominal trauma: -Asymptomatic patient: observation for 24 hours -Local wound exploration: if you see that abdominal fascia has been penetrated, further evaluation is needed -DPL (diagnositc peritoneal lavage -If patient hemodynammicall unstable or has rigidity, gaurding, or significant tenderness then thats indication for lapartomy Then there is BLUNT abdominal trauma. BAT. Here is the uworld alogirthm for that. **IF BAT and UNSTABLE..do FAST examination. FAST+ (for blood)--->laparotmy FAST inconsulive---> do DPL -----DPL+-->laparotmy -----DPL(-) --> look for signs of EXTRA abdominal hemorrhage(pelvic fracture, etc). FAST negative---> look for signs of EXTRA abdominal hemorrhage +signs of extra abdominal hemorrhage--> stabilize (angiography, splint) (-)signs of extra abdominal hemorrhage---> stabilize, then CT abdomen **IF BAT and STABLE-->do CT scan. *motor vehicles accidents acount for most BATs, and the most freuently injured organs are the LIVER and SPLEEN. Free intraperitoneal fluid should raise suspicion for a lever or splenic LACERATION. -if the person hemodynamicaly stable then next step is to get a CT scan. If no stable and unresponsive to fluid administration, emergent exploratry lapartomy is required. *can acuse pancreatic contusion, crush injury, laceration or transection. Pancreatic injureis may be MISSED BY CT scan during first 6 hours of trauma. An untreated pancreatic injury can later be complicated by a retroperitoneal abscess or pseudocyst. ----Tx is percutaneous catheter drainage with antibiotics if symptomatic or enlarging. BUT if not, therapy should be not considered for at least 6 weeks cuz most of these spontaneously resolve . *the part of bowel that is especially susceptible to BAT is duodenum, which is almost entirely retroperitoneal. This segment of the small bowel is vulneralbe to crush injury where it overlies the vertebral bodies. Such injuries may cause duodenal hematoma and obstruction. Physical exam -Seat-belt sign: ecchymotic area found which can be associated with perofratin of bladder or bowel as well as lumbar distraction fracture -Cullen's sign: (perimbulical ecchymosis) is indicative of intrapertoneal hemorrhage -Grey-Turner's sing (flank ecchymosis) is indicative of retroperitoneal hemorrhage -Kehr's sign: left shoulder or neck pain secondary to spleen rupture. The pain increases when patient is in Trendelenburg position or with LUQ palpation (diaphragmatic irritation). *anything that causes peritonitis and irritates the diaphragm can cause this sign I think. In the setting of blunt abdominal trauma, hemoperitnoeum or spillage of bowel contents, bile, pancreatic secretions or urine can cause acute chemical peritonitis. With urine, the only region of bladder covered by peritoneum is the dome of the bladder so its the only part of bladder where leakage would cause peritonitis. With trauma, the bladder can rupture there.

Esophageal motor disorders

PRIMARY achalasia, DES, nutcracker esophagus, hypertensive LES SECONDARY sceroderma, neurmuscular disease, endocrine and metastatic disorders

Post-transplant malignancies?

PTLD: postransplant lymphoproliferative disorder is associated with EBV. Wide array of treatments. Hepatocellular cancionma: from Hep B and Hep C Kaposi sarcom: HHV-8 Cervical cacner: HPV *CMV can occur too when CMV seronegative recipient recieves an organ from a CMV seropositive donor but CMV does not predispose to malignancy though.

Algorhtim for palable breast mass. Breast cancer info

Palpable breast mass If under 30 years.. -do ultrasound. -----if ultrasound shows simple cyst--->do fine needle aspiration *remember that FNA can show cancer cells but not gona be useful for staging. For that you need to do an actual core biopsy. -----if ultrasound shows complex cyst/mass----> image guided core biopsy If over 30 years -do mammogram and ultrasound --if suspicious for malignancy---> do a core biopsy *prognostic indicators TNP staging-MOST important then..in order of decresing significance... -ER+ and PR + are good prognositc features. Triple receptive negative breast cancer means that the cancers are negative for receptors for estrogen, progesterone, and HEr2/neu...bad prognosis, being increasingly identifieid in premenopasual black and hispanice women. Triple negative treated with chemo. -Her-2/neu oncogen overexpression is related to worse prognosis -Histological grade of tumor; poorly differentiated is worse outcomes Management of breast cancer 1. Obtain tissue diagnosis and staging 2. Locoregional therapy -BCT and mastectomy offer equal survival benefits. BCT is breast conservation therapy : lumpectomy and radiation with axillary staining by sentinel lymph node biopsy. The rational for sentinel node sampling is to identify tumor involvement in the primary lymphatic drainage area and perform biopsy on only these nodes. Sentinel lymph nodes are localized following radiotracers and blue dye injections at the site of the primary tumor. *Sentinel lymph node biopsy is used most frequently in patients with breast cancer and melanoma who do not have clinically positive lymph node disease or metastases. *modified radicat mastectomy: take out breast and levels 1 and 2 nodes. A 'radical mastectomy" isn't done anymore, that's when you take out pec muscles too. 3. SYSTEMIC therapy: given to patients who are at risk for or who have knonw metastastatic disease (stages 3 and 4). But majority of stage 2 are offered it as well cuz of risk of recurrence. The options here are surgery followed by CHEMO or chemo and then surgery (neoadjuvant chemo). -Most common chemo regmines in US are FAC (5-flurouracil/doxorubicin (adriamycin)/cyclophosphamide) and AC (FAC w/o the F). -HER+. Trastuzamab is HER2/neu receptor antagonist and useful in tumors that overexpress HER2/neu. So HER+. Antiestrogen therapy is given for 5 years to patients with estrgen and/or progesterone recptor postivie tumors. Cardio toxicity associated so you need an echo before you use. -ER +. Tamoxifen is an estrogen receptor antagonist and raloxifene is a selective estrogen receptor antagonist. Antiestrogen therapies are used as adjuvant therapy for patients with hormone receptor positive cancers and as chemoprevention in patients with high risk of breast cancer -ER + tumors in post menopausal woman: aromatose inhibitor is hormonal therapy of choice. *multifocal means multiple tumors within 1 quadrant. ppl with multifocal disease can still get breast conservation surgery. contraindications to breast conservation therapy include diffuse microcalcifications, persistent postive margins after multile reescisions, pregnany, multicentric disease ( multiple tumors in separate quadrants), previous history of therapeutic radiation *purpose of radiation is to prevent local recurrence.

Treatment of stones

Passage: -Many stones smaller than 1 cm can be managed with hydration and analgesics until the stone passes spontaenously. For stones that are unliley to pass (bigger than 1cm) -Extracoporeal shock wave lithotripsy (ESWL) is pretty good. with 85% success rate. -Percutaneous nephrolithotomy, which establisehs a tract from skin to collectying system is used when ESWL doesn't cut it.

Long term affects of colon resection?

Patients may undergo resection of a large fractino of the colon and suffer little long-term change in bowel habits because the reserve capacity of the colon for water absorption greatly exceeds the normal requirement for maintaining stable bowel function.

Person traveled to mexicao comes back with liver mass. what should you think?

Patients with a liver mass who have traveled in endemic area should be evaluated for amebic liver abscesses with serologic testing and receive empiric treatment with oral metronidazole. Because of its associated risks, cyst aspiration is not typically recommended unless patient fails therapy. Pyogenic liver abscess shoudl be considered in DDx but its usually secondary o surgery, GI infection, or appendicits. Also presentation mroe severe than with amebic liver abscess. These you initially treat with perutaneous catheter drainage and antibiotics against gram-negative and anaerobic organisms.

Penile fracture?

Penis is far more vulnerable to trauma in erect state than it is in the flaccid state. This injury most often occurs during sex when the women is on top of the man cuz the penis may emerge from the vagina in this position and be subsequently injured in a bending fashion between its rigid fixation on the male and the downwardly moving female perineum. The injury and associated snapping sensation results from tearing of the tunica albuginea, which invests the corpus cavernosum. A hematoma rapidly forms at the site of injury causing bending of the shaft of the penis at the site of fracture. Treatment is with an emergent urethrogram to assess for urterhal injury as well as emergent surgery to evacuate the hematoma and mend the torn tunica albuginea.

Pvd vs Acute arterial occlusion?

Peripheral vascular disease -intermitten claudiction, rest pain (OFTEN prominent at night, awakens person from sleep), decreased pulses, ischemic ulcers, -Dx: arteriogram -Tx: claudication-conservative treatment. If you have rest pain then that resprents advances stages and warrants arterial reconstructive surgery. -Conservative treatment: So you start on LOW DOSE ASPIRIN and STATINs. Then endroll patient in SUPERVISED EXERCISE PROGRAM. The goal is to reproduce claudication symptoms each session. This has been shown to reduce symtpoms and improve max walking ditance. If this fail then go to cilostazol and percutaneous or surgical revasculariation. -Surgery: Angioplasty first. When that not an optino then aortofemoral graft in normal people. High risk people (bilateral iliac disease, problems with aorta)should get axillofemoral bypass. Acute arterial occlusion -6 Ps: pallor, pain, pulselessness, paresthesias, paralysis, polar (cold) -Dx: arteriogram -Tx: anticoagulation, emergent surgery

What is abdominal succussion splash

Physical exam can show an abdominal succussion splash, which is elicited by placing the stethoscope over the upper abdomen and rocking the patient back and forth at the hips. Retained gastric material >3hours after a meal with generate a splash sound and indicates the presence of hollow viscus filled with both fluid and gas. Gastric outlet obstruction will show this for example.

guy has a finger cut off, how should the finger be brought to the hosiptal?

Place the amputated finger in saline moistened gauze in a plastic bag; place the bag on a bed of ice and bring it along with patient to the ER.

miscellanous esophageal disorders

Plummer Vinson syndrome: Webs (thin mucosal structures protruding into lumen) associated with iron deficiency anemia.. the web is usually below the cricopharyngeus muscle. Treatment consists of dilatation and iron therapy Schatzki's ring: A narrow lower esophageal ringlike outgrowth associated with dysphagia. often associated with hital hernia. Tx: diltation +/- antirefulx medications to incision of ring and excisiion.

What do you think if you find air in biliary tract?

Pneumobilia. When you have patient with small bowel obstruction with a history of gallstones you should consider this, galstone ileus. In this condition, large chronic gallstone mechanically erodes through the wall of the ballbladder into adjacent stomach or duodenum. A connection is formed between the biliary stystem and the GI tract which allows air into the biliary tract. When the gallstone arrives in the distal ileum, the caliber of the boewl no longer allows passage, and a small-bowel obstruction develops. Surgical removal of gallstone is necessary.

Peripheral artery aneurysms?

Popliteal anuerysm is the most common *Asymptomatic aneurysms less than 2cm usually left alone. Symptomatic or bigger than 2cm should be repiared. second most common is femoral artery aneurysm these two are frequently associated with abdominal aneurysms

Whiplash injury can cause what?

Post-traumatic syringomyelia. Symptoms can develop months to years later. Condidtion involves enlargement of the central canal of the spinal cord due to CSF retention, resulting in impaired strength and pain/temperature sensation in the upper extremeities. MRI is used to diagnosis.

Person has RUQ and just had cholecystectomy but pain is sitll there?

Postcholecystectomy sydnrome Spincter of Oddi dysfunciton, common bile duct stones, or functional etiology (diagnosies of exclusion) Sphincter of Oddi dysfunction is characterized by RUQ pain, elveation of liber enzyems during pain attack which resolve when attacks subside. -elevated biliary spincter pressure highly specific for it -ERcp with a spincterotomy is the treatment of choice

What is refeeding syndrome?

Potentially fatal complication of nutritional rehabilitation in malnourished patients. Happens as a result from surge in insulin activity as body resumes anabolism. During periods of starvation, electorlyes are shifted to extracellular space to maintain adequate serum concentrations.Carb intake stimulates insulin acivity, which in turn promotes cellular uptake of phosphorus, potassium, and magnesium. Result is potential hypokalemia, hypomagnesemia, and hypophosphatemia. Clincial manifestations inculde arrhythmias and cardiopulmonary failure, seizures, and Wernickes encephalopathy. So aggressive initiation of nutrition without adequate electorly repletion can lead to problems. *This can happen with TPN therapy for malnourished patient too.

General surger pre-op and post-0p stuff

Pre-operation, assess... -renal: BUN and creatinine. increased means a loss of renal reserve. you wanna avoid hypotension during and after surgery. -pulmonary -hematologic: CBC, blood types. ---anemia: determine cause, patients whi will not tolerate anemia well include chronic hypoxia, ischemic heart disease or cerebral ischemia --thrombocytopenia --coagulopathy: check PT and PTT. expect to be elevated in liver problem patients. Antibioic prophylaxis: **appropriate dosing and timing of antibiotic prophylaxis is a single dose, no greater than 1 hour prior to incision. In general: Cefazolin 2. Colorectal surgery, appendectomy: Cefoxitin or cefotetan 3. Urological procedures: Ciprofloxacin 4. Head and neck: Cefazolin or clindamycin and gentamicin Common complications: Ileus: in general, await return of bowel function before advancing diet. --return of bowel function: small intestine, then stomach, then colon --to estimate when bowel function will return, allow one Post-op day per decade for major abdominal surgery *postoperative ileus follows most surgeries where the peritoneal cavity is entered. Morphine and other opiates compund this problem by decreasing GI motility. *signs and syntpoms of postoperative ileus include nausea, vomiting, abdominal distention, failure to pass flatus or stool and hypoactive or absent bowel sounds on physical exam (in contrast, mechanical bowel obstruction causes hyperactive "tinkling" bowel sounds C. dif colitis DVT/PE Wount infection Instruction to patient: -NPO: to decrease the risk of apsiration with intubation patietns whould refraine from solids 6-8 hours prior, and from liquids 2-3 hours prior to surgery -Bowel Prep: purpose is to clear bowel of stool, therby reductin bacterial count and risk of contamination ----Types: mechanical prep and oral antibiotics Medications: -Aspirin: avoid for 10 days post-op to allow platelets to regenerate -Hold clopidogrel for 7 days -Warfaran: 3 options -------1. Avoid 3 days prior to operation and resume postoperative day 2 -------2. admit preoperativly and chnage to heparin, which can be held only a couple of hours ahead -------3. operate through warfarin -Antihypertensives: Continue , especially Betal blockers; hold diuretics the morning of surgery -Antithyroid: Hold on morning of surgery -Thyroid replacement: give on morning of surgery -Oral hypoglycemics: avoid on day of surgery -Insulin: give half usual dose on morning of surgery

DCIS managemnt

Precursor of invasive ductal carcinoma. Tx of DCIS is wide excision or wide excision +radiation -in case of multicentric DCIS simple mastectomy is recommended. the progression according to Ed: ADH (atypical ductal hyperplasia)---->DCIS----> invassive ductal carcinoma ADH Tx: just lumpectomy + radiation. DCIS: lumpectomy + radiation. after treatment, data says if they go on tamoxifen it's good. -what are reasons to do mastectomy in DCIS? large tumor compared to small breast, if palpable disease, can't get clar margins on lumpectomy, multicentric disease, inability to tolerate radiation (like Li Fraumeni patients who if they were to get radiation would prob cause other tumors) Invasive ductal: lumpectomy + radiation + sentinel node biopsy *you never have lumpectomy w/o radiation.

Nipple Discharge DDx

Pregnancy infection/mastitis/or abscess --purulent discharge;; nipple is erythematous and tender galactorrhea secondary to pituitatry adenoma or medications or hypothyroid, --bilateral milky white discharge fibrocystic changes, -discharge can be yellow, brown, or green intraductal papilloma (benign epithelial lesion but can contain atypical hyperplasia or ductal cercinoma in situe) -usually unilateral serous or bloody discharge -this is 70% of pathologic bloody nipple discharge diffuse papillomatosis, -serous rather than bloody discharge, often involves multiple ducts more distant from nipple carcinoma -bloody or serous nipple discharge (or none), newly inverted inpple, abnormal skin changes, suspicious mass on examination Management: in many cases you can do a ductogram (radiologic test with contrast injected into the duct causing the discharge). If that is abnormal, then go to surgical biopsy.

Ischemic colitis symptoms and mangement?

Presents as hematochezia, fever, and abdominal pain. Unlike acute mesenteric ischemia, which affects the small intestine and erquires emergent intervention, ischemic colitis rarely requires surgical intervention unless full thickness necrosis, perforation, or refractory bleeding is present. Tx: expectant managament with IV fluids, bowel rest, and supportive care is treatment of choice.

Acute bacterial parotitis

Presents with painful swelling of the parotid gland that is aggravated by chewing. High fever and a tender, swoleen and erythematous parotid gland are common. This post-operative complication can be prevented with adequate fluid hydration and oral hygiene. The most common infectious agent is Staphylococcus aureus.

Causes of hypercalcemia

Primary hyperparathryoidism is most common cause in outpatient Malignany is most comon cause in inpatient -hypercalcemia of malignancy have cerum calcium levels gnerally much higher (>13) than those with primary hyperparathyroidism CHIMPANZEES Calcium supplementation Hyperparathrydoisism/Hyperthyroidism Immobility/Iatrogenic Metastasis Paget's disease Addison's/Acromegaly Neoplasm Zollinger-Ellison Excessive Vit A Excessive Vit D Sarcoidosis -----sarcoidosis causes hypercalcemia through increased production of 1-alpha-hydroxylase and can be treated with prednisone Clinical signs: CONSTIPATION (due to altered intestinal smooth muscle tone), anoreaxia, vomiting, weakness, POLYURIA, and CONFUSION First thing you wanna do with elevated corrected calcium levels that is confirmed with repeat testing is...measure pTH levels to know whether this is PTH independent or dependent. *with the exception of familiar hypocalciuric hypercalcemia, which may be associated with a mild increase in serum PTH levels, all other causes of hypercalcemia are associated with suppresesed PTH levels. *Hyperparathryoidism is also associated with certain well-described cardiovascular effects, including increased prevalence of hypertension, left ventricular hypertrophy, and calcifications of th emyocardium and mitral and aortic valves.

Types of wound closures (healing)

Primary intention -cloasures of the edges of the wound are brought together and sealed immediately with simple suturing, sking graft placement, or flap at end of surgical procedure. Secondary intention -closures involve leaving the wound open without active intent to searl the wound. The wound heals by re-epitheliaziation which resultins in contraction of the wound. This type of closure generally reserved for highly contaminated wounds in which a primary intetion repiar would fail. Tertiary intention (or delayed primary closure) -when a contaminted wound is initially treated with repeated therapy (antibiotic, debridements, negative pressure dressing) until the infection is controlled and then it is closed with surgical interventions such as suturing, skin graft placement, or flap.

Lymphedema

Primary: classified as primary when etiology is unkown. Hypoplasia of lymphatic system of lower extremity accounts for more than 90% of cases of primary lymphedema. If edema is present at birth, it is referred to as congenital: if it starts early in life, it is called praecox. If it appears after age 35 it is tarda.

Ulcerative colitis

Process is limited to mucosa, unlike Crohns which is transumural. Begins in rectums and goes backwards. Extraintestinal manifestations: anklylsoing spondylitis, uveiis, scleroderma, sclerosing cholangitis, arthritis, dermatomyosisit, hypercoagulable states. Medical management: anti-inflammaotry with antibiotics. When those fail, go to steroids. Surgery: Main indications for surgery in UC are fulminant colitis, or toxic megaclon, dysplasia, cancer, or intractable disease. -You do total proctocolectomy with end ileostomy (typically reserved for older or incontinent patients) or total proctocolectomy with ileoanal puch anastomasis. -If you're doing surgery for toxic megacolon, then you do subtotal colectomy with end ilesotomy. For patients with UC and toxic megacolon, complection of proctectomy can be perfored at later date. -Flulminant colitis: condiction characterizes by abdoinal pain, fever, sepsis that most commonly develops in setting of UC but sometimes Crohns too. -Toxic megacolon: when above findings are associated with x-ray evdience of colonic distention (>6cm). For either of these conditions, give fluids, antibiotics. will need colectomy if medical therapy fails. *Because of colorectal cancer risk increase with chronic UC, patients with duration of disease more than 7 to 9 years should undergo annual or biannual colonoscopies with biopsy.

Soft tissue sarcoma approach?

Proper biopsy is improtant cuz if improper one done it can complicate the care. Incisional biopsy should be done for most masses. Excisional biopsy should be reserved for small masses for which complete escision would not jeapordize subsequent treatment should a sarcoma found.

Explain difference between prothrombin time? Partial prothrombin time? thrombin time? bleeding time?

Prothrombin: measures speed of coagulation in the extrinsice pathway. Test will detect defiicencies in factors 2, 5, 7, 10 and fibrinogen and is used to moitor patients receiveing Coumadin derivatives. However, even sall amounts of heparin will artificially prolong the clotting time, so that accurate prothrombin times can be obtained only when the patient has not received heparin for at least 5 hours. Partial thromboplastin time: tests for defects in the contact and intrinsic phases of coagulation (2,5, 8, 9, 11, 12, fibrinogen) and is used to motiro the status of patients on heparin Bleeding time: assess the interaction of plateleys and fthe formation of the platelet plug. Thrombin time: assess qualitative abnormalities in fibrinogen and the presence of inhibotrs to fibrin polymerization. A standard amount of fibrin is added to a fixed volume of plasma and clotting time is measured.

Swanz-Ganz catheter

Pulmonary artery occlusion pressure (PAOP or PCWP). (normal 6-12) This reflects the pressures of left ventricle (end-diastolic). Cardiac output: normal 4-6 mmHg. Remember, CO=stroke volume x heart rate. The Swan Ganz allows CO to be measured via the thermodilutional technique: the temperature change is measured at the distal end of the catheter when cold fluid is injected from the proximal port. The difference in temperature reflects the CO, which can be though of as pump function. How is wedge pressure obtained? The "wedge" pressure is obtained by inflating the distal end of the Swan-Ganz and "floating" this "balloon" down the pulmonary artery until it "wedges." The pressure measured equals the pulmonary capillary pressure, and this theoretically, represents the left atrial pressure and ultimately the LVEDP. *what is the significance of wedge pressure? It reflects the left ventricular pressure, which will be increased with left ventricular failure.

What is crossmatch looking for and how do you do it?

Purpose of a cross-match is to determine whether the recipient has circulating antiboides against donor HLA antigens. These antibodes would not occur naturally but are the result of prior sensitazation during pregancy, blood transfusions or previos tranplantation. What you do is add recipient serum and complement to dontor cells (T, B celss, monoctyes). If antibodies are present you're gonna see cell lysis. This is detected by a dye, which is taken up by damaged cell membrane. *Blood type matching prior to organ allograft is similar to cross-mathcing prior to transfusion; O is the inversal donr and AB the universal recipient.

Hemobilia?

Quincke triad of abdominal pain in right upper quadrant, jaundice, and GI bleeding is present in 40% of patients with hemobilia. With more frequent use of percutaneous liver procdures, iatrogenic injury has replaced other trauma s the most common cause of bloody bile. 1st Line therapy for major hemobilia is transarterial embolization (TAE). Most cases of minor hemobilia can be managed conservativiely with monitoring, biliary drainage if needed. Surgical treatment is reserved for cases in which conservative therapy and TAE have failed.

What artery does midshaft fracture of humerus normally affect?

Radial nerve. Leads to wrist drop due to marked limitation of extension at wrist joing.

What is the way to obatin a histological evaluation of testicular mass?

Radical inguinal orchiectomy with high ligation of spermatic cord near the internal inguinal ring is procedure of choice. Violation of scrotum must be avoided because may alter lymphatic drainage of testis.

Blood pressure side effect of morhpine?

Rapid IV injection may cause hypotension.

Hesselbach's triangle

Rectus muscle Inferior epigastric vessels Inguinal ligament

Stress ulcers?

Refers to acute gastric or duodenal erosive lesions that occur following shock, sepsis, major surgery, trauma or burns. Are superficial and multiple sites. Unlike chronic benign gastric ulcers, which are generally lesser curvature and antrum, acute erosive lesions usually invovle the body and fundus and spare the antrum. These people do not have increased gastric secrtion but rather have decreased splanchnic blood flow. So ischemic damage to the mucosa may play a role.

"carcinoma in situ" means what?

Refers to the presence of malignant cells in the mucosal layer only.

RQ

Respiratory quotient. The RQ is the steady state ration of C02 produced to oxygen consumed per unit time and may be used to make assessments of the metabolism taking place in particular organs or in the body as a whole. An RQ close to 1.0 indicated that carbohydrate is the major nutrient being oxidized and you have net lipogenesis. Metabolism of proteins alone yields an RQ of approsximately 0.8 and oxidation of fatty acids alone yields an RQ of about 0.7. Assessment of RQ is important when attemtping to wean patiens from mechanical ventilation, as overfeeding, especially with carbs, can cause excessive C02 production and make weaning more challenging. *sepsis is a hypermetaoblic, heypercatabolic state wherin both body and fat and protein are borken down in addition to glucose being oxidized. For this reasong, the RQ in a septic patient is typically less than 1.0. *The respiratory quotient is the ratio of rate of C02 production over the rate of oxygen uptake. RQ of .75 to .85 is ideal. The RQ is dependent on the composition of nutritional support. The RQ is dependent on the composition of nutritional support. An RQ of .7 indicates a diet comprised largely of lipids. AN RQ of 1 or greater indiates a diet comproisiing primarily of carbohydrates, which can impair weaning from the ventilator.

Diabetic foot ulcers

Result from 3 things: neuropathy, microvascular insufficiency, and immunosuppresion. They classicaly occur on the plantar surface of the foot under points of greatest pressure, such as under the head of the first metatarsal bone.

What is porcelain gallbladder?

Results from chronic cholecysitis and its the intramural deposition of calcium salts that happens. Its usually diagnosed on abdominal imaging Associated with increased risk of gallbladder cancer so for this it requires surgical resection.

Short bowel syndrome

Results from either extensive bowel resection or a funcitonal defect such as radiation enteritis, or severe inflammatory bowel disease. Characterized by diarrhea, dehydration, electrolyte distrubance, malabsorpotion, and progressive malnutrition. NOrmal small bowel length is 300 to 500 cm. An individual with 90 to 180 cm of small bowel (about 1/3 of normal) may develop transient diarrhea and malabsorption, whereas individuals with less than 60 cm of small bowel may require permanent parenteral nutrition. Most common causes of SBS are Crohns disease and mesenteric infarction in adults, In kids its nectrotizing enterocolitis and midgut volvulus. Treatment: 3 stages of treatment... Acute phase: stuff like parenteral nutrition, assessing residual bowel funciton Early adaptiation phase: increase enteral nutritional support as tolearated, glutamin supplemenation, etc. Long-term adaptation phase: surgery like bowel lengthening procedures, small bowel transplant *Nutritiona support is the cornerstone of SBS. Provides nutrition and promotes gut adaptation, so that the patient may eventually survive an oral diet. So you can to TPN initially and slowly reintroduce enteral feeding. -Enteral feeding: via mouth, nasogastric or nasojejunal tube. Advantage of enteral are that its more physiologic and more economical and it promotes intestinal mucosal hyperplasia and gut adaptation. The disadvanatge is that it requires enough healthy intestines to absorb sufficient nutrients. -TPN: Advantages is that it doesn't rely on length of bowel. Disadvatages include inttestinal atrophy, IV line problems, high cost, liver dysfunction,.

APproach to GI bleeding

Resusciation -IV access and tranfuse fluids and blood if needed -Evaluate for hypotension, tachycardia, orthostatis -Type and crossmatch, CBC, coag studies Determine source of bleeding-upper GI vs Lower GI -H&P, Rectal exam, Stool exam for melana/guaiac, Nasogasstric lavage looking for bright red blood or coffe grounds (this procedure r/o UGI bleed)

What test is predictive of perioperative ischemic cardiac events amonth patinets undergoing noncardiac surgery?

Reversible perfusion defects on dipyridamole-thallium imaging.

Surgical treatment of morbid obesity

Route-en-Y small pouch gastric bypass: proximal pouch to Roux limb of jejunum. Preference for this compared to vertical banded gastroplasy. -Complications: Most common and most serious complication: leakage from attachment of stomach to intestine can be devastating. Characterized by fever, leukocytosis, and left shoulder pain on Post-op days 3 to 5. Also B12 deficiency, iron deficiency, osteoporosis and ulcers. Patients need supplemental vitamins, calcium, oral iron, and Vit B12 after procedure. Vertical banded gastroplasty: proximal lesser curve pouch to main stomach with circumferential band at communication -Complications: high reoperation rate for stoma erosoin, frequent GERD Adjustable lap bad: Band around upper stomach with circumferential balloon accessibele by subcutaneous port -Complications: Band slippage can happen, resulting in reoperation. Duodenal switch: Division of duodenum at pylorus. *Any patient who has lost much of the ileum (whether from injury, disese, or elective surgery) is at high risk of developing enteric hyperoxaluria if the colon remains intact. Patient selection: BMI of 35 to 40 w/comorbiditiy (diabetes, sleep apnea, hypertension, lipds). Or greater than 40 w/o comorbidities. Also all patients must be psychologically stable and willing to comply with post-op lifestyle changes of diet, exercise and f/u programs. Traditionally, bariatric surgery is offered to patients from 18 to 60. *BMI is weight in pounds/ (height in inches squared (so like 60 inches x 60 inches)) x 703. A BMI of less than 18 means you are under weight. A BMI of less than 18.5 indicates you are thin for your height. A BMI between 18.6 and 24.9 indicates you are at a healthy weight. A BMI between 25 and 29.9 suggests you are overweight for your height. A BMI of 30 or greater indicates obesity. If you are obese, consider consulting a doctor or losing weight.

Appendicitis signs

Rovsing's sign- when you palpate the left side and there is pain ilicited on right side Psoas sign- pain when flex hip Obturator sign- pain when turn hip internallly McBurney's sign-is the name given to the point over the right side of the abdomen that is one-third of the distance from the anterior superior iliac spine to the umbilicus (navel). This point roughly corresponds to the most common location of the base of the appendix where it is attached to the cecum

SIRS, Sepsis, severe sepsisk, septic shock

SIRS (systemic inflammatory response syndrome): 2 or more of....high temp, tachycardia, high RR, high WBC Sepsis: SIRS and a nidus of infection Severe sepsis: Sepsis with evidence of organ hypoperfusion (lactic acidosis, oliguria, altered mental status) Tx: administer activated protein C; improved survival rates with people w/ APACHE score >25. Septic shock: severe sepsis w/ hypotension despite adequate fluid resuscitation requiring vasopressors to mainatin blood pressure Tx: most commonly used vasopressor is norepinephrine

Explain how aspirin causes two types of metabolic conditions

Salicylates directly stimulate the resiratory center and produce respiratory alkalosis. By building up an accumulation of organic acids, salicylates also produce a concomitant metabolic acidosis.

Pheochromocytoma

Screening: Plasma metanephrines Confirmatory study: 24 hour urine metanephrine (VMA, metanephrine, and normetanehprines) How to diagnose pheochromocytoma? With abdominal CT scan. *A MIBG scan is highly specific for confirmation of pheochromocytoma and usually obtained after CT to fonfirm DO NOT give beta blocers without alph blockers. Blocking beta-receptors leads to unopposed stimulation of vascular alpha-receptors by circulating catecholamines, which may result in a rapid, catastrophic increase in blood pressure. Alsways give an alpha blocker first, followed by a beta blocker; doing this in the wrong order can lead to a dangerous increase in blood pressure.

Breast cancer in pregnancy

Sentinel lymph node biopsy not recommended in pregnancy cuz of radioactivity of methylene blue. You can use, however, radiosulfer collid. Radiation contraindicated in all 3 trimesters Chemo: congtraindicated in 1st trimester Most of the time the most appropriate treatment is immediate mastectomy

What do you look at to assess protein nutritional adequacy?

Serum albumin. The accuracy of this estimate is affected by long half-life of albumin (3 weeks) and the vagaries of hemodilution.

What is the physiologic consequence of shivering?

Shivering is the body's effort to generate heat to maintain core temperature. In healthy person, shivering increases the metabolic rate by 3 to 5 times and results in increased 02 consumption and C02 production. In critically ill patients, these metabolic consequences are almost always counterproductive and should be prevented with other means employed to correct systemic hypothermia. In the presence of vigorous shivering, 02 debt in the muscles and lactic acidemia.

Rib fractures

Should be suspectd in all patients with localized chest wall tenderness following trauma; up to half of rib fracrures will NOT be evident on initial X-ray. Rib fracture is associated with significant pain, which causes hypoventilation that may ultimately result in atelectasis and pneumonia. Thus, pain mangaemtn and respiratory support are the priorities in the management of rib fractures.

How does Gastrinoma (Zollinger Ellison syndrome present? Where do they present?

Should be suspected in patients with multiple stomach ulcers and thickened gastric folds on endoscopy. Dx is strongly suggested by a FASTING SERUM GASTRIN LEVEL of >1000 pg/ml. Patiens with non-diagnositc serum gastrin levels should be evaluated with a secretin stimulation test. 90% of gastrinomas are located within the gastrinoma triangle-the 3 corners of the triangle are defined by the junction of the second and third portions of the duodenum, the junction of the neck and body of pancreas, and the junction of the cystic and common bile duct.

GERD signs, dx, tx

Signs -substernal burning pain -dysphagia -hypersalivation, with SOUR tase in mouth -cough (particularly nocturnal), hoarseness -wheezing Dx; Often, you go first with a trial run of PPi and see if symptoms are relieved. But, if you have alarm signals for cancer...weight loss, advanced age, nausea, refractory symptoms, guaiaic positive, or dysphagia, vomiting, GI bleeding... an ENDOSCOPY to r/o cancer is required. Tx: Phase 1: Lifestyle modicitaion: Elevate head of bed, discontinue foods that decreases LED tone, avoid food <3 hours before bed. Antacids Phase II: add H2 blocker Phase III: switch to PPI Phase IV: add a promotility agent like metoclopramide (dopamine blocker) Phase V: combination of medications Phase VI: surgery correction *surgery for those who have persistent or progressive disease despite maximal medical therapy or with structureally defective LES. Goald of surgery is to return normal sphincter length and function and return the physiologic swallowing function of the esophagus. Procedure of choice is a fundoplication-wrapping the fundus of the stomach around the distal portion of the esophagus to create a sphincter, called the Nissen procedure. PHarmocologic: H2 blocker, PPI Complications of GERD. Know. -Recurrent pneumonia due to pulmonary aspiration: cytologic apsirate findings on broncho that can dignose aspiration of gastric contents are LIPID LADEN MACROPHAGES (from phagocytosis of fat). -Esophagitis - Esophageal (peptic) stricture: dysphagia to solids -barret's esophagus; rmember, Barret's does NOT regress, even if GERD is successfully treated -esophageal cancer;: upper two thirds is isuallu squamos cel and lower one thir is usually adenocarcinoma

Rectal carcinoids

Slow growing tumors. Manifests systemic signs of carcinoid only when hepatic metastases present. Malignant potential is low for tumors less than 2 cm.

Paget disease of breast

Small percentage of breast cancers. Progress towrad nipple-areola complex where it causes typical clinical findings of nipple eczema and erosion. Up to half of Paget disease patients have associated breast mass. Diagnosis: Mammogram and PUNCH biopsy. 'vacuolated cells' is the path buzzord. Recommended sugical procedure or Paget disease is radical mastectomy. Waiting, steroid creams, and antibiotics are not appropatie forms of managemetn in a woman who presents with rash involving the nipple.

Liver transplant contraindications?

Specific exclusion criteria for liver transplantation are not formally established. Some of the more common contraindications are ongoing or recent substance abuse, active sepsis, current malignancy, poor cardiac or pulmonary function, hepatocellular carcinoma w/metastatic disease, significant tumor burden.

What should you worrry about in a non-healing wound that is a cancr?

Squamos cell carcinoma may arie with chronically wounded, scarred, or inflamed skin. SCC arising within burn wounds is known as a Marjolin ulcer.

esophageal cancer

Squamos cells (smoking and drinking) and adenocacrinoma (this one related to barrets esopagus) To diagnose: barium swallow followed by endoscopy. Surgery can be done that is curative if the metastasis not present. If metastiss then go with chemo and radiation for palliative treatment becase disease usually not curable. Surgical resection, espeically for sypmtoms control.

You think someone has aortic dissection cuz of tearing chest pain and different blood pressures in each arm, what do you do?

Start IV BETA BLOCKER stat *cuz you want to reduce the rate of change of blood pressure and reduce shear on the aortic wall. Order CT angiogram to confirm aortic dissection. Type A (looks like ascending and descending) dissections typically require surgery and medical therapy. Type B ( just descending) usually be managed with medical therapy. *this can require operative intervention if there are signs of end organ failure (renal failure, lower extremity ischemia, intestinal ischemia), inadequate pain relief despite optimal medical therapy, and rupture or signs of impending rupture)

Guy says that people in his family have gotten paralysis from some types of anesthesia?

Suggestive of pseudocholinesterase deficiency which prolongs the effects of succinylcholine ( depolarizing neuromuscular blocking agent) as well as mivacurium (nondepolarizing). Other nondepolarzing agents like vecuronium aren't affected by this enzyme deficiency.

4 basic types of Melanoma

Superficial Spreading Melanoma -Most common type -raised borders and brown lesions with pinks, whites, grays, or blues Lentigo Maligna -Least common of four types. When found, its most often found in the eldely. -Tan to brown Acral Lentiginous Melanom -Most common type found in Africans Americans and Asians. Usually found under nails, soles of feet, palms of hand. Nodular Melanoma -Most agressive and 2nd most common type. -Brown to black lesions that arise from nevi or normla skin. *Melanoms less than 1mm thick have a low rate of metastasis *Microstaging of melanomas (prognostic indicator i think): Breslow method (more accurate than clark ethod) is based on depth of invasion:vertical height of melanoma from granular layer to the area of of deepest penetration. Tx: surgery is primary theerapy for patients with nodal involvement. margins are important. like you want a 1-2 cm margin for a lesion 1-2 cm in thickness. And 3 cm margin for legions >4 cm in thickness. no role for radiation therapy pre or post operatively for melanoma.

Treatment of stage A bladder cancer? (superficial and submucosal)

Superficial and submucosal: transurethral resection of lesions and intravesicular chemo More advances stages: surgical resection, systemic chemo, and radiation.

Tibial nerve?

Supplies the muscles of the posterior compartment of the thigh, posterio compartment of the leg, and the plantar muscles of the foot. These muscles control flexion of the knee and digits, and plantar flexion of the foot. Tibial nerve provides sensation to the leg (except medial side) and plantar floot.

Pancreas transplant rejection signs?

Suspect in presence of high amylase, high lipase, or high glucose. BUt these aren't diagnosit. You need a biopsy of the pancreas for confirmation of diagnosis. *in pancreas transplant recipient with bladder drainage (connectino of donor duodenum to the bladder), a decreasing urinary amylase is suggestive of rejection.

Insulinoma presentation and treatment

Symptoms relate to rapidly falliing blood glucose level and are caused by epinheprhine release triggered by hypoglycemia (sweating, tachy, weakness). Whipple triad summarizes the clinical findings in insulinoma patients 1. attackss precipitated by fasting or exertion 2. fasting blood glucose concenctrations below 50 mg/dL 3. symptoms relieved by oral or IV glucose. Tx: simple excision of the adenoma is curative in majority of cases

Lung abscess treatment?

Systemic antibiotics. Therapy can last up to 12 weeks. Often, the abscess drain spontaneously but if it fails medical therapy, intevention ranging from percutaneous to surgical drainage to resection.

Treatemtn of pyoderma gangrenosum?

Systemic steroids and innumosuppressants like cyclosporine.

TNM staging

T: how big the tumor is N: how much clavicualr and axilary node involvement M: judugin if metasttic disease present STages 0-4 Stage 0: carcinoma in situ by definition are not invasive Stage 1 Tx: lumpectomy + axillary dissection + adjuvant radation. OR modified radical mastectomy. Stage 2 Stage 3 Stage 4 : when metastes are present

Common cause of primary adrenal insufficency (Addison's disease) in endemic areas? Common cause of secondary adrenal insufficency (most common cause of adrenal insufficency that is seen in hospital according to Dr. basil?)

TB is a common cause globally. Causes aldosterone deficiency and presents with Non AG, hyperkalemic, hyponatremic metabolic acidosis. Common cause of secondary adrenal insufficency: not tapering sufficient when taking someone off steroids, if someone has been using steroids for longer than 2 weeks and if you just abruptly stop you will be left with an adrenal gland being insufficient, so that is why you taper when you take someone off steroids Another thing to remember about adrenal isufficiency, if the person has increased physilogical stress they should receive a stress dosage of IV hydrocortisone. People with adrenal insufficiency are eductaed to increase their corticosteroid dosage during stressful event, such as infection or surgery.

Everything about blood transfustions

TRANSFUSION REACTIONS Seconds-Minutes: Anaphylaxis ---IgA deficient individuals are at risk due ot presence of anti-IgA IgG antibodies. Minunted-hour: Acute hemolytic ---Careful cross matching of blood prevents this but sometimes ABO mismathcing can occur due to clerical errors..whci presents with fever, chills, flank pain and hemoglobinuria within an hour of transfustion. Can progress to renal failoure and DIC. The direct antiglobulin test (Coombs) is positive and plasma free hemoglobin is >25 mg/dl. UA also shows hemoglobinuria. TX: Lead to hypotension, oliguria, hemoglonbinuria. Tx begins with discontinuation of transfusion, followed by aggressive fluid resuscitation to support the hypotensive episode and increase urine output . inducing diuresis through agressive fluid resucuitation and osmotic diuretics is important to clear the hemollyzed red cell membranes, which can otherwise collect in glomeruli and cause renal damage. *A a foley catheter should be inserted and hourly urine output should be monitered cuz of potential renal damage from precipitated hemoglobin in tubues. The precipitation is inhibited in alkaline environemnt. So alkalinizing the urine is a good thing to do too. 1 hour-6 hours: Febrile nonhemolytic, TRALI ---febrile hemolytic: when red cells and plasma are separted from whole blood, small amounts of residual plasma and/or leukocyte debris may remain in the red cell concentrate. During blood storage, these leukocytes release cytokines, which when transfused can cause transient fevers, chills, and malaise, without hemolysis. 2 days-10 days: Delayed hemolytic ----causes low grade hemolysis, rquires no treatment in most cases. Result from an anamnestic antibody response to a red blood cell antigen to which the patient was previously sensitized. The antibody is undetectable prior to transfusion but reappears rapidly following transfusion. INDICATIONS FOR SPECIALIZED RBC treatment IRRADIATED -Bone Marrow transplant recpiients -Acquired or congenital ceullular immunodeficiency -Blood components donated by first or 2nd degree relatives LEUKOREDUCED: involves reducing the number of transfused leukocytes through filtering or other methods such as saline washing, freezing, and deglycerolizing or buffy coat removal. It also reduces the risk of HLA allimmunization. -Chronically transfused patients -CMV at risk -Potential transplant recipietns -Previous febrile nonhemoltyic tranfustion reaction WASHED -IgA deficiency ---Residualt plasma in red cell concentrates contains proteins, including IgA. IgA defieicnt patients develop antibodies against IgA. These antibodies can react with IgA-containing donor products and induce an anaphylactic reaction that can progress rapidly to loss of consciousness, shock and respiratory failure. To reduce this risk, red cells should be washed to remove as much of the plasma as possible for patients with IgA deficiency or prior allergic transfusion reaction. -Complement dependent autoimmune hemolytic anemia -Contiued allergic reactions with red cess transfusion despite antihistamine treatment also.. -calcium gluconate infustion is used to avoid or treate severe hypocalcemia following massive blood transfusion. What happens is prior to storage, whole blood (for transfusions) is generall ymized with solutions containing citrate anticoagulant. Packed cells derived from these whole blood collections also contain citrate. Infused citrate can chelate serum calcium, causing hypocalcemia. -Warming blood is recommended only during rapid massive transfusion to prevent hypothermia. -Whole blood contains numberous leukocytes and increases the risk of febrile reaction. There, it is rarely used except in cases of acute massive hemorrhage (trauma). -stored RBCs gradually lose intracellular potassium in to the surrounding solution. This has the potential to cause hyperkalemia. -there is also urticarial/alergic transfusion reaction: within 2-3 hours, caused by recipient IgE antibodies and mast cell activation -there is also transfusion-related acte lung injury (TRALI) respiratory distress and signs of noncardiogenic pulmnoary edema. Within 6 hours. Caused by donor anti=leukocyte antibodies. TX: Respiratory support, including mechanical ventilaino if needed. -there is also trasnfusion associated circulation overload (TACO) which occurs in patients with underlying heart failure who receive large volume transfusions-the treatment is administration of diuretics

Testing for Cushings syndrome

Tests: -24hr cortisol on urinalysis -midnight salivary cortisol (normal people: low at night, high in morning) -overnight low-dose dexmethsone suppresion test ---First you measure ACTH. If that is low then you either have an adrenal tumor (MRI to confirm) or you're using exogenous steroids. If the ACTH is elevated then that means you further have to distinguish between Cushing disease and ectopc ACTH . Dexamethsasone is glucocorticoid analog that can inhibit ACTH secretion from pituiatary. If you do the suppression test now, if ectoic secretion ACTH will not decrease with dexamethasone because the source if resistant to negative feedback. If ACTH does decrease then that points to Cushings disease.

How does carcinoid syndrome present?

The carccinoid tumors release products like histamine, serotonin, VIP that are released in blood but liver metabolizes. If liver gets meatstasized and starts to fail with the metabolizeing, the products go out to systemic circulation. Episodic flushing Secretyory diarrhea Wheezing Murmur of tricuspid regurgitation-pathognomonic plaque-like deposits of fibrous tissue occur most cmmonly on ednocardium on right side of heart, leading to tricuspid regurg and right sided heart failure Dx; elevated 24 hour urinary 5-hydroxyindoleacetic acid can confirm diagnoses Surgical resection: -WHen tumors are larger than 1 to 2 cm or involve the base of appendix a right hemicoectomy should be performed. -WHen tumors are at tip of appendix and are less than 1 cm in size, simple appendectomy is the procedure of choice.

Fascia Dehiscence

The disruption of fascia closure within days of an operation; this complication may occur with or without evisceration. ---drainage of a large amount of serosanguineous fluid is highly suggestive of fascia dehiscence. This can help with differentiating between fluid from dehiscence or enterocutenous fistula, whose fluid is either bilious or feculent. Evisceration: The presence of abdominal viscera (bowel or omentum) protruding through a fascial dehiscence or traumatic injury. Complications: can lead to evisceration, enterocutaneous fistulas (communication between small bowel lumen and skin opening, and formation of incisional hernia. Risk factors: failure of surgical technique or anesthetic relaxation. Deep wound infection. Age, diabetes, malnurtrion and perioperative pulmonary disease. Tx: local wound care, followed by elective repar of facial defect at later time. Immediate surgery is indicated fo evisceration or bowel exposure. *The tensile strength of uncopmlicated wounds steadily increases for approsximately 8 weeks, when it reaches 80% of that of normal tissue; but the strength never reaches that of uninjured tissue.

What does positive pressure mechanical ventilation do? Indications for it?

The inability to maintain a Pa02 of 60 or an oxgen sat. of more than 91% with a supplemental non-rebreathing -2 mask is indicative of a significant alveolar-arterial (A-a) gradient, and intubation and mechanical ventilation may be needed. Acutely increases intrathoracic pressure, increasing right atrial pressure and decreaseing systemic venous return. The sudden loss of venous return may cause acute circulatory failure and death. In patients with hypovolvemic shock, this effect may cause ciruclatory collapse if the patient's intravascular volume is not replaced before mechanical ventialtion is attempted. *Also, you can get Ventilator-induced Lung Injuy (VILI). inury can happen secondary to overinflation, distension, and repetitive opening and closing of the alveoli. HIgh oxygen content delivery to the lungs also prodcues oxygen-free-radical associated lung injuries. Ways to minimze VILI include using a low tidal volume, reducing Fi02, and application of positive ende expiratory pressure (PEEP).

Most commonly fracture bone among the carpals?

The scaphoid, commonly seen in young adults following a fall on outstretched hand. Wrist immobilization is recmmended treatment of all nondisplaced scaphoid fracutre (fracure with <2 mm of displacement and no angulation). You want to do a X-ray then and an additional X-ray in 7-10 days. ALso UWorld says you want to do a MRI or CT of the wrist additional imagin. Open reduction and internal fixation is rquired if X-ray shows frature displacement. *scaphoid fracutres are particularly concerning because of the tenuous blood supply of the scaphoid, making it vulnerable to avascular necrosis. Initial X-ray can be normal if the fracture is compressed or minimally displaced, so that can take up to 10 days to show on x-ray. So if the clinical picture fits with scaphoid fracture but X-ray is normal, immobilize thumb in spica cast and repeat x-ray in 7-10 days. *patiens usually ahve pain on the radial aspect of the wrist in the anatomic snuffbox, minimally decreased ROM, decreased grip strenth, and possible swelling. *Scaphoid fractures that are not treated approripately or in a timely fashion can result in avasculary necrosis and increase the risk of nonunion.

Gastrointestinal stromal tumors (GISTs) are treated with what?

These express c-kit receptor which is a tyrosine kinsae. So imatinib which is a selective tyrosine kinsase inhibitor can be used as neoadjuvant or palliative therapy for GIST.

Thoracic outlet syndrome

Thought to be result of brachail plexus or subclavian vessel compression. Signs: pain, paresthesias, edema, venous congestion, Tx: Initially should be exercise program to strengthen shoulder girdle muscles and decrease shoulder droop. Then there is operative treatment ( division of sclenus anticus and medius muscles, first rib resection, cervical rib resection, or a combo of all 3). from wiki:TOS may result from a positional cause, for example, by abnormal compression from the clavicle (collarbone) and shoulder girdle on arm movement. There are also several static forms, caused by abnormalities, enlargement, or spasm of the various muscles surrounding the arteries, veins, and/or brachial plexus, a fixation of a first rib, or a cervical rib. A Pancoast tumor (a rare form of lung cancer in the apex of the lung) can lead to thoracic outlet syndrome in the progressive stages of the disease. The most common causes of thoracic outlet syndrome include physical trauma from a car accident, repetitive strain injury from a job such as frequent nonergonomic use of a keyboard, sports-related activities, and anatomical defects such as having an extra rib.

Patient has appendicit >5 days?

Thse people have high incidence of perforation with formation of an abscess. They often have a contained abscess. If the patients are otherwise stable, they may be treated with intravenous hydration, antibiotics, bowel rest and interval appendectomy (which means they can return in 6-8 weeks for an elective appendectomy).

Anterior mediastinal mass (4Ts)

Thymoma T-cell lymphoma Teratoma Thyroid goiter

Thyroid cancers.

Thyroid cancers. CARCINOMA (4) -Papillary: most common, good prognosis, propensity for lymphatic spread; lymphadenopathy. Radiation and family history. Tx: of high-risk ones are near-total or total thyroidectomy. If lymph node metastases in the lateral neck, concomitant modified radical neck dissection should be performed with total thyroidectomy. Papillary carcinoma of thyroid frequently metastasizes to cervical lymph nodes, but distant metastasis is uncommon. -Follicular: good prognosis, hematogenous spread Tx: surgery followed by radiodine ablation *<4cm in size can do thryoid lobectomy cuz most are adenomas <4cm or confirmed carcinoma then total thyroidectomy should be done. follicular carcinomas cannot be diagnosed by FNA; capsular or vascular invasion on histology confirms diagnosnsos of malignancy. -Anaplastic: older patients invades local structure, very ppor prognosis -Medullary: from parafollicular C cells; produces calcitonin LYMPHOMA -assoicated with Hashimotos thyroiditis. good prognosis. Tx: surgery followed by radioidine ablation Genral clinical presentation is painless enlargement of thyroid, more advanced signs of local invacion are hoarseness, dhysphagia, cough. *Age is a very important prognositc factor in papillary and follicular tumors

Echinococcus liver cyst management?

Total pericystectomy. Definitive treatment requires surgical resection, enucleation, or evacuation of cysts. At time of surgery, inject silver nitrate or hypertonic saline into cyst.

A rare but deadly complication of a tracheostomy?

Tracheoinnominate artery fistula (TIAF). When suspecrted, the diagnosis should be confirmed or ruled out in the operating room. TIAFs can occur as early as 2 days after tracheostomy or as late as 2 months after procedure. OFten associated with low placement of a tracheostomy (distal to 2nd and third tracheal rings). The patient may have a sentinel bleed in 50% of TIAF cases, followed by a very impressive bleed. If a sentinel bleed is sucpected, the patient should be transported immediately to the operating room for bronchoscopic evaulation. Initial maneuvers for management of a TIAF inculde overinflation of the cuff on the tracheostomy or reintubation from above followed be removal of the tracheostomy and finger compression of the innonminate artery against the sternum trhough the tracheostomy wound.

Pleural effusion.Types? Dx?

Transudative-CHF, Cirrhosis, Nephrosis. Due to incrased hydrostatic pressure or decrased oncotic pressure. Exudative-due to increased capillary permeability, usually due to inflammatory process. Infection, tumor, trauma, pulmonary embolism. Management: Any pleural effusion should be tapped for analysis. Except if there are signs of CHF, in which a trial of diuretics is warranted. For analysis do total and differnetial cell count, protein, glucose, LDH, and ph (if there is suspicion for empyema). -Ph <7.2 and/or glucose <60 is indication for thoracostomy. LDH and protein can point to exudative causes but are not absolute indications for thoracostomy. -why low glucose? due to high metabolic activity of leukocytes and bacteria within the pleural fluid. Light's critera: Effusion is exudate if one or more more is present. The reasons is because the increased capillary permeability lets more protein and LDH through to reach the pleural space -Ratio of pleural to serum protein > 0.5 -Ratio of pleural to serum LDH >0.6 -Plerual fluid LDS> 2/3 upper normal limit of serum LDH -also if ph is pretty normal that helps point to transudative because the transudative fluid is just normal body fluid -low glucose usually seen in complicated parapneumonic effusions, malignancy, TB, or rheumatoid arthritis. Parapneumonic effusion devleops in the setting of pneumonia. This specturm. A simple can become 'complex" and 'complicated" with the other end being empyema in which frank pus can be aspirated. Empyemas always require chest tube On x-ray you see a loculated abnormally contoured effusion WITH ADJACENT pulmonary consodliation.

What is cause of Trendelenburg sign?

Trendelenburg sign is the drooping of teh contralateral hemipelvis below its normal horiztontal level during monopedal stance. It is caused by weakness or paralysis of the gluteus medius and minimus muscles, which are innervated by the superior gluteal nerve.

Testicular cancer

Tumor cell types generally divided inot seminoma and nonseminomatous germ cell tumors Radical orchiectomy: a surgical procedure in which an inguinal incision is made over the cord leading to the testicle to be removed. The surgical specimen includes testis, epididymis, and spermati cord taken at internal iliac ring. --this should be done when lesion within scrotum is a solid mass. As ultrasound of scrotum is helpful in this . You should also test for B-hcg and AFP which are elevated in 85% of ppl with nonseminomatous germ cell tumors. LDH can also be elevated in seminomas. *high AFP EXCLUDES pure seminoma or pure choriocarcinoma Earlt stage seimonmas should be treated with orchiectomy AND EBRT. Seminomas are really radiosensitive so lympho node dissection not necessary. Testicular cacner often progresses in an orderly fashion up the lymphactic drainage of the testis. Testicular lymphatics flow from the testis through the spermatic cord following the testicular artery into the retroperitoneum where they drain into the nodes around the vena cava and aorta. After radiacl orchiectomy, lymph node dissection is indicated in embryonal carcinoma, teratocarcinoma, and adult teratoma. Choriocarcinoma are ferquently associated with pulmonary metastasis and is treated with chemotherapy. 90% of testis cacers are derived from the germinal epithelium: choriocarcinoma, embyronal carcinoma, seminoma, teratoma, and yolk sac tumor. 10% of other testis cancer are stromal, or secondary tumors.

Glucagonoma

Tumor of islet alpha cell, causes a characteristic rash, diabetes, anemia, weight loss and elevated glucagon. Usually present in body or tail of pancrease and easily identficiable on CT scanning. Tx is surgical excision with distal pancreatectomy. Metastases are common and should be resected whenever feasible.

BPH vs prostate cancer

Tx for BPH: alpha 1 blockers (doxazosin or tamsulosin) prefferred alone or in combination with 5-alpha reductase inhibitors (finastreide) for symptoms relief. When medical therapy fails, next step is surgery with the standard operavitve procedure being TURP (tranurethral resection of the prostate). *Indications for surgery in patients with BPH include recurrent UTI, renal insufficnecy secondary to outflow obstruction, bladder stones, upper tract dilation. *Patients with symptoms suggestive of BPH should undergo a renal funciton test (creatinine), a PSA test, urinalysis and a DRE. PSA antigen can screen for prostate cancer. However, PSA can also be elevated (>4ng/dl) in BPH. Prostate cancer is more peripheral in growth and rectal exam shows NODULES. BPH is more central and rectal exam is smooth. Remember, all patients with suspected BPH should hav e UA (to look for blood) and serum creatinine tested. You wanna make sure you're not having bladder outlet obstruction that is causing renal failure. Generally, creatinine is not elevated by unilateral urinary obstruction but wil be elevated due to bilateral obstruction. As a result, patients with elevated creatinine require imaging (renal US) to assess for hydronephrosis and other causes of obstruction. PSA screening controversy: USPSTF (US preventative services task force) does not recommend any population screening using PSA. But American Cancer Society and American urological association disagree with this.

Tx of hypercalcemia

Tx of hypercalcemia SHORT TERM IV fluids +/- Lasix (use diuretics if volume overload present ) Calcitonin LONG TERM Bisphophonates (pamidoronate, alendronate, zolendronate) ----binsphosonates bind to bone, get engulfed by oseteoclasts and kill thems. Side effects: pill esophagitis, osteonecrosis of jaw *apparently tho bisphosphonates are the initial drugs of choice for hypercalcemia of malignancy. also good for osteoporosis. *Parathyroidectomy is recommended for asymptomatic hypercalcemia patients who have at least one of the following features.. -serum calcium level >1.omg/dl above the upperlimit of normal -young age (<50 yrs) -bone mineral density <T-2.5 at any site -reduced renal function (GFR<60) **all patients under consideration of parathyroidectomy should undergo preoperative localication study to determine feasibility of minimum invasive parathyroidectomy or unilateral explorations. Some common ones are ultrasound, sestambi scan (nuclear imagin), MRIs, and CTs.

Types of lung cancer:

Types of lung cancer: 1.Small cell lung cancer: central, senstive to chemo, NO surgery, poor prognosis. most paraneoplastic stuff associated with small cell. 2.Non-small cell lung cancer: squamos cell, large cell, adenocarcinoma. poor response to chemo. Treated with surgery if early stage Squamos cell: central. Only paraneoplastic associated with non-small cell is PTH related peptide (presents with hypercalcemia, classically in squamos subtype). -hypertrophic osteoarthropathy usually associated with adenocarcinoma. -adenocarcinoma is most common type of lung cncer in both smokes and nonsmokers as is the predominant type seen in nonsmokers. squamos, large cell, small cel are rarely seen in nonsmokers. Presents peripheraly. Large cell: Peripheral. Gynceomastial and galactorrhea.

Acute rejection?

Typically occurs bewteen 1 week and 3 months posttransplantation and is characterized by an increasing creatinine, decreased urine output, and possible fevers or tenderness over the graft. Diagnosis is confirmed with a biopsy of the graft and treatemtn includes high-dose steroids and anti-T-cell antibody (OKT3).

Muscles of the hand

Ulnar nerve innervates 15 of 20 intrinsic muscles of the hand Musculocutaneous: forarm supination Radial: forearm supination and innervated extensor muscles Median nerve: "eye" of hand because of large contribution to sensory erception, also maintaines most of long flexors, pronators of forearm, and thenar muscles.

Malignant hyperthermia?

Uncontrolled efflux of calcium from the sarcooplasmic reticulum is the underlying pathophys of malignant hyperthermia, a rare autosomal disorder. It occurs in genetically susceptbiel patients after administration of the anesthetic drugs halothane and succinylcholine and may be associated with severe hyperthermia ( up to 113).

Why put urinay cathter for spinal cord trauma?

Urinary ctheter placement can assess for urinary retention and prevent acute bladder distention and damage.

Esophageal stricture

Usually a results of inflammatory or neoplastic process Risk factors and causes -long standing GERD -radiation esophagitis -infectious esophagitis -corrosive esophagitis -sclerotherapy for bleeding varices Signs -progressive dysphagia for solids -odynophagia may or may not be presents Dx: esophagoscopy is necessary in all cases since the stricture should be evaulated for malignancy Tx: bougie dilators for dilation. most feared complication is esophageal rupture

where do psoas abcess come from usually?

Usually hematogenous spread from another site, usually the skin. Get a CT scan if you suspect psoas abscess. The people generally present with fever and lower abdominal or back pain

Types of ulcers

VENOUS stasis ulcers -typically located over medial malleolaus, painless, brawny induration sometimes. Tx: compression stockings, elevation, wound care, ISCHEMIC ulcers: often associated with other symptoms of severe peripheral vascular disease like rest pain. Commonly located on dorsum of foot or lateral first or fifth toes. Tx: patiens undergo urgent evaluation for lower extremitiy revasculuriaztion. DIABETIC ulcers: often associated with trauma or pressure secondary to sensory neuropathy and typicall occur on plantar side of foot. Tx: good glucose control, protective shoe wear, and debridement of necrotic tissue. PRESSURE (decubitus) ulcer: pressure on bony prominence like in contact with bed. typical locations are sacrum, HEELS, elbows and ears. MARJOLIN ulcer: swuamoc cell carcinoma that arises in a chronic wound and is treated with surgical excision PYDOERMA GANGRENOSUM: IBD associated. Lesions are painful ulcerations surrounded by erythema. Tx: steroids and immunosuppresants.

Splenectoy things to know

Vaccine with people who get splenectomy? Vaccinations should be given for the encapsulated organisms Strept. pneumonia, Neisseria meningitidis, Haemophilus influenzae. Vaccinations against each of these organissms should be administered either >14 days before scheduled splenectomy or >14 days after splenectomy. What should you give someone who is getting splenectomy? Current recommendations state that patients should recieve anti-pneumococcal, Hameophilus, and meningococcal vaccines severeal weeks before the operation, and daily oral penicillin prophylaxis for three to five years following splenectomy.

CVP stuff

Venous tone, right ventricular compliance, intrahoraci pressure and blood volume all influnece CVP. Increase CVP: vasoconstrictor drugs, postiive-pressure ventilation (with and without PEEP), mediastinal compression and hypervolemia all increase CVP. Aucte pulmonary embolism can elevate CVP too by causing RV onverload. Decrease: sepsis decreases CVP through both the release of vasodilatory mediators and the loss of IV plsasma volume due to increasd capillary permeability.

Pulmonary contusion vs ARDS

Very important to differntialte pulmonary contusion from ARDS. Pulmonary contusion: parenchymal bruising of the lung, which may or may not be associated with rib fractures. The clinical manifestations develop usually in the first 24 hours (often with in few minutes_; tachypnea, tachycardia, and hypoxia are characteristic. Physical exam typically reveals chest wall bruising and decreased breath sounds on the side of pulmonary contusion. Chest x-ray reveals patchy irregular alveolar infiltrate, ABG typically shows hypoxemia and itself is an indication to suspect pumonary contusion. ARDS: manifests 24-48 hours from trauam, besides that, bilateral involvement is present.

What are risk factors for polyp progressing into malignancy

Villous adenoma, sessile (opposited of pedunculated)adenoma, and size >2.5cm. Polyps classified as following 1. Hyperplastic: Most common non-neoplastic kind, no further workup needed. 2. Hamartomatous: Include juvenile polyp (non-malignant) and Peutz Jeghers poly (non-malignant) 3. Adneoma: Most commont type. Potentially premalignant, but <1% are malignant. Most are asymptomatic. Adenomas... can be sessile or stalked (pedunculated) Can be tubular, tubulovillous, or villous And various sizes.

You see a polyp, take it out, and pathology sees invasive component, next step?

What you've already done is adqueate provided that 1. No vascular or lympatic invation is present 2. there is an adequate nagative margin (2 mm), the the cancer is not poorly differentiated.

Oliguria post-op

When oliguria occurs post-op, its important to differntiate between low output caused by the physioligc response to intravascular hpovolemia and that caused by ATN. FeNa <1% suggests aggressive sodium reclamation in working tubuels. Values aboe this suggest tubular injury such that Na cannot be approriately reclaimed.

Post Op Fever

Wind-atelectasis, pneumonia Water-UTI Wound-infection Womb-endometritis, uterin infection Walking-DVT Wonder: drugs,

What kind of wounds should be left open?

Wounds that are dirty or contaminated (animal bites), that are traumatically induced by a puncture, gunshot, or crush injury, or that are older than 6 hours should be left open. Prophylactic antibiotics have not been demonstrated to prevent wound infections.

When do you give Tetanus-diptheria toxoid vaccine vs the vaccine + tetanus immune globulin when dealing with a wound?

You give the regular vaccine for a minor wound if its been more that 10 years since patient had it. If the wound is dirty or severe then you give the regular vaccine if its been more than 5 years since the last booster was given. If the patient has an uncertain vaccination history of its incomplete (< 3 teatnus vaccines given) then you only give the regular vaccine if the wound is minor. BUT if the wound is dirty or severe then you give the regular toxoid vaccine + tetauns immune globulin.

PFT stuff to remember from lecture with Dr. Chadha

You know the FEV1/FVC stuff. Know this as well. DLCO-low in COPD, pulmonary hemorrhage. DLCO-normal to high in asthma TLC-normal to high in obstructive TLC-decreased in restrictive Need to correlate Fev1/FVC w/TLC to see if it agrees. If not, maybe there is obstructive AND restrictive going on. DLCO: measures the ability of lungs to transfer gas from alveoli to RBCc in pulmonary capillaries. Low in conditions characterized by loss of lung tissue or barriers to gass diffustion (for exaple, interstiial edema, intersittial infiltrates, tissue fiborsis) or loss of lung tissue (emphysema).

Approach to platelet abnormalities

You wanna think first is it a quantitative (high or low) problem or qualitative (normal count but abnormality in function) QUANTITATIVE 1. Thrombocytopenia ---Decreased production: bone marrow failure, bone marrow invation, bone marrow injury by drugs, chemicals, radiation. ---Increased destruction: durugs, HIT, ITP, TTP, DIC ---Sequestration from splenomgaly 2. Thrombocytosis ---Reactive or autonomous

Torus palatinus?

Young person with fleshy immobile mass on midline hard palate. No medical or surgical therapy is required unless the groth becomes symptomatmic or interers with speech or eating.

When to suspect a soft tissue sarcoma?

Young, healthy, active individuals with extremity STSs are frequently misdiagnosed as having a hematoma or a bruised muscle because sarcomas are infrequently encouraged by physicians in most medical practices. So, its imperative to consider STS whever an unexplained sot tissue mass or swelling is identified in an individual of any age. Things that raise suspicion for STS are large size, absence of a specfici event to account for a hematoma of this size, the firmness of the mass, absence of surrounding skin changes to suggest an inflammtory or infectious process. *also if the mass is nontender then soft-tissue injury from a trauma should be ruled out. Most patients with STS present without regional lymphadenopathy or systemic symptoms like weight loss, nigtht sweats, or cachexia. *Li-Frameni syndrome: cancer presdispostion syndrome associated with soft tissue sarcoma, breast cancer, leukemia, osteosarcoma, melanoma, and cancer of the colon, pancreas, adrenal cortex and brain. DDx for includes benign lipoma, which is much more common. Tx: current standard of treatment is wide local excision. Complete resection with 2cm MARGIN is reasonable to ensure ngative microscopic margins.

Testicular torsion presenatation and management

acute pain, High-riding (as the tesis twists, come to lie in higher positiion), firm and markedly tender testes. Usually happens in boys ages 12-18. Dx: Color Doppler US. However, this isn't required for diagnoses. Tx is srugery immediately. If treated within 4-6 hours, chance of saving testicle is high. If testicle still viable, do ORCHIOPEXY is performed on BOTH affected and non-affected tesis. If testicle not viable, ORCHIECTOMY on bad testicle and ORCHIOPEXY still done on non-affected testis. Done because it has greater-than-normal chance of torsion in future.

Shoulder athroplasty

axillary nerve damage most common complication also watch out for cephalic vein

Patient has energy expendiutre of 2000 kcal/day. Match these situations with the appropriate daily energy requirement. starvation, multiple organ failure, third degree burns over 60%, after surgery

basal metabolic rate is the energy required to maintain cell integrity in the resting state at a normal physiolic temperature. Patient's clnical condition also impacts the basal energy expendiutre. During starvation: metabolic rate is decreased by 10% Trauma, stress, sepsis, burns, and surgery all increase the metabolic rate. The basal energy expenditure can be multiplied by a stress factor to better approiximate caloric requirements. The stress factor after a routine operation is 1.1, multiple organ failure or severe injury is 1.5, and more than 50% body surface area burns is 2.0.

Fractures

categorized by anatomy: proximal or middle third of shaft. direction of fracture line: transverse, oblique, spiral OPEN FRACTURE -fracture communicates with the external enrivronment due to a breach of the overlying soft tissue -TRUE ORTHO EMERGENCY: almost always results in bacterian contamination of soft tissues and bones Tx: early irrigation and debridement, IV ANTIBIOTICS to prevent infection, compartment decompression, and internal or external decompression. *given the amount of muscle damage in open fractures, fasciotamies are liberally performed. PATHOLOGIC FRACTURES -minimal trauma fractures bone that was weakened due to prexisting disease: like osteoporosis (most common) -predisposing conditions: carcinoma, cysts, endochondroma, giant cell tumors, osteomalacia, Paget's, scurvy, rickets -Tx: treat the bone but also you should diagnose and treat the underlying condition STRESS or FATIGUE FRACTURE -complete fracture resulting from repetitive applicatin of minor trauma -most occur in lower extremities and in people involved in sports or military "march fracture" -2nd metatarsal is the one that is most commonly injured. -tibia is the most common bone in body to be affected. classicaly occurs in anterior part of the middle third of the tibia. patient usually develops medial tibial stress syndrome ("shin splints" with no tibial tenderness on palpation) which ca progress with further activity to a complete or incomplete fracture, resulting in pain to palpation of the tibia. -x-ray likely to be NORMAL if seen within first 2 weeks of symptoms -pain with activity only. Tx: decrease physical activity. COMMINUTED FRACTURE -fracture in which the bone is divided into more that two fragments by fracture lines SALTER-HARRIS FRACTURE -fracture involving the physis. Occur irregularly throught the weak zone of hypertrophic cartilage. Type 1 and 2: transverse fracture that doe not vertically cross the germinal cell layer Type 3 and 4: germinal cell layer (growth plate) is traversed by the fracture. Surgical intervention is necessary. Type 5: crush injury to the phsysi such that metaphysis and epiphysis are impacted on one another. No visible fracture line. Poor prognosis with high risk of growth plate arrest. GREENSTICK FRACTURE -very common in children, rarely seen in adults -incomplete and agnulated fracture of long bones. A transveres crack that hangs on to its connection. -think of a young, most twig which would break without snapping apart

Compartment syndrome

causes: fractures, crush injuries, vascular injuries, drug overdose with prolonged limb compression, burn injuries Hallmark finding: first sign in leg is parasthesia inbetween great and second toes. pain that is out of porportion to injury, look for pain on PASSIVE STRETCH. -pain, pallor, parasthesia, pralysis, NOT pulselessness? -per Jill ortho resident: pain out of proportion to palpation is first thing you see. when you get paresthesias and paralysis then it's getting worse. pulselessness would be last step of symptoms and by then big trouble. common places are on ventral foreal area and common reason there is a fracture of both radius and ulna. other common place in LLE is anterior tibia region. -in one of my small groups some students said PAIN is the first sign of acute limb ischemia whereas PARASTHESIA is first sign of compartment syndrome Dx: measure pressure within compartment with monitors: -pressure >30 mmHg if indication for fasciotomy. *reperfusion of a limb folowing arterio-occlusive ischemia for longer than 4-6 hours can lead to intracellular and interstitial edema. Compartment syndrome may occur when edema causes the pressure within a muscular fascial copmartment to rise above 30 mm Hg, leading to further ichemic injury. *If fasciotomy is indicated, all 4 compartments should always be opened (anterior, lateral, superficial posterior, and deep posterior).

notes from lecture

crystalloid: 3 flavors-isotonic (normal saline,lactated ringers) , hypotonic, hypertonic -normal saline: has 154 Na and Cl and ph is 5.7. so its hypernatremic, hyperchlorimic, and acidotic. but it normal situations our serum has good buffering capacity so you won't see a big change in ph of blood. but sometimes in shock, you lose that good buffering capacity, you're already getting acidotic due to lactic acidosis. so using this might worsen metabolic acidosis. also, when you give a huge chloride load your kidneys all they see is an anion so they excrete bicarb which is an anion and you end up with even worse capacity to buffer the acid. colloid (you're looking at volume expansion here): albumin (comes in 5% and 25%), packed RBCs, FFP -how much volume expansion do you get with a given aliquot of colloid vs crystalloid? 1:3 rule: any colloid or blood product you give..multiply that by 3 and thats the amount of crystalloid you would have to give to attain the same amount of volume expansion. this is cuz colloid and blood products stays intravascular, crystalloid leaks out indications for colloid: CHF, ascities. patients unable to synthesize proteins necessary for enough oncotic pressure: liver disease, malnutruion, tranplant recipients. severe hemorrhage or coagulopathy. *Know when replacing fluids: -large volumes may lead to peripheral and/or pulmonary edema -large amoiunts of dextrose may cause hyperglycemia -large amounts of NS may cause hyperchloremic metabolic acidosis -Ringer's lactate given when patient is hypovolemic and in metabolic alkalosis may worsen the alkalosis when lactate is metabolized *average 70kg adult needs about 2.5 L/day or 100 ml/hr unless other factor warrant a higher rate *Urine output: normal is .5cc/kg/hr for adults *oliguria is less than 400cc or less than 6cc/kg per day

What do you do for a tension pneumothorax?

immediate needle thoracostomy should be performed in the 2nd intercostal space at the midclavicular line

Acute pancreatitis

inflammation of pancrease due to parenchyman autodigestion by proteolytic enzymes GETSMASHED Gallstones, ethanol, trauma, steroies, mumps, autimmune, scorpion sting, hypercalemia/hypertiglyceridema (>1000), ERCP, drugs -hypercalemia can cause pancreatitis and pancreatitis can cause hypocalcemia a. acute pancreatitis can generate free fatty acids that avidly helate insoluble calcium salts in the pacreatic bed, resuling in hypocalcemia Signs: -constant midepigastric or LUQ pain that radiates to back, fever, vomiting, tender abdomen, The GREy TURNER signs (flank discoloration) and CULLEN SIGN (perumbilical discoloration) suggests RETROPERITONEAL HEMORRHAGE -remember that US should be peformed in ALL patients with acute pancreatitis once the Dx has been confirmed to look for gallstones Dx: amylase levels, lipase levels (more specific than amylase ), CT, Abdominal x-ray "sentinal loop" , US (may identify gallstones as the cause), ERCP (shoulds be avoided in most cases) Tx: IV hydration, Bowel rest, Antibiotics (can develop infection with gut-derived bacteria). If Gallstones the cause then do cholecystectomy. Sequelae: < 48 hours: usually isolated pleural effusion containing high amylase peripancreatic fluid 1-4 weeks: Pseudocyst (diagonsed by CT). ------treated with percutaneous catheter drainage with antibiotics 4-6 weeks: Abscess; required surical drainage Complications: pleural effusion, ARDS, ileus, and renal failure Prognosis: "GA LAW", Ransons's criteria that predict risk of mortality Glucose (>200) Age >55 LDH >700 AST >250 WBC >16,000

Explain LMWH

inhibit 10a like heparin but don't inhibit as much 2a or plateley aggregation can't be monitored by PT or PTT cuz don't affect either Longer half life than heparin but also nothing exists to reverse effects examples are enoxaparin (Lovenox) Are being used more now because of better conveniance than heparin and decreases risk of side effects (HIT, ostoporosis). -given subq, PTT monitoring not necessary, eeasire to use as outpatient. Patient may be discharged if stable and the patient can continue LMWH until the level of long-term anticoagulation (warfarin) is therapeutic (check INR for this). excreted in kidneys so use cautiously in patient with renal fucntions

How to treat gastric lymphoma from MALT?

intial therapy for MALT lymphas is antibiotic regimen to treat H. pylori. eradication of H.pylori has been assoaicted with complete regression of low-grade MALT lymphomas. Chemo and/or EBRT are used to reat persistent lymphoa after antibiotic therapy or advanced MALT lymphomoas. Surgery is rarely indicated.

Tarsal tunnel syndrome

involves compression of the tibial nerve as it passes through the ankle and is usually caused by a fracture of the bones around the ankle. Patients typically present with burning, numbness, and aching of the distal plantary surface of the foot or toes that sometimes radiate up to the calf.

Chronic renal failure definition

kidney damage of greater than 3 months and/or GRF less than 60. Stages 1-5. Stage 1: kidney damage with normal or increased GFR. >90. Stage 2: kidney dmage with mild decrease in GFR (60-89) Stage 3: moderate decrease in GFR (30-59) Stage 4: severe decrease in GFR (15-29, predialysis stage) Stage 5: kindey failure (GFR<15, usually indication for chronic dialysis ).

Li Fraumeni syndrome

most commonly associated with p53 mutation of this tumor suppressor gene 1. bone or soft tissue sarcoma in person before age 45 2. 1st degree relative with that beofer 45 3. 1st or 2nd degree relative with sarcoma at any age or any cancer before 45

Osteomyelitis

mostly hematogenou spread, rarely trauma. most of hhematogenous reason is Staph aureus. elevated WBCs, ESR, Crp DDx: septic arthritis, rheumatic fever, Ewing's sarcoma Tx: Medical: IV antibiotics after obtaining culture specimen. monitor temp, swelling, pain, joint motility and WBCs Surgical: open drainage of abscess if antibioitcs fail or signs of abscess appear. After surgical drainage, wound is left open to heal by secondary intention. *Chronic osteomyelitis often seen in extremities of diabetic patients usually polymicrobial Tx: extensive debrdiement of all necrotic and granulation tissue along with reconstruction of bone and soft tissue defects with concomitant antibiotics

CKD, patho, Tx:

nephrons are lost and remaining healthy ones try to compensate by increasing their GFR. This damages them as well. You decrease syntehsis of activeaed VIt D, ammonia, and erythropoeitin. usually asymptomatic until GFR<30 Tx: ACE-inhibitors may slow progression, treat reversible causes, diet: protein restriction, dialysis (decision is CLINICAL, no absolute lab value that requires dialysis)

Shock Types

shock is imbalance between tissue substrate delivery and tissue susbtrate demand Perfusion determined by -CO= SV x HR -SVR= MAP -CVP/CO -BP=CO x SVR shock -Sv02 is one of things you first look at i think to differentiate types of shock -in addition to hypovolemic shock, burns can cause distributive shock. burns can be pretty complicated -decreased pulse pressure means stroke volume is down. you have an increase in SVR to maintain blood pressure. 3 major types of shock: hypovolemic, distributive (septic, anaphylactic, neurogenic) HYPOVOLEMIC -CO decreased, SVR increased, CVP decreased, Sv02 decreased -early on patients will have cool skin, tachycardia, orthostatic hypotension -as it progresses have decreased pulse pressure, become confused, and have cold, clammy skin due to "clamping down" of peripheral vessels via increased sympathetic tone -Severity: ----class 1. compensated: loss of <15% of ciculating blood volume. little clinical manifestations. ---- class 2. partially compensated: loss of 15-30% of blood volume. tachy, orho hyptension, decreased pulse pressure, oliguria ---- class 3. uncompensated: loss of 30-40% of blood volume. hypotension, oliguria, marked tachy, confusion ---- class 4. life threatenig. loss of >40% circulating blood. *of vital organs, the first "casualty" of hypovolemic or cardiogenic shock is the kidneys, as blood is shunted away from the constricted renal arteries. Therefore, it is crucial to monitor for renal fialure. As adequate urine output is one of the crucial signs that the treatment is adequate. Tx: -Rapid inital FLUID resuscitation. Crystalloids (normal saline /lactated ringers) infusion via two large-bore peripheral IVs is best for volume repletion. ----Large-volume NS infusion may result in hyperchloremic acidosis. Therefore, LR (containing alternative anions to Cl-) is the preferred choice. ----Colloid vs isotonic crystalloid: No evidence that colloid fluids improve mortality in critically ill patients -Replacement of blood if hemorrhage is cause. A masssive blood transufion is defined as greater than 10 units of packed RBCs over 24 hours. When you give so much blodd you can develop dilutional thrombocytopenia and deficiences in factors V and VIII. So you should give FFP and platelets along with packed RBCs at a 1:1 ratio. DISTRIBUTIVE SHOCK -a family of shock that are caused by severe decrease SVR. Include septic shock, neurogenic shock, and anaphylactic shock. These patients will have warm skin from vasodilation. 1. Septic shock: infeciton vessels dilate and leak, causing hyptension refractory to fluid recuscitation. ----fever, tachypnea, warm skin and full peripheral pulses (from vasodilation), normal urine output ----positive blood cultures SIRS-->Sepsis--> Severe sepsis-->Septic shock -Tx: fluids!, start broad spectrum antibiotics eraly and empirically treat until blood cultures come back *Gram negative bacteria are notoroiusfor causing septic shock 2. Anaphylactic shock: systemic type 1 hypersenstivity causing angioedema, increased vascular permeability, resulting in hypotension and/or airway compromise. 3. Neurogenic shock: CNS injusry causing disruption of sympathetic system, resulting in unopposed vagal outflow and vasodilation. Hypotension and bradycardia (absence of reflex sympathetic tachycardia and vasoconstriction). -All decresaed: CO, SVR, CVP, Sv02 Tx: IV fluids and vasopressors --phenylephrine is often used as first line CARDIOGENIC SHOCK -pump failure decreased cardiac output. Wedge pressure and SVR are elevated. -CO decreased, SVR increased, CVP increased, Sv02 decreased -Cold, clammy skin from peripheral vasoconstriction. Tx: correct electrolyte abnormalities. pain control. also treatment of cardiogenic ause is specific to cause. OBSTRUCTIVE SHOCK obstructive shock: stuff that doesn't let blood in the hear or out of the heart -cardiac tamponade -tension pneumothorax -massive PE -aortic dissection liver failure, pancreatitis, trauma can all look like sepsis and thus can look like distributive shock. one of hallmarks of distributive shock is shunting of blood away from tissues that need it. so the Sv02 remains high. great assessor of if tissues being perfused: urinaruyu output vasodilator is important to treating a failing heart in cardiogenic shock measure of lactic acidosis in anyone who presents with shock. so this is something you can monitor to see how well the patient is doing.

Stable patient with tibial shaft fracture tx?

surgical fixation with intramedullary nailing.

Urology stuff

tests to consider ordering for initial visit depending on whats wrong PSA, ultrasound,UA, bladder scan, cystoscopy, CT scan, biopsy, tumor markes,

Esophageal hernias (hiatal hernias) types

type I (sliding) hernia: characterized by an upward herniation of the cardia and GE junction in the posterior mediastinum. The most common one. -Tx: endoscopy first to assess what you're dealing with and to exlcude malignancy, etc. - surgical treatment only considered in symptomatic patients with objecteively documented esophagitis or stensosi. Overwhelming majority of these are asymptomatic. Symptomatic ones should be treated vigorously by variety of medical measures. type II (rolling or paraesophageal) hernia (PEH): characterized by an upward herniation of the gastric fundus alongside a normally positioned cardia. The GE junction is in its normal place. -should be surgically excisesd whenever diagnosed because of substantial risk for bot strangulation and obstruction type III (combined sliding-rolling or mixed) hernia: characterized by an upward herniation of both the cardia and the gastric fundus Tx: endoscopy first to assess what you're dealing with and to exlcude malignancy, etc.

Flail chest

typically aused by 3 or more adjacent rib fractures that break in two places and create an unstable chest wall segment that moves in a pradoxic motion with respiration. The segment tends to contract during inspiration (normally bulges out) and bluge out during expiration (normally retracts)

Lateral collateral ligament injry

very rare, would be seen in adduciton injury tothe knee. Varus stress test will help with Dx.


Conjuntos de estudio relacionados

unit 10 - political parties and elections

View Set

Biochem II Ch. 26: RNA Metabolism

View Set

T/GEOMETRY - UNIT 4 CONGRUENT TRIANGLES AND QUADRILATERALS

View Set

male and female reproductive system quizzes

View Set